Solución: a) Verdadera. El estado gaseoso es un estado ... · Solución: Aplicando la ecuación de...

47
PROBLEMAS DE QUÍMICA 2º BACHILLERATO − Termodinámica y equilibrio − 26/01/2013 Pág. 1 Licencia Creative Commons 3.0. Autor: Antonio José Vasco Merino C 1.– La reacción 2 H 2 O(ℓ) 2 H 2 (g) + O 2 (g) no es espontánea a 25 ºC. Justifique si las siguientes afirmaciones son verdaderas o falsas. a) La variación de entropía es positiva porque aumenta el número de moles gaseosos. b) Se cumple que K p /K c = R T. c) Si se duplica la presión de H 2 , a temperatura constante, el valor de K p aumenta. d) La reacción es endotérmica a 25 ºC. Solución: a) Verdadera. El estado gaseoso es un estado mucho más desordenado, por lo que la entropía aumenta si hay más moles de sustancias en estado gaseoso. b) Falsa. Como el incremento de moles estequiométricos gaseosos es 3, aplicando la relación entre K p y K c : K p = K c (R T) n = K c (R T) 3 K p /K c = (R T) 3 . c) Falsa. La constante (como indica su propio nombre) es constante y solo depende de la temperatura. d) Verdadera. Aplicando la expresión de la energía libre de Gibbs: G = H T S H = G + T S. Como todos los términos del segundo miembre son positivos, H también lo será por lo que la reacción es endotérmica: H > 0. 3 2.– ¿Por qué se dice que en el cero absoluto de temperatura cualquier reacción exotérmica ha de ser espontánea? Solución: Aplicando la ecuación de la Energía libre de Gibbs, G = H T S, por ser la reacción exotérmica, tendrá un valor de H negativo, y por ser la temperatura absoluta nula, G = H – 0 · S = H < 0. Al ser G negativo, la reacción es espontánea, independientemente del valor de S. 3 3.– A partir de los siguientes datos de energías de ruptura de enlaces (E D ): Molécula H 2 N 2 NH 3 Enlaces H−H N≡N N−H E D (kJ mol –1 ) 436 946 389 estime la entalpía estándar de formación de la molécula de amoníaco. Todos los datos se refieren a condiciones estándar. Solución: La reacción que tiene lugar es: ½ N 2 (g) + 3/2 H 2 (g) NH 3 (g). Para obtener un mol de NH 3 , se tiene que aportar la energía necesaria para romper ½ mol de enlaces N≡N y 3/2 moles de enlaces H−H, mientras que posteriormente se desprende la energía correspondiente a la formación de 3 moles de enlaces N−H. Por tanto: H 0 f [NH 3 (g)] = ½·E(N≡N) + 3/2·E(H–H) − 3·E(N–H) = ½·946 kJ + 3/2·436 kJ − 3·389 kJ H 0 f [NH 3 (g)] = −40 kJ mol –1 . 3 4.– A partir de los valores de las entalpías de formación a 298 K del metanol [CH 3 OH(ℓ)], dióxido de carbono [CO 2 (g)] y agua [H 2 O(ℓ)], que son respectivamente, –238,6 kJ mol –1 , –393,5 kJ mol –1 y –285,8 kJ mol –1 , calcule: a) la entalpía de combustión del metanol, haciendo uso de la Ley de Hess; b) ¿Qué cantidad de calor se desprenderá en la combustión de 150 g de metanol? Datos: M at H = 1 ; C = 12 ; O = 16 Solución: a) La reacción que tiene lugar es: CH 3 OH(ℓ) + 3/2 O 2 (g) CO 2 (g) + 2 H 2 O(ℓ). H 0 c = H 0 f [CO 2 (g)] + 2·H 0 f [H 2 O(ℓ)] − H 0 f [CH 3 OH(ℓ)] H 0 c = −393,5 kJ + 2 · (−285,8 kJ) −(−238,6 kJ) = −726,5 kJ. b) Por ser la variación de entalpía negativa, se desprende el calor (por tanto el valor final ha de ser positivo porque se pregunta calor desprendido). Aplicando factores de conversión: d = 150 g de CH 3 OH · 1 mol CH 3 OH 32 g CH 3 OH · 726,5 kJ 1 mol CH 3 OH = 3405 kJ. 3 5.– Al calentar carbonato de calcio, CaCO 3 , se produce por descomposición óxido de calcio, CaO, y dióxido de carbono, CO 2 . Calcule: a) la entalpía estándar de la reacción de descomposición; b) la cantidad de óxido de calcio, expresada en kg, que se podrá obtener mediante dicha reacción si aplicamos 5000 kJ de energía. Datos: H 0 f (kJ mol –1 ): CaCO 3 (s) = –1209,6 ; CO 2 (g) = –392,2 ; CaO(s) = –635,15 ; M at (g mol –1 ): C = 12 ; O = 16 ; Ca = 40 Solución: a) La reacción que tiene lugar es: CaCO 3 (s) CO 2 (g) + CaO(s), por lo que: H 0 r = H 0 f [CO 2 (g)] + H 0 f [CaO(s)] − H 0 f [CaCO 3 (s)] H 0 r = −392,2 kJ + (−635,15 kJ) −(−1209,6 kJ) = 182,2 kJ. b) Aplicando factores de conversión: = 5000 kJ · 1 mol CaO 182,2 kJ · 56 g CaO 1 mol CaO · 1 kg 1000 g = 1,54 kg de CaO. 3 6.– Calcule la entalpía de descomposición del CaCO 3 en CaO y CO 2 . Como datos se conocen las entalpías de formación de los compuestos, que son: H 0 f (CaCO 3 ) = −1206 kJ mol –1 , H 0 f (CaO) = −635 kJ mol –1 y H 0 f (CO 2 ) = −393 kJ mol –1 . Solución: a) La reacción que tiene lugar es: CaCO 3 (s) CO 2 (g) + CaO(s), por lo que: H 0 r = H 0 f [CO 2 (g)] + H 0 f [CaO(s)] − H 0 f [CaCO 3 (s)] H 0 r = −393 kJ + (−635 kJ) −(−1206 kJ) = 178 kJ. 3 7.– Calcule la entalpía estándar de formación del metanol líquido a partir de los siguientes datos: H 0 f [H 2 O(ℓ)] = −285,5 kJ mol –1 ; H 0 f [CO 2 (g)] = −393,5 kJ mol –1 ; H 0 comb [CH 3 OH(ℓ)] = − 714,4 kJ mol –1 . Solución: La reacción que tiene lugar es: C(s) + 2 H 2 (g) + ½ O 2 (g) CH 3 OH(ℓ). Resolviendo el problema aplicando la Ley de Hess con entalpías de combustión −las entalpías de formación del agua líquida y del dióxido de carbono son idénticas, respectivamente, a las de combustión del hidrógeno gas y del carbono sólido, ya que son el mismo proceso con otro nombre; no se pone la del oxígeno porque es un comburente que no da lugar a un proceso de combustión−: H 0 f [CH 3 OH(ℓ)] = Σ∆H 0 c reac Σ∆H 0 c prod = H 0 c [C(s)] + 2·H 0 c [H 2 (g)] − H 0 c [CH 3 OH(ℓ)] H 0 f [CH 3 OH(ℓ)] = H 0 f [CO 2 (g)] + 2·H 0 f [H 2 O(ℓ)] − H 0 c [CH 3 OH(ℓ)] H 0 f [CH 3 OH(ℓ)] = –393,5 kJ + 2·(–285,5 kJ) − (−714,4 kJ) = −250,1 kJ.

Transcript of Solución: a) Verdadera. El estado gaseoso es un estado ... · Solución: Aplicando la ecuación de...

Page 1: Solución: a) Verdadera. El estado gaseoso es un estado ... · Solución: Aplicando la ecuación de la Energía libre de Gibbs, ∆G = ∆H – T ∆S, por ser la reacción exotérmica,

PROBLEMAS DE QUÍMICA 2º BACHILLERATO − Termodinámica y equilibrio − 26/01/2013 Pág. 1

Licencia Creative Commons 3.0. Autor: Antonio José Vasco Merino

C

1.– La reacción 2 H2O(ℓ) 2 H2(g) + O2(g) no es espontánea a 25 ºC. Justifique si las siguientes afirmaciones son verdaderas o falsas. a) La variación de entropía es positiva porque aumenta el número de moles

gaseosos. b) Se cumple que Kp/Kc = R T. c) Si se duplica la presión de H2, a temperatura constante, el valor de Kp aumenta. d) La reacción es endotérmica a 25 ºC.

Solución: a) Verdadera. El estado gaseoso es un estado mucho más desordenado, por lo que la entropía aumenta si hay más moles de sustancias en estado gaseoso. b) Falsa. Como el incremento de moles estequiométricos gaseosos es 3, aplicando la relación entre Kp y Kc: Kp = Kc (R T)∆n = Kc (R T)3 ⇒ Kp/Kc = (R T)3. c) Falsa. La constante (como indica su propio nombre) es constante y solo depende de la temperatura. d) Verdadera. Aplicando la expresión de la energía libre de Gibbs: ∆G = ∆H − T ∆S ⇒ ∆H = ∆G + T ∆S. Como todos los términos del segundo miembre son positivos, ∆H también lo será por lo que la reacción es endotérmica: ∆H > 0.

3

2.– ¿Por qué se dice que en el cero absoluto de temperatura cualquier reacción exotérmica ha de ser espontánea?

Solución: Aplicando la ecuación de la Energía libre de Gibbs, ∆G = ∆H – T ∆S, por ser la reacción exotérmica, tendrá un valor de ∆H negativo, y por ser la temperatura absoluta nula,

∆G = ∆H – 0 · ∆S = ∆H < 0. Al ser ∆G negativo, la reacción es espontánea, independientemente del valor de ∆S.

3

3.– A partir de los siguientes datos de energías de ruptura de enlaces (ED):

Molécula H2 N2 NH3

Enlaces H−H N≡N N−H

ED (kJ mol–1) 436 946 389

estime la entalpía estándar de formación de la molécula de amoníaco. Todos los datos se refieren a condiciones estándar.

Solución: La reacción que tiene lugar es: ½ N2(g) + 3/2 H2(g) NH3(g). Para obtener un mol de NH3, se tiene que aportar la energía necesaria para romper ½ mol de enlaces N≡N y 3/2 moles de enlaces H−H, mientras que posteriormente se desprende la energía correspondiente a la formación de 3 moles de enlaces N−H. Por tanto: ∆H0

f [NH3(g)] = ½·E(N≡N) + 3/2·E(H–H) − 3·E(N–H) = ½·946 kJ + 3/2·436 kJ − 3·389 kJ ∆H0

f [NH3(g)] = −40 kJ mol–1.

3

4.– A partir de los valores de las entalpías de formación a 298 K del metanol [CH3OH(ℓ)], dióxido de carbono [CO2(g)] y agua [H2O(ℓ)], que son respectivamente, –238,6 kJ mol–1, –393,5 kJ mol–1 y –285,8 kJ mol–1, calcule: a) la entalpía de combustión del metanol, haciendo uso de la Ley de Hess; b) ¿Qué cantidad de calor se desprenderá en la combustión de 150 g de metanol?

DDaattooss:: Mat H = 1 ; C = 12 ; O = 16

Solución: a) La reacción que tiene lugar es: CH3OH(ℓ) + 3/2 O2(g) CO2(g) + 2 H2O(ℓ). ∆H0

c = ∆H0f [CO2(g)] + 2·∆H0

f [H2O(ℓ)] − ∆H0f [CH3OH(ℓ)]

∆H0c = −393,5 kJ + 2 · (−285,8 kJ) −(−238,6 kJ) = −726,5 kJ.

b) Por ser la variación de entalpía negativa, se desprende el calor (por tanto el valor final ha de ser positivo porque se pregunta calor desprendido). Aplicando factores de conversión:

∆𝑄d = 150 g de CH3OH ·1 mol CH3OH32 g CH3OH

· 726,5 kJ

1 mol CH3OH = 3405 kJ.

3

5.– Al calentar carbonato de calcio, CaCO3, se produce por descomposición óxido de calcio, CaO, y dióxido de carbono, CO2. Calcule: a) la entalpía estándar de la reacción de descomposición; b) la cantidad de óxido de calcio, expresada en kg, que se podrá obtener mediante dicha

reacción si aplicamos 5000 kJ de energía. DDaattooss:: ∆H0

f (kJ mol–1): CaCO3(s) = –1209,6 ; CO2(g) = –392,2 ; CaO(s) = –635,15 ; Mat (g mol–1): C = 12 ; O = 16 ; Ca = 40

Solución: a) La reacción que tiene lugar es: CaCO3(s) CO2(g) + CaO(s), por lo que: ∆H0

r = ∆H0f [CO2(g)] + ∆H0

f [CaO(s)] − ∆H0f [CaCO3(s)]

∆H0r = −392,2 kJ + (−635,15 kJ) −(−1209,6 kJ) = 182,2 kJ.

b) Aplicando factores de conversión:

𝑚 = 5000 kJ · 1 mol CaO182,2 kJ ·

56 g CaO1 mol CaO ·

1 kg1000 g = 1,54 kg de CaO.

3

6.– Calcule la entalpía de descomposición del CaCO3 en CaO y CO2. Como datos se conocen las entalpías de formación de los compuestos, que son: ∆H0

f(CaCO3) = −1206 kJ mol–1, ∆H0

f(CaO) = −635 kJ mol–1 y ∆H0f(CO2) = −393 kJ mol–1.

Solución: a) La reacción que tiene lugar es: CaCO3(s) CO2(g) + CaO(s), por lo que: ∆H0

r = ∆H0f [CO2(g)] + ∆H0

f [CaO(s)] − ∆H0f [CaCO3(s)]

∆H0r = −393 kJ + (−635 kJ) −(−1206 kJ) = 178 kJ.

3

7.– Calcule la entalpía estándar de formación del metanol líquido a partir de los siguientes datos: ∆H0

f[H2O(ℓ)] = −285,5 kJ mol–1 ; ∆H0f[CO2(g)] = −393,5 kJ mol–1 ;

∆H0comb[CH3OH(ℓ)] = − 714,4 kJ mol–1.

Solución: La reacción que tiene lugar es: C(s) + 2 H2(g) + ½ O2(g) CH3OH(ℓ). Resolviendo el problema aplicando la Ley de Hess con entalpías de combustión −las entalpías de formación del agua líquida y del dióxido de carbono son idénticas, respectivamente, a las de combustión del hidrógeno gas y del carbono sólido, ya que son el mismo proceso con otro nombre; no se pone la del oxígeno porque es un comburente que no da lugar a un proceso de combustión−: ∆H0

f [CH3OH(ℓ)] = Σ∆H0c reac − Σ∆H0

c prod = ∆H0c [C(s)] + 2·∆H0

c [H2(g)] − ∆H0c [CH3OH(ℓ)]

∆H0f [CH3OH(ℓ)] = ∆H0

f [CO2(g)] + 2·∆H0f [H2O(ℓ)] − ∆H0

c [CH3OH(ℓ)] ∆H0

f [CH3OH(ℓ)] = –393,5 kJ + 2·(–285,5 kJ) − (−714,4 kJ) = −250,1 kJ.

Page 2: Solución: a) Verdadera. El estado gaseoso es un estado ... · Solución: Aplicando la ecuación de la Energía libre de Gibbs, ∆G = ∆H – T ∆S, por ser la reacción exotérmica,

PROBLEMAS DE QUÍMICA 2º BACHILLERATO − Termodinámica y equilibrio − 26/01/2013 Pág. 2

Licencia Creative Commons 3.0. Autor: Antonio José Vasco Merino

3

8.– Calcule la variación de entalpía que tiene lugar en la reacción: C(s) + 2 H2(g) CH4(g), teniendo en cuenta que las entalpías de combustión del carbono, hidrógeno y metano son, respectivamente, –393,5 kJ, –285,8 kJ y –890,4 kJ.

Solución: C(s) + 2 H2(g) CH4(g) ; ∆H0

f [CH4(g)] = Σ∆H0c reac − Σ∆H0

c prod = ∆H0c [C(s)] + 2·∆H0

c [H2(g)] − ∆H0c [CH4(s)]

∆H0f [CH4(g)] = –393,5 kJ + 2·(–285,8 kJ) −(−890,4 kJ) = −74,7 kJ mol–1.

3

9.– Calcule, en condiciones estándar, la entalpía de la reacción: Zn(s) + 2 HCl(ac) ZnCl2(s) + H2(g)

DDaattooss:: ∆H0f(HCI) = −167,2 kJ mol–1 ; ∆H0

f(ZnCl2) = −415,9 kJ mol–1

Solución: a) Resolviendo el problema aplicando la Ley de Hess con entalpías de formación −no se pone ni la del cinc ni la del hidrógeno porque están en su estado natural en condiciones estándar {∆H0

f [Zn(s)] = 0 y ∆H0f [H2(g)] = 0}−:

Zn(s) + 2 HCl(ac) ZnCl2(s) + H2(g); ∆H0

r = ∆H0f [ZnCl2(s)] − 2 · ∆H0

f [HCl(ac)] = (–415,9 kJ) − 2· (−167,2 kJ) = −81,5 kJ.

3

10.– Calcule: a) la entalpía de combustión del etino a partir de los siguientes datos: ∆H0

f [H2O(ℓ)] = –285,8 kJ mol–1 ; ∆H0

f [CO2(g)] = –393,5 kJ mol–1 ; ∆H0f [C2H2(g)] = 227,0 kJ mol–1;

b) la cantidad de calor, a presión constante, desprendida en la combustión de 1 kg de etino. DDaattooss:: Masas atómicas: Mat (g mol–1): H = 1 ; C = 12

Solución: a) Resolviendo el problema aplicando la Ley de Hess con entalpías de formación −no se pone la del oxígeno porque está en su estado natural en condiciones estándar {∆H0

f [O2(g)] = 0}−: C2H2(g) + 5/2 O2(g) 2 CO2(g) + H2O(ℓ); ∆H0

c [C2H2(g)] = 2·∆H0f [CO2(g)] + ∆H0

f [H2O(ℓ)] − ∆H0f [C2H2(g)]

∆H0c [C2H2(g)] = 2·(–393,5 kJ) + (–285,8 kJ) − (227,0 kJ) = −1299,8 kJ.

b) La cantidad de calor a presión constante desprendida es la entalpía con el signo cambiado:

∆𝑄 = 1 kg C2H2 · 1000 g

1 kg · 1 mol C2H2

26 g C2H2 ·

1299,8 kJ1 mol C2H2

≈ 50000 kJ = 50 MJ.

3

11.– Considere la combustión de carbón, hidrógeno y metanol. a) Ajuste las reacciones de combustión de cada sustancia. b) Indique cuáles de los reactivos o productos tienen entalpía de formación nula. c) Escriba las expresiones para calcular las entalpías de combustión a partir de las entalpías

de formación que considere necesarias. d) Indique cómo calcular la entalpía de formación del metanol a partir únicamente de las

entalpías de combustión.

Solución: a) C(s) + O2(g) CO2(g) ; H2(g) + ½ O2(g) H2O(ℓ) ; CH3OH(ℓ) + 3/2 O2(g) CO2(g) + 2 H2O(ℓ). b) Tienen entalpía de formación nula los reactivos o productos que están formados por un solo elemento en su estado natural a 25 ºC, por lo que serían el C(s), el O2(g) y el H2(g). c) 1: ∆H0

c [C(s)] = ∆H0f [CO2(g)] ; 2: ∆H0

c [H2(g)] = ∆H0f [H2O(ℓ)] ;

3: ∆H0c [CH3OH(ℓ)] = ∆H0

f [CO2(g)] + 2·∆H0f [H2O(ℓ)] − ∆H0

f [CH3OH(ℓ)] d) C(s) + ½ O2(g) + 2 H2(g) CH3OH(ℓ) ; ∆H0

f [CH3OH(ℓ)] = ∆H0c [C(s)] + 2·∆H0

c [H2(g)] − ∆H0c [CH3OH(ℓ)]

3

12.– Considere la combustión de tres sustancias: carbón, hidrógeno molecular y etanol. a) Ajuste las correspondientes reacciones de combustión. b) Indique razonadamente cuáles de los reactivos o productos de las mismas tienen entalpía

de formación nula. c) Escriba las expresiones para calcular las entalpías de combustión de cada una de las tres

reacciones a partir de las entalpías de formación. d) Escriba la expresión de la entalpía de formación del etanol en función únicamente de las

entalpías de combustión de las reacciones mencionadas en el primer apartado.

Solución: a) C(s) + O2(g) CO2(g) ; H2(g) + ½ O2(g) H2O(ℓ) ; CH3–CH2OH(ℓ) + 3 O2(g) 2 CO2(g) + 3 H2O(ℓ). b) Tienen entalpía de formación nula los reactivos o productos que están formados por un solo elemento en su estado natural a 25 ºC, por lo que serían el C(s), el O2(g) y el H2(g). c) 1: ∆H0

c [C(s)] = ∆H0f [CO2(g)] ; 2: ∆H0

c [H2(g)] = ∆H0f [H2O(ℓ)] ;

3: ∆H0c [CH3–CH2OH(ℓ)] = 2·∆H0

f [CO2(g)] + 3·∆H0f [H2O(ℓ)] − ∆H0

f [CH3–CH2OH(ℓ)] d) 2 C(s) + ½ O2(g) + 3 H2(g) CH3–CH2OH(ℓ) ; ∆H0

f [CH3–CH2OH(ℓ)] = 2·∆H0c [C(s)] + 3·∆H0

c [H2(g)] − ∆H0c [CH3–CH2OH(ℓ)]

3

13.– Considere la reacción de hidrogenación del propino: CH3−C≡CH + 2 H2 CH3−CH2−CH3.

a) Calcule la entalpía de la reacción, a partir de las entalpías medias de enlace. b) Determine la cantidad de energía que habrá que proporcionar a 100 g de hidrógeno

molecular para disociarlo completamente en sus átomos. DDaattooss:: Entalpías de enlace en kJ mol–1: (C−C) = 347 ; (C≡C) = 830 ; (C−H) = 415 ; (H−H) = 436 ;

Mat (g mol–1): H = 1

Solución: La reacción que tiene lugar es: CH3−C≡CH + 2 H2 CH3−CH2−CH3. a) Para que tenga lugar esta reacción se tiene que aportar la energía necesaria para romper 1 mol de enlaces C−C, 4 moles de enlaces H−C, 1 mol de enlaces C≡C y 2 moles de enlaces H−H, mientras que posteriormente se desprende la energía correspondiente a la formación de 2 moles de enlaces C−C y 8 moles de enlaces H−C [1 (C≡C), 2 (H−H) 1 (C−C), 4 (H−C)]. Por tanto: ∆H = E(C≡C) + 2·E(H−H) − E(C−C) + 4·E(H−C) ∆H = 830 kJ + 2·436 kJ − 347 kJ − 4·415 kJ = −305 kJ. b) La energía se aporta, puesto que se rompen los enlaces. Aplicando factores de conversión:

∆𝑄 = 100 g H2 · 1 mol H2

2 g H2 ·

436 kJ1 mol H2

≈ 21800 kJ.

Page 3: Solución: a) Verdadera. El estado gaseoso es un estado ... · Solución: Aplicando la ecuación de la Energía libre de Gibbs, ∆G = ∆H – T ∆S, por ser la reacción exotérmica,

PROBLEMAS DE QUÍMICA 2º BACHILLERATO − Termodinámica y equilibrio − 26/01/2013 Pág. 3

Licencia Creative Commons 3.0. Autor: Antonio José Vasco Merino

3

14.– Conteste a las siguientes preguntas: a) Calcule la variación de entalpía de formación del amoniaco, a partir de los siguientes datos

de energías de enlace: E(H–H) = 436 kJ mol–1 ; E(N–H) = 389 kJ mol–1 ; E(N≡N) = 945 kJ mol–1.

b) Calcule la variación de energía interna en la formación del amoniaco a la temperatura de 25 ºC.

DDaattooss:: R = 8,31 J K–1 mol–1

Solución: a) La reacción que tiene lugar es: N2(g) + 3 H2(g) 2 NH3(g). Para obtener dos moles de NH3, se tiene que aportar la energía necesaria para romper 1 mol de enlaces N≡N y 3 moles de enlaces H−H, mientras que posteriormente se desprende la energía correspondiente a la formación de 6 moles de enlaces N−H. Por tanto: 2· ∆H0

f [NH3(g)] = 1 · E(N≡N) + 3 · E(H–H) − 6 · E(N–H) = 945 kJ + 3 · 436 kJ − 6 · 389 kJ ∆H0

f [NH3(g)] = −40,5 kJ mol–1. b) En la formación de 1 mol de NH3 inicialmente hay 2 moles de gases (1/2 mol de N2 y 3/2 mol de H2). Aplicando el Primer Principio de la Termodinámica: ∆U = ∆Q + W = ∆H − ∆n R T = −40 kJ mol–1 −(−1 mol) · 8,31 J K–1 mol–1· 298 K ≈ −38 kJ mol–1.

3

15.– Cuando se quema 1,0 g de ácido etanoico en condiciones estándar, se desprenden 14,5 kJ. Determine, razonadamente: a) el valor de la entalpía de combustión; b) el valor de la entalpía de formación.

DDaattooss:: ∆H0f (CO2) = –394 kJ mol–1 ; ∆H0

f (H2O) = –259 kJ mol–1 ; Masas atómicas: Mat (g mol–1): H = 1,0 ; C = 12,0 ; O = 16,0

Solución: a) Aplicando factores de conversión (teniendo en cuenta que es negativo por ser energía desprendida:

∆𝐻𝑐0 = −1 mol CH3-COOH ·

60 g CH3-COOH1 mol CH3-COOH ·

14,5 kJ1,0 g CH3-COOH = -870 kJ.

b) La reacción de formación del ácido etanoico es: 2 C(s) + O2(g) + 2 H2(g) CH3–COOH(ℓ); Las entalpías de formación del agua líquida y del dióxido de carbono son idénticas, respectivamente, a las de combustión del hidrógeno gas y del carbono sólido, ya que son el mismo proceso con otro nombre. Por tanto, se puede resolver el problema con entalpías de combustión: ∆H0

f [CH3–COOH(ℓ)] = 2·∆H0c [C(s)] + 2·∆H0

c [H2(g)] − ∆H0c [CH3–COOH(ℓ)]

∆H0f [CH3–COOH(ℓ)] = 2·∆H0

f [CO2(g)] + 2·∆H0f [H2O(ℓ)] − ∆H0

c [CH3–COOH(ℓ)] ∆H0

f [CH3–COOH(ℓ)] = 2·(–394 kJ) + 2·(–259 kJ) − (–870 kJ) = −436 kJ.

3

16.– Determine la entalpía de reacción para el proceso: C3H4(g) + 2 H2(g) C3H8(g) DDaattooss:: Entalpías estándar: de combustión del C3H4(g) = –1937 kJ mol–1 ; de combustión del C3H8(g) = –2219 kJ mol–1 ; de formación del H2O(ℓ) = –286 kJ mol–1

Solución: La entalpía de formación del agua líquida es idéntica a la de combustión del hidrógeno gas, ya que es el mismo proceso con otro nombre. Por tanto, se puede resolver el problema con entalpías de combustión: ∆H0 = ∆H0

c [C3H4(g)] + 2·∆H0c [H2(g)] − ∆H0

c [C3H8(g)] ∆H0 = ∆H0

c [C3H4(g)] + 2·∆H0f [H2O(ℓ)] − ∆H0

c [C3H8(g)] ∆H0 = –1937 kJ + 2·(–286 kJ) − (–2219 kJ) = −290 kJ.

3

17.– Determine la variación de energía interna de un sistema que: a) absorbe 150 J de calor y hace un trabajo de 100 J; b) desprende 300 J y hace un trabajo de 0,15 kJ.

Solución: a) Aplicando el Primer Principio de la Termodinámica sabiendo que las energías absorbidas son positivas y las desprendidas negativas: a) ∆U = ∆Q + W = 150 J + (−100 J) = 50 J. b) ∆U = ∆Q + W = −300 J + (−0,15 kJ · 1000 J kJ−1) = −450 J.

3

18.– El benceno (C6H6) se puede obtener a partir del acetileno (C2H2) según la reacción siguiente: 3 C2H2(g) C6H6(ℓ). Las entalpías de combustión, a 25 ºC y 1 atm, para el acetileno y el benceno son, respectivamente, –1300 kJ mol–1 y –3267 kJ mol–1. a) Calcule ∆H0 de la reacción de formación del benceno a partir del acetileno y deduzca si es

un proceso endotérmico o exotérmico. b) Determine la energía (expresada en kJ) que se libera en la combustión de 1 gramo de

benceno. DDaattooss:: Masas atómicas: Mat (g mol–1): H = 1,0 ; C = 12,0

Solución: a) Calculamos primero ∆H0 a partir de las entalpías de combustión: ∆H0

c = Σ ∆H0c (reac) – Σ ∆H0

c (prod) = 3 · (–1300 kJ) − (–3267 kJ) = –633 kJ mol–1. El proceso es exotérmico ya que la variación de entalpía es negativa (se desprende energía). b) Aplicando los factores de conversión necesarios (la energía desprendida es positiva):

∆𝐸 = 1 g de C6H6 · 1 mol de C6H6

78 g de C6H6 ·

3267 kJ1 mol de C6H6

= 41,9 kJ.

3

19.– El CaCO3(s) se descompone térmicamente para dar CaO(s) y CO2(g). a) Calcule el cambio de entalpía en kJ cuando en la reacción se producen 48,02 g de CO2. b) Razone la espontaneidad de una reacción química en función de los posibles valores

positivos o negativos de ∆H y ∆S. DDaattooss:: Masas atómicas: Mat (g mol–1): C = 12,0 ; O = 16,0 ; ∆H0

f [CaO(s)] = –635,6 kJ mol–1 ; ∆H0

f [CO2(g)] = –393,5 kJ mol–1 ; ∆H0f [CaCO3(s)] = –1206,9 kJ mol–1

Solución: a) La reacción que tiene lugar es: CaCO3(s) CO2(g) + CaO(s), por lo que, aplicando la Ley de Hess con calores de formación: ∆H0

r = ∆H0f [CO2(g)] + ∆H0

f [CaO(s)] − ∆H0f [CaCO3(s)]

∆H0r = −393,5 kJ + (−635,6 kJ) −(−1206,9 kJ) = 177,8 kJ.

Aplicando ahora factores de conversión con los datos del problema:

∆𝐻 = 48,02 g CO2 ·1 mol CO2

44,0 g CO2 ·

177,8 kJ1 mol CO2

= 194,0 kJ.

b) La espontaneidad la mide la variación de energía libre de Gibbs: ∆G = ∆H – T ∆S; el proceso es espontáneo cuando ∆G < 0. El valor de ∆H es positivo, como hemos calculado en el apartado anterior y el valor de ∆S debe ser positivo ya que entre los productos se encuentra un gas (estado de agregación mucho más desordenado que el sólido). Por tanto, ∆G será positivo a bajas temperaturas y negativo a altas temperaturas . El proceso será espontáneo a altas temperaturas.

Page 4: Solución: a) Verdadera. El estado gaseoso es un estado ... · Solución: Aplicando la ecuación de la Energía libre de Gibbs, ∆G = ∆H – T ∆S, por ser la reacción exotérmica,

PROBLEMAS DE QUÍMICA 2º BACHILLERATO − Termodinámica y equilibrio − 26/01/2013 Pág. 4

Licencia Creative Commons 3.0. Autor: Antonio José Vasco Merino

3

20.– El carbonato de calcio(s) se descompone térmicamente en óxido de calcio(s) y dióxido de carbono(g). a) Calcule, a partir de los datos que se le dan, el calor de la reacción de descomposición. b) Calcule el calor absorbido o desprendido en el proceso si se obtienen 12,8 g de óxido de

calcio. DDaattooss:: Masas atómicas: Mat (g mol–1): O = 16,0 ; Ca = 40,0 ; ∆H0

f [CaO(s)] = –633 kJ mol–1 ; ∆H0

f [CO2(g)] = –393 kJ mol–1 ; ∆H0f [CaCO3(s)] = –1207 kJ mol–1

Solución: a) La reacción que tiene lugar es: CaCO3(s) CO2(g) + CaO(s), por lo que: ∆H0

r = ∆H0f [CO2(g)] + ∆H0

f [CaO(s)] − ∆H0f [CaCO3(s)]

∆H0r = −393 kJ + (−633 kJ) −(−1207 kJ) = 181 kJ.

b) Aplicando factores de conversión:

∆𝐻𝑚 = 12,8 g de CaO ·1 mol CaO56,0 g CaO ·

181 kJ1 mol CaO = 41,4 kJ (absorbidos por ser ∆𝐻 positiva).

3

21.– El ciclohexano se puede obtener a partir del benceno a elevadas temperaturas (1000 K) según la siguiente reacción: C6H6(g) + 3 H2(g) C6H12(g). Calcule:

a) la variación de entalpía de esta reacción de hidrogenación, sabiendo que los calores de combustión del benceno (C6H6) y del ciclohexano (C6H12) son respectivamente −3312,06 kJ mol–1 y −3964,06 kJ mol–1. El calor estándar de formación del agua es de −241,60 kJ mol–1.

b) Si quemamos 1 gramo de benceno o 1 gramo de ciclohexano, ¿cuál de los dos compuestos libera mayor cantidad de energía?

Solución: a) Resolviendo el problema aplicando la Ley de Hess con entalpías de combustión −las entalpías de formación del agua es idéntica a la de combustión del hidrógeno gas, ya que es el mismo proceso con otro nombre−: ∆H0

r = Σ∆H0c reac − Σ∆H0

c prod = ∆H0c [C6H6(g)] + 3·∆H0

c [H2(g)] − ∆H0c [C6H12(g)]

∆H0r = ∆H0

c [C6H6(g)] + 3·∆H0f [H2O] − ∆H0

c [C6H12(g)] ∆H0

r = –3312,06 kJ + 3·(–241,60 kJ) − (−3964,06 kJ) = −72,80 kJ. b) Aplicando factores de conversión con los datos de las entalpías de combustión (negativas por ser energía desprendida):

∆𝐸 = 1 g de C6H6 · 1 mol de C6H6

78 g de C6H6 ·

3312,06 kJ1 mol de C6H6

= 42,5 kJ.

∆𝐸 = 1 g de C6H12 · 1 mol de C6H12

84 g de C6H12 ·

3964,06 kJ1 mol de C6H6

= 47,2 kJ.

Prácticamente igual, pero libera mayor cantidad de energía el ciclohexano (C6H12).

3

22.– El clorato de potasio (sólido) se descompone. a altas temperaturas, para dar cloruro de potasio (sólido) y oxigeno molecular (gas). Para esta reacción de descomposición, calcule: a) la variación de entalpía estándar; b) la variación de energía de Gibbs estándar; c) la variación de entropía estándar; d) el volumen de oxigeno, a 25 °C y 1 atm, que se produce a partir de 36,8 g de clorato de

potasio.

∆H0f (kJ mol–1) ∆G0

f (kJ mol–1) S0 (J K–1 mol–1)

KClO3(s) –391,2 –289,9 143,0

KCl(s) –435,9 –408,3 82,7

O2 0 0 205,0

DDaattooss:: Masas atómicas: K =39,1 ; Cl = 35,5 ; O = 16,0

Solución: Escribimos y ajustamos la reacción de descomposición: KClO3(s) KCl(s) + 3/2 O2(g) a) Calculamos primero ∆H0

r a partir de las entalpías de formación: ∆H0

r = Σ∆H0f (prod) – Σ∆H0

f (reac) = –435,9 kJ – (–391,2 kJ) = –44,7 kJ. b) Calculamos ∆G0

r utilizando los valores que da el problema: ∆G0

r = Σ∆G0f (prod) – Σ∆G0

f (reac) = –408,3 kJ – (–289,9 kJ) = –118,4 kJ. La reacción, por tanto, a 25 ºC, será espontánea. c) Calculamos después ∆S0

r a partir de las entropías: ∆S0 = ΣS0 (prod) – ΣS0 (reac) = 82,7 + 3/2·205,0 – 143,0 J K–1 = 247,2 J K–1. También se podía haber calculado, sin tener en cuenta los datos de ∆G0

f de la tabla aplicando:

∆𝐺r0 = ∆𝐻r

0 − 𝑇∆𝑆r0 ⇒ ∆𝑆r

0 =∆𝐻r

0 − ∆𝐺r0

𝑇 =[−44,7 − (−118,4)] kJ mol-1

298 K = 247 J K-1mol-1. d) Aplicando factores de conversión:

𝑉 =𝑛 𝑅 𝑇

𝑝

𝑉 =�36,8 g KClO3 · 1 mol KClO3

122,6 g KClO3· 1,5 mol O2

1 mol KClO3� · 0,082 atm L

mol K · 298 K

1 atm = 11,0 L de O2.

Page 5: Solución: a) Verdadera. El estado gaseoso es un estado ... · Solución: Aplicando la ecuación de la Energía libre de Gibbs, ∆G = ∆H – T ∆S, por ser la reacción exotérmica,

PROBLEMAS DE QUÍMICA 2º BACHILLERATO − Termodinámica y equilibrio − 26/01/2013 Pág. 5

Licencia Creative Commons 3.0. Autor: Antonio José Vasco Merino

3

23.– El etanol y el dimetiléter son dos isómeros de función, cuyas entalpías de formación son ∆H0

f (etanol) = −235 kJ mol–1 y ∆H0f (dimetiléter) = −180 kJ mol–1.

a) Escriba las reacciones de formación y de combustión de ambos compuestos. b) Justifique cuál de las dos entalpías de combustión de estos compuestos es mayor en valor

absoluto, teniendo en cuenta que los procesos de combustión son exotérmicos.

Solución: a) 2 C(s) + ½ O2(g) + 3 H2(g) CH3−CH2OH(ℓ); 2 C(s) + ½ O2(g) + 3 H2(g) CH3−O−CH3(g); CH3−CH2OH(ℓ) + 3 O2(g) 2 CO2(g) + 3 H2O(ℓ); CH3−O−CH3(g) + 3 O2(g) 2 CO2(g) + 3 H2O(ℓ). b) Comparando las expresiones de sus entalpías de combustión basadas en la Ley de Hess con entalpías de formación: ∆H0

c [CH3−CH2OH(ℓ)] = 2·∆H0f [CO2(g)] + 3·∆H0

f [H2O(ℓ)] − ∆H0f [CH3−CH2OH(ℓ)]

∆H0c [CH3−O−CH3(g)] = 2·∆H0

f [CO2(g)] + 3·∆H0f [H2O(ℓ)] − ∆H0

f [CH3−O−CH3(g)] Como solo difieren en las entalpías de formación respectivas (con signo negativo) tendrá mayor entalpía de combustión en valor absoluto la que tenga el valor más negativo de esta [todos los proceso de combustión son exotérmicos (∆H0

c < 0)], por lo que será la que tenga menor valor de entalpía de formación, ya que el término a sumar será menor. Por lo tanto la mayor entalpía de combustión en valor absoluto le corresponde al dimetiléter. ∆H0

c[CH3OCH3(g)]−∆H0c[CH3CH2OH(ℓ)] = ∆H0

f [CH3CH2OH(ℓ)]−∆H0f [CH3OCH3(g)] < 0.

3

24.– El método de Berthelot para la obtención de benceno (C6H6) consiste en hacer pasar acetileno (etino) a través de un tubo de porcelana calentado al rojo.

a) Escriba y ajuste la reacción de obtención. b) Determine la energía (expresada en kJ) que se libera en la combustión de 1 gramo de

benceno. c) Calcule ∆H0 de la reacción de formación del benceno a partir del acetileno.

DDaattooss:: Mat (g mol–1): H = 1 ; C = 12 ; Entalpias de combustión (kJ mol–1): Acetileno: −1300 ; Benceno: −3270

Solución: a) La reacción que tiene lugar es: 3 C2H2(g) C6H6(ℓ). b) Aplicando factores de conversión con los datos de la entalpía de combustión (negativa por ser energía desprendida):

∆𝐸 = 1 g de C6H6 · 1 mol de C6H6

78 g de C6H6 ·

3270 kJ1 mol de C6H6

= 41,9 kJ.

∆H0f [C6H6(ℓ)] = 3·∆H0

c [C(s)] + 7·∆H0c [H2(g)] − ∆H0

c [C6H14(ℓ)] ∆H0

f [C6H14(ℓ)] = 6·(–393,1 kJ) + 7·(–285,8 kJ) − (–4192,0 kJ) = −167,2 kJ b) Aplicando la Ley de Hess con entalpías de combustión: ∆H0

r = Σ ∆H0c (reac) – Σ ∆H0

c (prod) = 3 · (–1300 kJ) − (–3270 kJ mol–1) = –630 kJ mol–1.

3

25.– El proceso de fotosíntesis se puede representar por la ecuación química siguiente: 6 CO2(g) + 6 H2O(ℓ) C6H12O6(s) + 6 O2(g) ; ∆H0 = +3402,8 kJ. Calcule:

a) la entalpía de formación estándar de la glucosa, C6H12O6; b) la energía necesaria para la formación de 500 g de glucosa mediante fotosíntesis.

DDaattooss:: Mat (g mol–1): H = 1 ; C = 12 ; O = 16 ; ∆H0f [CO2(g)]= −393,5 kJ mol–1 ;

∆H0f [H2O(ℓ)] = −285,8 kJ mol–1

Solución: a) Aplicando a la reacción dada la Ley de Hess con entalpías de formación −no se pone la del oxígeno porque está en su estado natural en condiciones estándar {∆H0

f [O2(g)] = 0}−: ∆H0

r = ∆H0f [C6H12O6(s)] − {6· ∆H0

f [CO2(g)] + 6·∆H0f [H2O(ℓ)]}

∆H0f [C6H12O6(s)] = ∆H0

r + 6· ∆H0f [CO2(g)] + 6·∆H0

f [H2O(ℓ)] ∆H0

f [C6H12O6(s)] = 3402,8 kJ + 6·(–393,5 kJ) + 6·(−285,8 kJ) = −673 kJ. b) La energía necesaria para la formación (que hay que aportar puesto que ∆H es positiva) es la entalpía:

∆𝐸 = 500 g de C6H12O6· 1 mol C6H12O6

180 g C6H12O6 ·

3402,8 kJ1 mol C6H12O6

≈ 9452 kJ.

3

26.– En el proceso de descomposición térmica del carbonato de calcio se forma óxido de calcio y dióxido de carbono. Sabiendo que el horno en el que ocurre el proceso tiene un rendimiento del 65 %, conteste a los siguientes apartados. a) Formule la reacción y calcule su variación de entalpía. b) Calcule el consumo de combustible (carbón mineral), en toneladas, que se requiere para

obtener 500 kg de óxido cálcico. DDaattooss:: ∆H0

f(carbonato de calcio) = –1206,9 kJ mol–1 ; ∆H0f (óxido de calcio) = –393,1 kJ mol–1 ;

∆H0f (dióxido de carbono) = –635,1 kJ mol–1 ; 1 kg de carbón mineral desprende 8330 kJ ; Masas

atómicas: Mat (g mol–1): O = 16 ; Ca = 40

Solución: a) La reacción que tiene lugar es: CaCO3(s) CO2(g) + CaO(s), por lo que: ∆H0

r = ∆H0f [CO2(g)] + ∆H0

f [CaO(s)] − ∆H0f [CaCO3(s)]

∆H0r = −393,1 kJ + (−635,1 kJ) −(−1206,9 kJ) = 178,7 kJ.

b) Aplicando factores de conversión:

𝑚 = 500 kg obtenidos de CaO · 1000 g

1 kg ·1 mol CaO56 g CaO ·

178,7 kJ1 mol CaO ·

1 kg carbón8330 kJ ·

1 t1000 kg

𝑚 = 0,192 t ; 𝑚′ = 𝑚 · 100 % necesario65 % obtenido = 0,295 t (necesarias) de carbón.

Page 6: Solución: a) Verdadera. El estado gaseoso es un estado ... · Solución: Aplicando la ecuación de la Energía libre de Gibbs, ∆G = ∆H – T ∆S, por ser la reacción exotérmica,

PROBLEMAS DE QUÍMICA 2º BACHILLERATO − Termodinámica y equilibrio − 26/01/2013 Pág. 6

Licencia Creative Commons 3.0. Autor: Antonio José Vasco Merino

3

27.– En la reacción de combustión del metanol líquido se produce CO2(g) y H2O(ℓ). Sabiendo que el metanol tiene una densidad de 0,79 g cm–3, calcule: a) la entalpía estándar de combustión del metanol líquido; b) la energía desprendida en la combustión de 1 L de metanol; c) el volumen de oxígeno necesario para la combustión de 1 L de metanol, medido a 37 ºC y

5 atm. DDaattooss:: R = 0,082 atm L mol–1 K–1 ; Masas atómicas: C = 12 ; O = 16 ; H = 1 ; Entalpías estándar de formación en kJ mol–1: metanol(ℓ) = −239 ; CO2(g) = −393 ; H2O(ℓ) = −294

Solución: a) La reacción que tiene lugar es: CH3OH(ℓ) + 3/2 O2(g) CO2(g) + 2 H2O(ℓ). Resolviendo el problema aplicando la Ley de Hess con entalpías de formación −no se pone la del oxígeno porque está en su estado natural en condiciones estándar {∆H0

f [O2(g)] = 0}−: ∆H0

c [CH3OH(ℓ)] = ∆H0f [CO2(g)] + 2·∆H0

f [H2O(ℓ)] − ∆H0f [CH3OH(ℓ)]

∆H0c [CH3OH(ℓ)] = –393 kJ + 2·(–294 kJ) − (−239 kJ) = −742 kJ.

b) La energía desprendida es la entalpía con el signo cambiado:

∆𝐸 = 1 L de CH3OH · 790 g de CH3OH

1 L de CH3OH · 1 mol CH3OH32 g CH3OH ·

742 kJ1 mol CH3OH ≈ 18000 kJ = 18 MJ.

c) Aplicando factores de conversión:

𝑉 =𝑛 𝑅 𝑇

𝑝

𝑉 =�1 L · 790 g CH3OH

1 L CH3OH · 1 mol CH3OH32 g CH3OH · 1,5 mol O2

1 mol CH3OH� · 0,082 atm Lmol K · 310 K

5 atm = 188 L de O2.

3

28.– En una reacción química del tipo 3 A(g) A3(g) disminuye el desorden del sistema. El diagrama entálpico del sistema es el de la figura adjunta.

a) ¿Qué signo tiene la variación de entropía del sistema? b) Indique razonadamente si el proceso indicado puede ser espontáneo a temperaturas altas o

bajas. c) ¿Qué signo habría de tener la variación de entalpía de la reacción para que esta no fuese

espontánea a ninguna temperatura?

Solución: a) Como el enunciado dice que disminuye el desorden del sistema y el desorden lo mide la entropía, la variación de la entropía del sistema es negativa. De todas formas, aunque no nos hubieran dado el dato de la disminución del desorden, la transformación de 3 moles gaseosos en un solo mol gaseoso suele conllevar un aumento del orden del sistema. b) La espontaneidad la mide la variación de energía libre de Gibbs. Una reacción es espontánea cuando ∆G < 0. Como ∆H es negativo (se ve en la gráfica) e ∆S es negativo, el resultado global, ∆G = ∆H – T ∆S, será positivo a altas temperaturas (ya que el término entrópico sería mayor en valor absoluto) y negativo a bajas temperaturas (donde el término entálpico tiene mayor valor absoluto). Por tanto será espontáneo a bajas temperaturas. c) Un valor positivo de ∆H haría que ambos términos (entálpico y entrópico) actuaran aumentando el valor positivo de ∆G por lo que la reacción nunca sería espontánea.

3

29.– Indique, justificándolo, si las siguientes proposiciones son ciertas o falsas. a) "En un proceso reversible, la variación de energía libre de Gibbs siempre es

negativa". b) "Un proceso con la variación de entalpía positiva y la variación de entropía

negativa puede no ser espontánea". c) "Cuando un gas se disuelve en un líquido, la variación de entropía es menor que

cero". d) "La entalpía de formación del agua líquida es un proceso en el cual se desprenden

285,8 kJ mol–1; por tanto, todas las entalpías de formación son exotérmicas".

Solución: a) Es falso. En un proceso reversible la variación de energía interna ha de ser nula para que puedan tener lugar los dos procesos, lo que implica que para ninguno de los dos la energía libre de Gibbs sea positiva. Una variación de energía libre de Gibbs negativa implica que el proceso es espontáneo. b) Verdadero. Siempre es no espontánea ya que el factor entálpico de la energía libre (∆H) es positivo y el factor entrópico (−T ∆S) también. c) Verdadero. Un gas que se disuelve en un líquido es una situación más ordenada que el gas libre por todo el volumen de un recipiente o de la atmósfera. Está más confinado dentro del líquido por lo que el orden es mayor. Como la entropía mide el estado de desorden, el proceso tiene una variación de entropía negativa. d) Falso. Que una entalpía de formación sea negativa (se desprende energía) no implica que todas las entalpías de formación sean negativas, ya que en la formación se forman enlaces pero también se rompen enlaces. La anilina, el benceno, el cianuro de hidrógeno, el eteno, el monóxido de nitrógeno, etc… son ejemplos de compuestos con entalpías de formación positivas.

Page 7: Solución: a) Verdadera. El estado gaseoso es un estado ... · Solución: Aplicando la ecuación de la Energía libre de Gibbs, ∆G = ∆H – T ∆S, por ser la reacción exotérmica,

PROBLEMAS DE QUÍMICA 2º BACHILLERATO − Termodinámica y equilibrio − 26/01/2013 Pág. 7

Licencia Creative Commons 3.0. Autor: Antonio José Vasco Merino

3

30.– La combustión de mezclas hidrógeno−oxígeno se utiliza en algunas operaciones industriales cuando es necesario alcanzar altas temperaturas. Teniendo en cuenta la reacción de combustión del hidrógeno en condiciones estándar, H2(g) + ½ O2(g) H2O(ℓ); ∆H0

1 = −285,8 kJ y la reacción de condensación del vapor de agua en condiciones estándar, H2O(g) H2O(ℓ); ∆H0

2 = −44,0 kJ, calcule: a) la entalpía de combustión del hidrógeno cuando da lugar a la formación de vapor de agua:

H2(g) + ½ O2(g) H2O(g); ∆H03;

b) la cantidad de energía en forma de calor que se desprenderá al quemar 9,0 g de hidrógeno, H2(g), y 9,0 g de oxígeno, O2(g), si el producto de la reacción es vapor de agua.

DDaattooss:: Mat (g mol–1): H = 1 ; O = 16

Solución: a) La reacción pedida la podemos obtener como suma de las que nos dan ya que: H2(g) + ½ O2(g) H2O(ℓ) ∆H0

1 = −285,8 kJ (Reacción 1) H2O(ℓ) H2O(g) −∆H0

2 = 44,0 kJ (Reacción 2 invertida) H2(g) + ½ O2(g) H2O(g) ∆H0

1 − ∆H02 = −285,8 kJ + 44,0 kJ = −241,8 kJ

b) Hay que calcular quién actúa como reactivo limitante:

9,0 g H · 1 mol H2

2 g H · 1 mol de producto

1 mol H2 = 4,5 mol de producto

9,0 g O · 1 mol O2

32 g O · 1 mol de producto

12� mol O2

= 0,56 mol de producto.

Por tanto el oxígeno es el reactivo limitante. La cantidad de calor a presión constante desprendida es la entalpía con el signo cambiado:

∆𝑄 = 9,0 g O · 1 mol O2

32 g O · 241,8 kJ

12� mol O2

≈ 136 kJ.

3

31.– La levadura y otros microorganismos fermentan la glucosa a etanol y dióxido de carbono: C6H12O6(s) 2 C2H5OH(ℓ) + 2 CO2(g).

a) Aplicando la Ley de Hess, calcule la entalpía estándar de la reacción. b) Calcule la energía desprendida en la obtención de 4,6 g de etanol a partir de glucosa. c) ¿Para qué temperaturas será espontánea la reacción? Razone la respuesta.

DDaattooss:: Entalpías de combustión estándar (kJ mol–1): glucosa = –2813 ; etanol = –1367 ; Mat (g mol−1): H = 1 ; C = 12 ; O = 16

Solución: a) Aplicando la Ley de Hess con entalpías de combustión (la del CO2(g) es nula puesto que ya es la combustión natural del carbono): ∆H0

r = Σ ∆H0c (reac) – Σ ∆H0

c (prod) = (–2813 kJ) − 2 · (–1367 kJ) = −79 kJ. b) Aplicando factores de conversión con los datos de la entalpía de la reacción (se desprende energía por ser ∆H negativa):

∆𝐸 = 4,6 g de C2H5OH · 1 mol de C2H5OH46 g de C2H5OH ·

79 kJ2 mol de C2H5OH = 3,9 kJ.

c) Como la variación de entalpía es negativa, y positivo ∆S (por ser la sustancia inicial un sólido y los productos varios moles de líquido y gas, lo que aumenta notablemente el desorden), a cualquier temperatura ∆H − T ∆S es negativa, por lo que la reacción siempre será espontánea.

3

32.– La reacción de obtención del gas de síntesis (mezcla de CO y H2), producto base para la obtención industrial de metanol, se puede expresar mediante la reacción: CH4(g) + H2O(g) CO(g) + 3 H2(g). Empleando los datos de la tabla adjunta, calcule: a) ∆H0 de la reacción a 298 K; b) ∆G0 de la reacción a 298 K; c) la temperatura a partir de la cual la reacción es espontánea.

CH4(g) H2O(g) CO(g) H2(g)

∆H0f (kJ mol–1) –74,85 –238,92 –110,52 –

S0 (J mol–1 K–1) 186,20 188,72 197,91 130,68

Solución: a) Como la reacción ya está ajustada pasamos a calcular ∆H0r a partir de las entalpías de

formación (no se pone la del hidrógeno puesto que es un elemento en su estado natural): ∆H0

r = Σ∆H0f (prod) – Σ∆H0

f (reac) = –110,52 kJ – [(–74,85 kJ) + (–238,92 kJ)] = 203,25 kJ. b) Para calcular ∆G0

r primero debemos calcular el valor de ∆S0r a partir de las entropías:

∆S0 = ΣS0 (prod) – ΣS0 (reac) = 197,91 + 3·130,68 – (186,20 + 188,72) J K–1 = 215,03 J K–1. ∆G0

r = ∆H0r – T ∆S0

r = 203,25 kJ – 298 K·(215,03 J K–1)·10–3 kJ J–1 = 139,17 kJ. c) Para calcular la temperatura a partir de la cual la reacción es espontánea, volvemos a utilizar la expresión de la energía libre, dándole un valor negativo:

∆𝐺r0 = ∆𝐻r

0 − 𝑇∆𝑆r0 < 0 ⇒ 𝑇 >

∆𝐻r0

∆𝑆r0 =

203,25 kJ215,03 J K-1 · 10−3 kJ J-1 = 945 K.

Por tanto será espontánea a temperaturas superiores a 945 K (672 ºC).

3

33.– Para la reacción de descomposición del peróxido de hidrógeno para dar agua y oxígeno a 298 K, a) calcule ∆H0 y ∆S0 estándar de la reacción; b) razone si el peróxido de hidrógeno será estable a 298 K.

DDaattooss:: ∆H0f (kJ mol–1): H2O(ℓ) = – 285,8 ; H2O2(ℓ) = –187,8 ; S0 (kJ K–1 mol–1): H2O(ℓ) = 69,9 ;

H2O2(ℓ) = 109,6 ; O2(g) = 205,1

Solución: a) La reacción que tiene lugar es: H2O2(ℓ) H2O(ℓ) + 1/2 O2(g). Aplicando la Ley de Hess para los calores de formación (el del oxigeno es nulo por ser su estado natural): ∆H0

r = Σ∆H0f prod − Σ∆H0

f reac = −285,8 kJ −(−187,8 kJ) = −98 kJ. El incremento de entropía se resuelve de forma similar: ∆S0

r = Σ∆S0prod − Σ∆S0

reac = 69,9 + ½·205,1 −109,6 J K–1 mol–1 = 62,8 J K–1 mol–1. b) La espontaneidad la mide la variación de energía libre de Gibbs: ∆G = ∆H – T ∆S, que dice que el proceso es espontáneo cuando ∆G < 0. En la descomposición del agua oxigenada ∆H es negativo y ∆S positivo, por lo que ambos términos, entrópico y entálpico, son negativos a todas las temperaturas por lo que el proceso será siempre espontáneo. Evidentemente a 298 K también, por lo que no es estable a esa temperatura.

Page 8: Solución: a) Verdadera. El estado gaseoso es un estado ... · Solución: Aplicando la ecuación de la Energía libre de Gibbs, ∆G = ∆H – T ∆S, por ser la reacción exotérmica,

PROBLEMAS DE QUÍMICA 2º BACHILLERATO − Termodinámica y equilibrio − 26/01/2013 Pág. 8

Licencia Creative Commons 3.0. Autor: Antonio José Vasco Merino

3

34.– Para la reacción PCl5 PCl3 + Cl2, calcule: a) la entalpía y la energía Gibbs de reacción estándar a 298 K; b) la entropía de reacción estándar a 298 K; c) la temperatura a partir de la cual la reacción es espontánea en condiciones estándar; d) cuál es el valor de la entropía molar del Cl2.

DDaattooss:: A 298 K. ∆H0f (kJ mol–1): PCl5: −374,9; PCl3: −287,0 ; ∆G0

f (kJ mol–1): PCl5: –305,0; PCl3: −267,8 ; S0 (J mol–1 K−1): PCl5: 365; PCl3: 312

Solución: a) Como la reacción ya está ajustada calculamos ∆H0r aplicando la ley de Hess con las

entalpías de formación (la del cloro es nula por ser su estado estándar): ∆H0

r = Σ∆H0f prod − Σ∆H0

f reac = –287 kJ mol–1 – (–374,9 kJ mol–1) = 87,9 kJ mol–1. Como la variación de energía libre del cloro es nula por ser su estado estándar: ∆G0

r = Σ∆G0f prod − Σ∆G0

f reac = –267,8 kJ mol–1 – (–305,0 kJ mol–1) = 37,2 kJ mol–1. b) Para hallar la entropía de reacción estándar, y dado que no tenemos el dato de la entropía de estándar del Cl2, tenemos que utilizar el valor de ∆G0

r y luego calcular ∆S0r:

∆𝐺r0 = ∆𝐻r

0 − 𝑇∆𝑆r0 ⇒ ∆𝑆r

0 =∆𝐻r

0 − ∆𝐺r0

𝑇 =(87,9 − 37,2) kJ mol-1

298 K = 170 J K-1mol-1. c) Para hallar la temperatura a la cual es espontánea utilizamos la ecuación de la energía libre (no cambiamos el sentido de la desigualdad por ser ∆S0 positiva):

0 > ∆𝐻0 − 𝑇∆𝑆0 ⇒ 𝑇 >∆𝐻0

∆𝑆0 =-87,9 kJ mol-1·103 J kJ-1

170 J K-1mol-1 = 517 K.

d) Como la variación total de entropía es el incremento de las entropías: ∆S0 = Σ∆S0

prod – Σ∆S0reac ⇒ ∆S0

Cl2 = ∆S0 + Σ∆S0reac − Σ∆S0

prod menos Cl2 ∆S0

Cl2 = 170 + 365 – 312 J mol–1 K–1 = 223 J mol–1 K–1.

3

35.– Para la siguiente reacción: CH4(g) + 4 Cl2(g) CCl4(g) + 4 HCl(g), calcule la entalpía de reacción estándar utilizando: a) las entalpías de enlace; b) las entalpías de formación estándar.

DDaattooss:: Entalpías de enlace en kJ mol–1: (C−H) = 415 ; (Cl−Cl) = 244 ; (C−Cl) = 330 ; (H−Cl) = 430 ; ∆H0

f CH4(g) = −74,9 kJ mol–1 ; ∆H0f CCl4(g) = −106,6 kJ mol–1 ; ∆H0

f HCl(g) = −92,3 kJ mol–1.

Solución: La reacción que tiene lugar es: CH4(g) + 4 Cl2(g) CCl4(g) + 4 HCl(g). a) Para que tenga lugar se tiene que aportar la energía necesaria para romper 4 moles de enlaces H−C y 4 moles de enlaces Cl−Cl, mientras que posteriormente se desprende la energía correspondiente a la formación de 4 moles de enlaces C−Cl y 4 moles de enlaces H−Cl. Por tanto: ∆H0 = 4·E(H−Cl) + 4·E(Cl−Cl) − 4·E(C−Cl) + 4·E(H−Cl) ∆H0 = 4·415 kJ + 4·244 kJ − 4·330 kJ − 4·430 kJ = −404 kJ. b) Aplicando la Ley de Hess (de formación) a la ecuación termoquímica planteada: ∆H0 = Σ∆H0

f prod − Σ∆H0f reac =∆H0

f [CCl4(g)]+4·∆H0f [HCl(g)]−3·∆H0

f [CH4(g)] − 4·∆H0f [Cl2(g)]

∆H0 = −106,6 kJ + 4·(−92,3 kJ) − (−74,9 kJ) − 4·0 = −400,9 kJ..

3

36.– Para la siguiente reacción: CH3–CH2OH(ℓ) + O2(g) CH3–COOH(ℓ) + H2O(ℓ), calcule: a) la variación de la entalpía de la reacción a 25 °C, en condiciones estándar; b) la variación de la entropía a 25 °C, en condiciones estándar; c) la variación de energía de Gibbs a 25 °C, en condiciones estándar; d) la temperatura teórica para la que la energía de Gibbs es igual a cero.

Datos a 25 °C ∆H0f (kJ mol–1) S0 (J mol–1 K–1)

Etanol(ℓ) –227,6 160,7

Ácido etanoico(ℓ) –487,0 159,9

O2(g) 0 205,0

H2O(ℓ) –285,8 70,0

Solución: a) Como la reacción ya está ajustada calculamos ∆H0r aplicando la ley de Hess con las

entalpías de formación (la del oxígeno es nula por ser su estado estándar): ∆H0

r = Σ∆H0f prod − Σ∆H0

f reac = (–487,0) + (−285,8) – (–227,6) kJ = −545,2 kJ mol–1. b) El incremento de entropía se resuelve de forma similar: ∆S0

r = Σ∆S0prod − Σ∆S0

reac = 159,9 + 70,0 − (160,7 + 205,0) J K–1 = −135,8 J K–1 mol–1. c) Calculamos ∆G0

r = ∆H0r – T ∆S0 (cambiando las unidades necesarias):

∆G0r = −545,2 kJ mol–1 – 298 K·(−135,8 J mol–1 K–1)·10–3 kJ J–1 = –504,7 kJ mol–1.

d) Para hallar la temperatura a la cual la energía libre es nula (límite entre la espontaneidad y la no espontaneidad):

0 = ∆𝐻0 − 𝑇∆𝑆0 ⇒ 𝑇 =∆𝐻0

∆𝑆0 =-545,2 kJ mol-1·103 J kJ-1

-135,8 J K-1mol-1 = 4015 K.

3

37.– Razone si son correctas o incorrectas las siguientes afirmaciones: a) En una reacción química no puede ser nunca ∆G = 0. b) ∆G es independiente de la temperatura. c) La reacción no es espontánea si ∆G > 0. d) La reacción es muy rápida si ∆G < 0.

Solución: a) Incorrecta. ∆G = 0 implica una reacción reversible en el equilibrio. b) Incorrecta. Depende de la temperatura ya que en la expresión de la energía libre de Gibbs esta está multiplicando a la variación de entropía: ∆G = ∆H – T ∆S. c) Correcta. La espontaneidad implica que ∆G < 0. d) Incorrecta. La velocidad de una reacción depende de la energía de activación y no de ∆G. Hay reacciones espontáneas que transcurren muy lentas, como muchos de los procesos de oxidación o la conversión del diamante en grafito, entre otras.

Page 9: Solución: a) Verdadera. El estado gaseoso es un estado ... · Solución: Aplicando la ecuación de la Energía libre de Gibbs, ∆G = ∆H – T ∆S, por ser la reacción exotérmica,

PROBLEMAS DE QUÍMICA 2º BACHILLERATO − Termodinámica y equilibrio − 26/01/2013 Pág. 9

Licencia Creative Commons 3.0. Autor: Antonio José Vasco Merino

3

38.– Sabiendo que la combustión de 1 g de TNT libera 4600 kJ y considerando los valores de entalpías de formación que se proporcionan, calcule: a) la entalpía estándar de combustión del CH4; b) el volumen de CH4, medido a 25 °C y 1 atm de presión, que es necesario quemar para

producir la misma energía que 1 g de TNT. DDaattooss:: ∆H0

f (CH4) = –75 kJ mol–1 ; ∆H0f (CO2) = –394 kJ mol–1 ; ∆H0

f [H2O(g)] = –242 kJ mol–1

Solución: a) La reacción que tiene lugar es: CH4(g) + 2 O2(g) CO2(g) + 2 H2O(ℓ). ∆H0

c = ∆H0f [CO2(g)] + 2·∆H0

f [H2O(ℓ)] − ∆H0f [CH3OH(ℓ)]

∆H0c = −394 kJ + 2·(−242 kJ) −(−75 kJ) = −803 kJ.

Hay un error en el planteamiento, ya que da la entalpía estándar de formación del H2O(g) cuando a 25 ºC y 1 atm el agua está en estado líquido. La respuesta, por tanto, no sería la entalpía estándar de formación sino una entalpía de formación. b) Por ser la variación de entalpía negativa, se desprende el calor (por tanto el valor final ha de ser positivo porque se pregunta calor desprendido). Aplicando factores de conversión:

𝑉 =𝑛 𝑅 𝑇

𝑝 = �1 g TNT · 4600 kJ

1 g TNT · 1 mol CH4803 kJ � · 0,082 atm L

mol K · 298 K

1 atm = 140 L de CH4.

3

39.– Sabiendo que las ∆H0 de formación del CO(g) y la del CH3OH(ℓ) son, respectivamente, −−110,5 kJ mol–1 y −239,0 kJ mol–1 y que la entropía estándar del CO(g) es 197,5 J K–1 mol–1, la del H2(g) es 130,5 J K–1 mol–1 y la del CH3OH(ℓ) es 127,0 J K–1 mol–1, calcule si a 25 ºC el proceso CO(g) + 2 H2(g) CH3OH(ℓ) es o no es espontáneo.

Solución: Como la reacción ya está ajustada calculamos ∆H0r aplicando la ley de Hess con las

entalpías de formación (la del hidrógeno es nula por ser su estado estándar): ∆H0

r = Σ∆H0f prod − Σ∆H0

f reac = –239,0 kJ – (–110,5 kJ) = −128,5 kJ. La variación total de entropía es el incremento de las entropías: ∆S0 = Σ∆S0

prod – Σ∆S0reac = 127,0 − (197,5 + 2·130,5) J K–1 = −331,5 J K–1.

Para reconocer la espontaneidad calculamos ∆G0r = ∆H0

r – T ∆S0 (cambiando las unidades necesarias): ∆G0

r = −128,5 kJ – 298 K·(−331,5 J K–1)·10–3 kJ J–1 = –29,7 kJ. Comprobamos si ∆G0 es negativa lo que haría que fuera espontánea. Como en este caso se cumple, la reacción es espontánea a 25 ºC.

3

40.– Sabiendo que las entalpías de combustión del etanol y del ácido etanoico (ácido acético) en condiciones estándar son, respectivamente, –1372,9 kJ mol–1 y –870,5 kJ mol–1 y que las entalpías normales de formación del agua líquida y del dióxido de carbono son respectivamente –285,5 kJ mol–1 y –393,04 kJ mol–1, calcule: a) la entalpía de la reacción correspondiente al proceso:

C2H5OH(ℓ) + O2(g) CH3−COOH(ℓ) + H2O(ℓ); b) la entalpía de formación del etanol.

Solución: a) Resolviendo el problema por entalpías de combustión (la del oxígeno y la del agua no se ponen por ser nulo su valor, ya que el oxígeno no se quema al ser un comburente y el agua es el resultado final de la combustión del hidrógeno). C2H5OH(ℓ) + O2(g) CH3−COOH(ℓ) + H2O(ℓ); ∆H0

r = ∆H0c [C2H5OH(ℓ)] − ∆H0

c [CH3−COOH(ℓ)] = –1372,9 kJ −(–870,5 kJ) = −502,4 kJ. b) La reacción de formación del etanol es: 2 C(s) + ½ O2(g) + 3 H2(g) CH3–CH2OH(ℓ); Las entalpías de formación del agua líquida y del dióxido de carbono son idénticas, respectivamente, a las de combustión del hidrógeno gas y del carbono sólido, ya que son el mismo proceso con otro nombre. Por tanto, se puede resolver el problema con entalpías de combustión: ∆H0

f [CH3–CH2OH(ℓ)] = 2·∆H0c [C(s)] + 3·∆H0

c [H2(g)] − ∆H0c [CH3–CH2OH(ℓ)]

∆H0f [CH3–CH2OH(ℓ)] = 2·∆H0

f [CO2(g)] + 3·∆H0f [H2O(ℓ)] − ∆H0

c [CH3–CH2OH(ℓ)] ∆H0

f [CH3–CH2OH(ℓ)] = 2·(–393,04 kJ) + 3·(–285,5 kJ) − (–1372,9 kJ) = −269,7 kJ.

3

41.– Sabiendo que las entalpías estándar de combustión del hexano líquido, carbono sólido e hidrógeno gas, son de –4192,0, –393,1 y –285,8 kJ mol–1 respectivamente. Calcule: a) la entalpía de formación del hexano líquido a 25 °C; b) el número de moles de hidrógeno consumidos en la formación del hexano líquido cuando

se han liberado 30,0 kJ.

Solución: a) Resolviendo el problema por entalpías de combustión: 6 C(s) + 7 H2(g) C6H14(ℓ); ∆H0

f [C6H14(ℓ)] = 6·∆H0c [C(s)] + 7·∆H0

c [H2(g)] − ∆H0c [C6H14(ℓ)]

∆H0f [C6H14(ℓ)] = 6·(–393,1 kJ) + 7·(–285,8 kJ) − (–4192,0 kJ) = −167,2 kJ

b) Aplicando factores de conversión:

𝑛 = 30,0 kJ · 7 mol H2

167,2 kJ = 1,26 mol H2.

Page 10: Solución: a) Verdadera. El estado gaseoso es un estado ... · Solución: Aplicando la ecuación de la Energía libre de Gibbs, ∆G = ∆H – T ∆S, por ser la reacción exotérmica,

PROBLEMAS DE QUÍMICA 2º BACHILLERATO − Termodinámica y equilibrio − 26/01/2013 Pág. 10

Licencia Creative Commons 3.0. Autor: Antonio José Vasco Merino

3

42.– Teniendo en cuenta los siguientes datos termodinámicos a 298 K, justifique si las siguientes afirmaciones son verdaderas o falsas:

∆H0f (kJ mol–1) ∆G0

f (kJ mol–1)

NO(g) 90,25 86,57

NO2(g) 33,18 51,30

a) La formación de NO a partir de N2 y O2 en condiciones estándar es un proceso endotérmico.

b) La oxidación de NO a NO2 en condiciones estándar es un proceso exotérmico. c) La oxidación de NO a NO2 en condiciones estándar es un proceso espontáneo.

Solución: a) La reacción que tiene lugar es: ½ N2(g) + ½ O2(g) NO(g). Es evidente que el proceso es el de formación del NO(g) por lo que la entalpía de la reacción coincide con la entalpía de formación del NO(g) que es positiva por lo que el proceso es endotérmico. La afirmación es verdadera. b) La reacción que ahora tiene lugar es: NO(g) + ½ O2(g) NO2(g). Resolviendo el problema aplicando la Ley de Hess con entalpías de formación −no se pone la entalpía del oxígeno gas porque está en su estado natural en condiciones estándar −{∆H0

f [O2(g)] = 0}−: ∆H0

r = Σ∆H0f prod − Σ∆H0

f reac = ∆H0f [NO2(g)] − ∆H0

f [NO(g)] = 33,18 kJ – 90,25 kJ = −57,07 kJ. Por ser negativa, el proceso es exotérmico, por lo que también es verdadera. c) Para reconocer la espontaneidad calculamos ∆G0

r de la misma manera que antes hemos calculado ∆H0

f −tampoco se pone la energía libre de Gibbs del oxígeno gas porque está en su estado natural en condiciones estándar −{∆H0

f [O2(g)] = 0}−: ∆G0

r = Σ∆G0f prod − Σ∆G0

f reac = ∆G0f [NO2(g)] − ∆G0

f [NO(g)] = 51,30 kJ – 86,57 kJ = −35,27 kJ. Por ser negativa, el proceso es espontáneo, por lo que también es verdadera.

3

43.– Utilizando los valores que aparecen en la tabla, todos obtenidos a la temperatura de 25 ºC, y considerando la reacción CO(g) + Cl2(g) COCl2(g) a) Calcule ∆S0 de la reacción. b) Calcule ∆H0 de la reacción. c) Calcule ∆G0 de la reacción. d) Razone si la reacción es o no espontánea.

Compuesto S0(J mol–1 K–1) ∆H0f (kJ mol–1)

CO(g) 197,7 –110,4

Cl2(g) 222,8 0,0

COCl2(g) 288,8 –222,8

Solución: a) Como la reacción ya está ajustada calculamos ∆S0r restando las entropías finales (la

de los productos) de las iniciales (las de los reactivos): ∆S0

r = Σ∆S0prod − Σ∆S0

reac = 288,8 − (197,7 + 222,8) J K–1 mol–1 = −131,7 J K–1 mol–1. b) El incremento de entalpía se resuelve de forma similar aplicando la ley de Hess con las entalpías de formación (la del cloro es nula por ser su estado estándar): ∆H0

r = Σ∆H0f prod − Σ∆H0

f reac = (–222,8) – (–110,4) kJ mol–1 = −112,4 kJ mol–1. c) Calculamos ∆G0

r = ∆H0r – T ∆S0 (cambiando las unidades necesarias):

∆G0r = −112,4 kJ mol–1 – 298 K·(−131,7 J mol–1 K–1)·10–3 kJ J–1 = –73,2 kJ mol–1.

d) Para conocer la espontaneidad de una reacción comprobamos si ∆G0 es negativa lo que haría que fuera espontánea. Como en este caso eso se cumple, la reacción es espontánea a 25 ºC.

4

44.– A una hipotética reacción química, A + B C, le corresponde la siguiente ecuación de velocidad: v = k [A] [B]. Indique: a) el orden de la reacción respecto de A; b) el orden total de la reacción; c) las unidades de la constante de la velocidad.

Solución: a) El orden de la reacción con respecto al reactivo A es 1, ya que ese es el exponente de la concentración de A en la ecuación cinética. b) El orden global de la reacción es 2 (1 + 1), por la misma razón anterior. c) Las unidades de la constante de velocidad se obtienen de la ecuación cinética.

⟦𝑘⟧ = �𝑣

[A] [B]� = �mol L-1 s-1

mol L-1· mol L-1� = �mol-1 L s-1�.

4

45.– Considerando el diagrama de energía que se muestra, para la reacción A B + C, conteste razonadamente a las siguientes preguntas: a) ¿Cuál puede ser la causa de la diferencia entre la curva 1 y la 2? b) ¿Para cuál de las dos curvas la reacción transcurre a mayor velocidad? c) ¿Qué les sucederá a las constantes de velocidad de reacción si se aumenta la temperatura? d) ¿La reacción es exotérmica o endotérmica?

Solución: a) Sobre una reacción que sigue la curva 1, la adición de un catalizador da lugar a la curva 2, ya que, al añadirlo, hacemos que la reacción sea más sencilla energéticamente (desciende la energía de activación). b) Al tener menor energía de activación la curva 2 son más rápidas tanto la reacción directa como la inversa. La constante de velocidad es mayor (se puede comprobar con la Ley de Arrhenius):

𝑘 = 𝐴 𝑒− 𝐸a𝑅 𝑇

c) Sucede algo similar a lo acontecido en el apartado anterior. A mayor temperatura, son más rápidas tanto la reacción directa como la inversa. La constante de velocidad es mayor ( también se puede comprobar con la Ley de Arrhenius). d) La reacción es exotérmica porque la variación de entalpía es negativa (la eneregía correspondiente a los productos es menor que la que le corresponde a los reactivos).

Page 11: Solución: a) Verdadera. El estado gaseoso es un estado ... · Solución: Aplicando la ecuación de la Energía libre de Gibbs, ∆G = ∆H – T ∆S, por ser la reacción exotérmica,

PROBLEMAS DE QUÍMICA 2º BACHILLERATO − Termodinámica y equilibrio − 26/01/2013 Pág. 11

Licencia Creative Commons 3.0. Autor: Antonio José Vasco Merino

4

46.– Conteste las siguientes preguntas. a) ¿Cuál es el concepto de velocidad de reacción? b) ¿En qué unidades se expresa? c) ¿Qué factores influyen en la velocidad de reacción? d) ¿Por qué un catalizador aumenta la velocidad de reacción?

Solución: a) Velocidad de una reacción es la velocidad a la que aumenta o disminuye la concentración de cada uno de los compuestos que intervienen en la reacción. Se puede concretar en una expresión común teniendo en cuenta la estequiometría de la reacción para que tenga el mismo valor para todas las sustancias presentes: a A + b B c C + d D.

𝑣 = −1𝑎

d[A]d𝑡 = −

1𝑏

d[B]d𝑡 =

1𝑐

d[C]d𝑡 =

1𝑑

d[D]d𝑡

b) La velocidad se expresa en mol L−1 s−1 (se puede comprobar en la expresión matemática). c) Influyen la naturaleza de los reactivos, su concentración, la temperatura y la presencia de catalizadores. d) Porque disminuye la energía de activación de la reacción (tanto directa como inversa si ambas pueden tener lugar) y por tanto más moléculas de los reactivos o productos pueden tener la energía suficiente para llegar al estado de transición.

4

47.– Dada la reacción elemental: O3(g) + O(g) 2 O2(g), conteste a las siguientes preguntas:

a) ¿Cuáles son los órdenes de reacción respecto a cada uno de los reactivos y el orden total de la reacción?

b) ¿Cuál es la expresión de la ecuación de velocidad? c) Si las unidades de la concentración se expresan en mol L–1 y las del tiempo en segundos,

¿cuáles son las unidades de la constante de velocidad? d) ¿Qué relación existe entre la velocidad de formación de O2 y la de desaparición de O3?

Solución: a) Como la reacción es elemental transcurre en un solo paso por lo que el orden de la reacción coincide con los coeficientes estequiométricos. El orden es 1 tanto frente al ozono como frente al oxígeno monoatómico y la reacción global es de orden 2. b) La expresión de la ecuación de velocidad es:

𝑣 = 𝑘 [O3] [O]. c) La velocidad se expresa en mol L−1 s−1 (siempre son estas las unidades en el SI; lo que puede variar son las unidades de la constante de velocidad).

⟦𝑘⟧ = �𝑣

[O3] [O]� = �mol L-1 s-1

mol L-1· mol L-1� = �mol-1 L s-1�.

d) La velocidad formación o descomposición de un compuesto es la velocidad a la que aumenta o disminuye la concentración de dicho compuesto. Se puede concretar en una expresión común teniendo en cuenta la estequiometría de la reacción para que tenga el mismo valor para todas las sustancias presentes. En este caso:

𝑣 = −11

d[O3]d𝑡 = −

11

d[O]d𝑡 =

12

d[O2]d𝑡

Por tanto O2 aparece al doble de la velocidad con la que desaparece el ozono.

4

48.– En una reacción endotérmica: a) dibuje el diagrama entálpico de la reacción; b) ¿qué es mayor, la energía de activación directa o la inversa? c) ¿cómo afectará al diagrama anterior la adición de un catalizador?

Solución: a) Ver figura adjunta. b) Es mayor la energía de activación directa puesto que la variación de entalpía es positiva y la ecuación que relaciona a las tres es: ∆H = Ea directa − Ea inversa. c) La adición de un catalizador disminuye ambas energías de activación en la misma cantidad (no en la misma proporción) con lo que hace más fácil la transición hacia ambos lados.

4

49.– La ecuación de velocidad para el proceso de reducción de HCrO4– con HSO3

– en medio

ácido es: v = k [HCrO4–

] [HSO3–

]2 [H+

]. a) Indique las unidades de la constante de velocidad (k). b) Indique el orden total de la reacción y los órdenes parciales correspondientes a las tres

especies. c) Explique los factores que influyen en la constante de velocidad de la reacción. d) Indique de qué forma se puede aumentar la velocidad de reacción, sin variar la

temperatura y la composición.

Solución: a) Aplicando la ecuación cinética: 𝑣 = 𝑘 [HCrO4

−] [HSO3−]2 [H+]

⟦𝑘⟧ = �𝑣

[HCrO4−] [HSO3

−]2 [H+]� = �mol L-1 s-1

mol L-1 · mol2 L-2 · mol L-1� = �mol-3 L3 s-1�.

b) El orden global es 4, que es la suma de los órdenes parciales, y es de orden 1 con respecto a HCrO4

– y H+ y de orden 2 con respecto a HSO3–..

c) Teneindo en cuenta la Ley de Arrhenius, 𝑘 = 𝐴 𝑒− 𝐸a𝑅 𝑇 , la constante depende de la temperatura,

de la energía de activación (que se podría variar añadiendo catalizadores), y de otras constantes como A, e y R (que no se pueden modificar). d) Como hemos visto en el apartado anterior, solo puede hacerse variando la energía de activación, lo que implica la adición de catalizadores en la reacción.

Page 12: Solución: a) Verdadera. El estado gaseoso es un estado ... · Solución: Aplicando la ecuación de la Energía libre de Gibbs, ∆G = ∆H – T ∆S, por ser la reacción exotérmica,

PROBLEMAS DE QUÍMICA 2º BACHILLERATO − Termodinámica y equilibrio − 26/01/2013 Pág. 12

Licencia Creative Commons 3.0. Autor: Antonio José Vasco Merino

4

50.– La reacción 2 H2 + 2 NO 2 H2O + N2 se ha estudiado mediante tres experimentos en los que para diferentes concentraciones iniciales de reactivos se ha determinado la velocidad de la reacción. A partir de los resultados que figuran en la tabla, determine la ecuación de velocidad (órdenes parciales y valor de la constante cinética con sus unidades).

[H2]0

mol L–1 [NO]0

mol L–1 Velocidad inicial

mol L–1 s–1

Experimento 1 1,8·10–3 2,1·10–2 5,4·10–5

Experimento 2 3,6·10–3 2,1·10–2 10,8·10–5

Experimento 3 1,8·10–3 6,3·10–2 4,9·10–4

Solución: Partimos de la expresión de la ecuación cinética de la reacción: 𝑣 = 𝑘 [H2]α [NO]β

Comparando los datos del experimento 2 y del experimento 1: 𝑣2

𝑣1=

𝑘 [H2]2α [NO]2

β

𝑘 [H2]1α [NO]1

β ⇒10,8·10-5 mol L-1 s-1

5,4·10-5 mol L-1 s-1 =[H2]2

α

[H2]1α =

(3,6·10-3 mol L-1)α (1,8·10-3 mol L-1)α

2 = 2α ⇒ log 2 = α · log 2 ⇒ α = 1.

Comparando el experimento 3 y el experimento 1: 𝑣3

𝑣1=

𝑘 [H2]3α [NO]3

β

𝑘 [H2]1α [NO]1

β ⇒4,9·10-4 mol L-1 s-1

5,4·10-5 mol L-1 s-1 =[NO]3

β

[NO]1β =

(6,3·10-2 mol L-1)β (2,1·10-2 mol L-1)β

9 = 3β ⇒ log 9 = β · log 3 ⇒ β = log 9log 3 =

log 32

log 3 = 2 · log 3

log 3 = 2.

Para hallar la constante utilizamos uno de los experimentos, por ejemplo, el 1: 𝑣 = 𝑘 [H2] [NO]2 ⇒ 𝑘 =

𝑣[H2] [NO]2

𝑘 = 5,4·10-5 mol L-1 s-1

1,8·10-3 mol L-1· (2,1·10-2 mol L-1)2 = 68 mol-2 L2 s-1.

El orden es 1 con respecto al hidrógeno y 2 con respecto al NO, siendo 3 el orden total.

4

51.– La reacción 2X + Y X2Y tiene órdenes de reacción 2 y 1 respecto a los reactivos X e Y, respectivamente. a) ¿Cuál es el orden total de la reacción? Escriba la ecuación velocidad del proceso. b) ¿Qué relación existe entre la velocidad de desaparición de X y la de aparición de X2Y? c) ¿En qué unidades se puede expresar la velocidad de esta reacción? ¿Y la constante de

velocidad? d) ¿De qué factor depende el valor de la constante de velocidad de esta reacción? Razone la

respuesta.

Solución: a) El orden total es la suma de los órdenes parciales, por lo que el orden total es 3. La ecuación cinética o ecuación de velocidad será:

𝑣 = 𝑘 [X]2 [Y]. b) La relación viene dada por los coeficientes estequiométricos, ya que las velocidades de aparición o desaparición son:

𝑣 = −12

d[X]d𝑡 = −

11

d[Y]d𝑡 =

11

d[X2Y]d𝑡

Por tanto, X desaparece el doble de rápido de lo que aparece X2Y. c) La velocidad de una reacción siempre se expresa en mol L–1 s−1. Las unidades de la constante de velocidad se calculan teniendo en cuenta la ecuación cinética:

𝑣 = 𝑘 [X]2 [Y] ⇒ ⟦𝑘⟧ = �𝑣

[X]2 [Y]� = �mol L-1 s-1

mol2 L-2 · mol L-1� = �mol-2 L2 s-1�.

d) La constante cinética es constante; sólo depende de la temperatura, aumentando cuando aumenta ésta.

4

52.– La reacción en fase gaseosa 2 A + B 3 C es una reacción elemental y por tanto de orden 2 respecto de A y de orden 1 respecto de B. a) Formule la expresión para la ecuación de velocidad. b) Indique las unidades de la velocidad de reacción y de la constante cinética. c) Justifique cómo afecta a la velocidad de reacción un aumento de la temperatura a volumen

constante. d) Justifique cómo afecta a la velocidad de reacción un aumento del volumen a temperatura

constante.

a) Como nos dan los órdenes parciales (que nos dicen que la reacción transcurre de forma elemental), conocemos los exponentes de la ecuación cinética, por lo que:

𝑣 = 𝑘 [A]2 [B]. b) La velocidad de una reacción siempre se expresa en mol L–1 s−1. Las unidades de la constante cinética se calculan teniendo en cuenta la ecuación cinética:

𝑣 = 𝑘 [A]2 [B] ⇒ ⟦𝑘⟧ = �𝑣

[A]2 [B]� = �mol L-1 s-1

mol2 L-2 · mol L-1� = �mol-2 L2 s-1�.

c) Un aumento de la temperatura conlleva siempre un aumento de la velocidad a la que transcurre la reacción ya que hay más choques intermoleculares y las moléculas están dotadas de mayor energía. d) Un aumento de volumen ocasiona que la concentración de las sustancias presentes disminuya, por lo que será más difícil que las moléculas choquen entre sí y, por tanto, disminuye la velocidad de la reacción.

Page 13: Solución: a) Verdadera. El estado gaseoso es un estado ... · Solución: Aplicando la ecuación de la Energía libre de Gibbs, ∆G = ∆H – T ∆S, por ser la reacción exotérmica,

PROBLEMAS DE QUÍMICA 2º BACHILLERATO − Termodinámica y equilibrio − 26/01/2013 Pág. 13

Licencia Creative Commons 3.0. Autor: Antonio José Vasco Merino

4

53.– La reacción en fase gaseosa: 2 NO(g) + O2(g) 2 NO2(g) es uno de los pasos intermedios en la síntesis de abonos nitrogenados. Se trata de una reacción de orden 2 respecto al monóxido de nitrógeno, y de orden 1 respecto al oxígeno.

a) Escriba la ecuación de velocidad para esta reacción. Indique, razonadamente, las unidades con las que se expresan la velocidad y la constante de velocidad de esta reacción.

b) ¿Qué sucede con la velocidad de la reacción si aumentamos la temperatura y mantenemos constante el volumen? ¿Y si aumentamos el volumen y mantenemos constante la temperatura?

Justifique las respuestas.

Solución: a) Como nos dan los órdenes parciales (que nos dicen que la reacción transcurre de forma elemental), conocemos los exponentes de la ecuación cinética, por lo que:

𝑣 = 𝑘 [NO]2 [O2]. La velocidad de una reacción siempre se expresa en mol L–1 s−1. Las unidades de la constante de velocidad se calculan teniendo en cuenta la ecuación cinética:

𝑣 = 𝑘 [NO]2 [O2] ⇒ ⟦𝑘⟧ = �𝑣

[NO]2 [O2]� = �mol L-1 s-1

mol2 L-2 · mol L-1� = �mol-2 L2 s-1�.

d) Al aumentar la temperatura, aumentamos el valor de la constante de velocidad �𝑘 = 𝐴 𝑒− 𝐸a𝑅 𝑇�

por lo que la reacción transcurre más rápidamente (no varían las concentraciones, pero sí lo hace la constante de velocidad). Al aumentar el volumen, manteniendo la misma cantidad de cada sustancia, disminuye la concentración de éstas, por lo que la velocidad de la reacción disminuye, ya que es proporcional a las concentraciones de los reactivos (la constante no varía, al no variar la temperatura, pero la velocidad sí).

4

54.– La velocidad de una reacción entre A y B, a 25 ºC, puede expresarse por: v = k [A]2 [B]. Conteste razonadamente a las siguientes preguntas: a) ¿Cuál es el orden de reacción respecto al compuesto A? ¿Y respecto al compuesto B? b) ¿Afectará un cambio en la temperatura a la velocidad de la reacción? Si la respuesta es

afirmativa, ¿en qué sentido le afectará? c) Si se duplica la concentración de uno solo de los reactivos, A o B, ¿qué ocurrirá con la

velocidad de reacción?

Solución: a) La reacción (viendo los exponentes de las concentraciones en la expresión de la ecuación cinética) es de orden 2 frente al reactivo A y de orden 1 frente al reactivo B. b) La velocidad de reacción es directamente proporcional a la constante cinética que a su vez aumenta cuando aumenta la temperatura (aproximadamente se duplica la velocidad con cada 10 K que asciende la temperatura) según predijo Arrhenius:

𝑘 = 𝐴 𝑒− 𝐸a𝑅 𝑇

c) Aplicando la ecuación cinética:

𝑣 = 𝑘 [A]2 [B] ⇒ 𝑣′

2[A] = 𝑘 (2[A])2 [B] = 𝑘 · 4 · [A]2 [B] = 4 𝑣𝑣′′

2[B] = 𝑘 [A]2 (2[B]) = 𝑘 [A]2 · 2 · [B] = 2 𝑣.

En el caso de que sea la de A, se cuadriplica y en el caso de que sea la de B se duplica.

4

55.– Para el proceso: 2 NO(g) + 2 H2(g) 4 N2(g) + 2 H2O(g), la ecuación de velocidad es: v = k [NO]2 [H2]2. a) Indique el orden de la reacción con respecto a cada uno de los reactivos. b) ¿Cuál es el orden total de la reacción? c) Deduzca las unidades de la constante de velocidad.

Solución: a) La reacción transcurre de forma elemental (ya que los coeficientes estequiométricos coinciden con los exponentes de las concentraciones en la expresión de la ecuación cinética) y el orden es 2 tanto frente al monóxido de nitrógeno como frente al oxígeno. b) El orden global es 4, que es la suma de los órdenes parciales. c) Aplicando la ecuación cinética:

𝑣 = 𝑘 [NO]2 [H2]2 ⇒ ⟦𝑘⟧ = �𝑣

[NO]2 [H2]2� = �mol L-1 s-1

mol2 L-2 · mol2 L-2� = �mol-3 L3 s-1�.

Page 14: Solución: a) Verdadera. El estado gaseoso es un estado ... · Solución: Aplicando la ecuación de la Energía libre de Gibbs, ∆G = ∆H – T ∆S, por ser la reacción exotérmica,

PROBLEMAS DE QUÍMICA 2º BACHILLERATO − Termodinámica y equilibrio − 26/01/2013 Pág. 14

Licencia Creative Commons 3.0. Autor: Antonio José Vasco Merino

4

56.– Para la reacción A + B C se obtuvieron los siguientes resultados:

ENSAYO [A] (mol L–1) [B] (mol L–1) v (mol L–1 s−1)

1º 0,1 0,1 ×

2º 0,2 0,1 2×

3º 0,1 0,2 4×

a) Determine la ecuación de velocidad. b) Determine las unidades de la constante cinética k. c) Indique cuál de los dos reactivos se consume más deprisa. d) Explique cómo se modifica la constante cinética, k, si se añade más reactivo B al sistema.

Solución: a) La ecuación cinética (o ecuación de velocidad) de la reacción es: 𝑣 = 𝑘 [A]α [B]β

Tenemos que determinar el orden de la reacción con respecto a los reactivos. Aunque se puede hacer sin cálculos, optamos por la demostración completa: Comparando los datos del experimento 2 y del experimento 1:

𝑣2

𝑣1=

𝑘 [A]2α [B]2

β

𝑘 [A]1α [B]1

β ⇒2× mol L-1 s-1

× mol L-1 s-1 =[A]2

α

[A]1α =

(0,2 mol L-1)α (0,1 mol L-1)α

2 = 2α ⇒ log 2 = α · log 2 ⇒ α = 1.

Comparando el experimento 3 y el experimento 1: 𝑣3

𝑣1=

𝑘 [A]3α [B]3

β

𝑘 [A]1α [B]1

β ⇒4× mol L-1 s-1

× mol L-1 s-1 =[B]3

β

[B]1β =

(0,2 mol L-1)β (0,1 mol L-1)β

4 = 2β ⇒ log 4 = β · log 2 ⇒ β = log 4log 2 =

log 22

log 2 = 2 · log 2

log 2 = 2.

𝑣 = 𝑘 [A] [B]2. b) Para hallar las unidades de la constante utilizamos la ecuación cinética:

𝑣 = 𝑘 [A] [B]2 ⇒ ⟦𝑘⟧ = �𝑣

[A] [B]2� = �mol L-1 s-1

mol L-1 · mol2 L-2� = �mol-2 L2 s-1�.

c) La velocidad de consumo de los reactivos viene dada por los coeficientes estequiométricos, ya que la velocidad de aparición o desaparición es:

𝑣 = −11

d[A]d𝑡 = −

11

d[B]d𝑡 =

11

d[C]d𝑡

Por tanto, ambos reactivos se consumen a la misma velocidad. d) La constante cinética (como cualquier constante) es constante, por lo que no varía con la concentración de B. Lo que varía es la velocidad de la reacción que sí depende de [B]. La constante de velocidad sólo depende de la temperatura.

4

57.– Para la reacción en fase gaseosa: CO + NO2 CO2 + NO la ecuación de velocidad es v = k [NO2]2, justifique si son verdaderas o falsas las siguientes afirmaciones. a) "La velocidad de desaparición del CO es igual que la velocidad de desaparición del

NO2". b) "La constante de velocidad no depende de la temperatura porque la reacción se produce

en fase gaseosa". c) "El orden total de la reacción es dos". d) "Las unidades de la constante de velocidad serán mol L–1 s–1".

Solución: a) Verdadera. La expresión que relaciona velocidades de aparición y desaparición es:

𝑣 = −11

d[CO]d𝑡 = −

11

d[NO2]d𝑡 =

11

d[CO2]d𝑡 =

11

d[NO]d𝑡 .

Por tanto las velocidades de aparición y desaparición de todos los reactivos y productos son iguales (no tenemos en cuenta el signo puesto que está implícito en la mención a desaparición). b) Falsa. La temperatura influye en el valor de la cosntante de velocidad como ya dedujo Arrhenius:

𝑘 = 𝐴 𝑒− 𝐸a𝑅 𝑇.

c) Verdadera. El orden total de la reacción (que coincide con el parcial frente al NO2) es dos. d) Falsa. Las unidades de la constante de velocidad, aplicando la ecuación cinética, son:

𝑣 = 𝑘 [NO2]2 ⇒ ⟦𝑘⟧ = �𝑣

[NO2]2� = �mol L-1 s-1

mol2 L-2 � = �mol-1 L1 s-1�.

Page 15: Solución: a) Verdadera. El estado gaseoso es un estado ... · Solución: Aplicando la ecuación de la Energía libre de Gibbs, ∆G = ∆H – T ∆S, por ser la reacción exotérmica,

PROBLEMAS DE QUÍMICA 2º BACHILLERATO − Termodinámica y equilibrio − 26/01/2013 Pág. 15

Licencia Creative Commons 3.0. Autor: Antonio José Vasco Merino

4

58.– Responda, razonadamente, las siguientes cuestiones: a) Escriba las expresiones de velocidad para las siguientes reacciones químicas referidas tanto

a la desaparición de reactivos como a la formación de productos: a.1) 3 O2(g) 2 O3(g). a.2) 4 NO2(g) + O2(g) 2 N2O5(g).

b) En la reacción: 4 NO2(g) + O2(g) 2 N2O5(g), el oxígeno molecular en un determinado momento se está consumiendo con una velocidad de 0,024 M s–1.

b.1) ¿Con qué velocidad se está formando en ese instante el producto N2O5? b.2) ¿Con qué velocidad se está consumiendo, en ese momento, el reactivo NO2?

Solución: a.1) La expresión que relaciona velocidades de aparición y desaparición es:

𝑣 = −13

d[O2]d𝑡 =

12

d[O3]d𝑡 .

a.2) En este caso es:

𝑣 = −14

d[NO2]d𝑡 = −

11

d[O2]d𝑡 =

12

d[N2O5]d𝑡

b.1) Por la expresión del apartado anterior comprobamos que la velocidad a la que aparece N2O5 es el doble de la velocidad a la que desaparece O2 por lo que:

−11

d[O2]d𝑡 =

12

d[N2O5]d𝑡 ⇒

d[N2O5]d𝑡 = 2 ·

d[O2]d𝑡 = 2 · 0,024 M s-1 = 0,048 M s-1.

b.2) Por la expresión del apartado a.2) comprobamos que la velocidad a la que desaparece NO2 es el cuádruple de la velocidad a la que desaparece O2 por lo que:

−14

d[NO2]d𝑡 = −

11

d[O2]d𝑡 ⇒

d[NO2]d𝑡 = 4 ·

d[O2]d𝑡 = 4 · 0,024 M s-1 = 0,096 M s-1.

En ningún caso tenemos en cuenta los signos porque ya están implícitos (todos positivos) en el enunciado.

4

59.– Teniendo en cuenta la gráfica adjunta que debe copiar en su hoja de contestaciones: a) indique si la reacción es exotérmica o endotérmica; b) represente el valor de ∆H de reacción; c) represente la curva de reacción al añadir un catalizador positivo; d) ¿Qué efectos produce el hecho de añadir un catalizador positivo?

Solución: a) Como la energía de los productos es menor que la de los reactivos se ha debido desprender energía en el proceso que es, por tanto, exotérmico. La variación de entalpía es negativa, porque descendemos en el nivel de energía. b) Es la distancia vertical entre la altura de la energía que corresponde a los productos y la de los reactivos con signo. c) Está en la figura adjunta. d) Al añadir un catalizador positivo, hacemos que la reacción sea más sencilla energéticamente (desciende la energía de activación) y por eso son más rápidas tanto la reacción directa como la inversa. La curva es la misma pero con un "pico" más bajo en la energía de activación. No cambia el equilibrio.

4

60.– Una de las aplicaciones del cloroetano durante el siglo XX ha sido la producción de un antidetonante para la gasolina. El cloroetano se hidroliza en una solución caliente de hidróxido de sodio, según la ecuación siguiente: CH3CH2Cl + OH– CH3CH2OH + Cl–. Estudiamos la variación de la velocidad inicial de esta reacción para diferentes concentraciones iniciales de los reactivos, a una temperatura determinada. Los resultados se pueden observar en la tabla siguiente:

Estudio experimental de la cinética de la reacción de hidrólisis del cloroetano

Concentración inicial de cloroetano (mol dm−3) 0,010 0,020 0,020

Concentración inicial de ion hidróxido (mol dm−3) 0,020 0,020 0,060

Velocidad inicial de la reacción (mol dm−3 s−1) 8,60·10−8 1,72·10−7 5,16·10−7

a) Determine el orden de reacción respecto a cada reactivo y el orden total de la reacción. Explique razonadamente las respuestas.

b) Calcule la constante de velocidad de la reacción.

Solución: a) Partimos de la expresión de la ecuación cinética de la reacción: 𝑣 = 𝑘 [CH3CH2Cl]α [OH−]β

Comparando los datos del experimento 2 y del experimento 1: 𝑣2

𝑣1=

𝑘 [CH3CH2Cl]2α [OH−]2

β

𝑘 [CH3CH2Cl]1α [OH−]1

β ⇒1,72·10-7 mol dm-3 s-1

8,60·10-8 mol dm-3 s-1 =[CH3CH2Cl]2

α

[CH3CH2Cl]1α =

(0,020 mol dm-3)α (0,010 mol dm-3)α

2 = 2α ⇒ log 2 = α · log 2 ⇒ α = 1.

Comparando el experimento 3 y el experimento 2: 𝑣3

𝑣2=

𝑘 [CH3CH2Cl]3α [OH−]3

β

𝑘 [CH3CH2Cl]2α [OH−]2

β ⇒5,16·10-7 mol dm-3 s-1

1,72·10-7 mol dm-3 s-1 =[OH−]3

β

[OH−]2β =

(0,060 mol dm-3)β (0,020 mol dm-3)β

3 = 3β ⇒ log 3 = β · log 3 ⇒ β = 1.

Por tanto el orden es 1 con respecto a ambos reactivos y orden total 2. b) Para hallar la constante utilizamos uno de los experimentos, por ejemplo, el 1:

𝑣 = 𝑘[CH3CH2Cl][OH−] ⇒ 𝑘 = 𝑣

[CH3CH2Cl][OH−]

𝑘 = 8,60·10-8 mol dm-3 s-1

0,010 mol dm-3 · 0,020 mol dm-3 = 4,3·10-4 mol-1 dm3 s-1.

4

61.– Usando criterios de termodinámica y/o cinética de las reacciones químicas, critique o justifique la afirmación: “Hay reacciones espontáneas, como la combustión de un papel de celulosa, que sin embargo no se producen salvo que se inicien con una llama o con calor”.

Solución: La espontaneidad de una reacción, estudiada desde el punto de vista de la energía libre de Gibbs (toda reacción es espontánea si su variación de energía libre es negativa), tiene como limitación la energía de activación necesaria para que la reacción tenga lugar, que actúa como límite que la impide hasta que no se aporte desde el exterior la energía necesaria para que se produzca. Por otro lado no hay una limitación cinética importante puesto que, al empezar a producirse la reacción esta transcurre de forma rápida, violenta y, hasta cierto punto, explosiva.

Page 16: Solución: a) Verdadera. El estado gaseoso es un estado ... · Solución: Aplicando la ecuación de la Energía libre de Gibbs, ∆G = ∆H – T ∆S, por ser la reacción exotérmica,

PROBLEMAS DE QUÍMICA 2º BACHILLERATO − Termodinámica y equilibrio − 26/01/2013 Pág. 16

Licencia Creative Commons 3.0. Autor: Antonio José Vasco Merino

5

62.– A 1100 ºC se mezclan en un matraz SO2 y O2 con presiones parciales de 1,00 y 5,00 atm, respectivamente. Si cuando se alcanza el equilibrio la presión total es de 5,55 atm, calcule la Kp para la reacción: 2 SO2(g) + O2(g) 2 SO3(g).

Solución: a) Aplicando la expresión del equilibrio para presiones: 2 SO2(g) + O2(g) 2 SO3(g)

presión inicial 1,00 atm 5,00 atm 0 gastado 2x x 0 formado 0 0 2x

en el equilibrio 1,00 atm − 2x 5,00 atm − x 2x La presión total de todas las sustancias que intervienen es: pT = (1,00 atm − 2x) + (5,00 atm − x) + 2x = 6,00 atm − x = 5,55 atm ⇒ x = 0,45 atm. Por tanto:

en el equilibrio 0,10 atm 4,55 atm 0,90 atm La expresión de la constante de presiones será, aplicando los valores obtenidos:

𝐾𝑝= 𝑝SO3

2

𝑝SO22 𝑝O2

=(0,90 atm)2

(0,10 atm)2· 4,55 atm= 18 atm–1.

5

63.– A 130 ºC el hidrogenocarbonato de sodio, NaHCO3(s), se descompone parcialmente según el siguiente equilibrio: 2 NaHCO3(s) Na2CO3(s) + CO2(g) + H2O(g), Kp = 6,25 a 130 ºC. Se introducen 100 g de NaHCO3(s) en un recipiente cerrado de 2,0 L de capacidad, en el que previamente se ha hecho el vacío y se calienta a 130 ºC. Calcule:

a) el valor de Kc y la presión total en el interior del recipiente cuando se alcance el equilibrio a 130 ºC;

b) la cantidad, en gramos, de NaHCO3(s) que quedará sin descomponer. DDaattooss:: Mat (g mol–1): H = 1 ; C = 12 ; O = 16 ; Na = 23 ; R = 0,082 atm L mol–1 K–1

Solución: a) Aplicando la fórmula que relaciona Kp y Kc (∆n = 2) : Kc = Kp (R T)−∆n = 6,25 atm2 · (0,082 atm L mol–1 K–1 · 403 K)−2 = 5,7·10−3 mol2 L–2. Aplicando la expresión del equilibrio:

2 NaHCO3(s) Na2CO3(s) + CO2(g) + H2O(g) Presiones iniciales 0 0

disminución de presión 0 0 incremento de presión p p

en el equilibrio p p Hallamos el valor de x aplicando la expresión de la constante:

𝐾𝑝= [CO2] [H2O] = 𝑝 · 𝑝 = 6,25 atm2 ⇒ 𝑝 = �6,25 atm2 = 2,5 atm. pT = 2 p = 2 · 2,5 atm = 5,0 atm. b) El bicarbonato de sodio que se ha descompuesto lo ha hecho en la proporción de 2 moles de bicarbonato por cada mol de CO2 formado. Por tanto se han descompuesto:

𝑝 𝑉 = 𝑛 𝑅 𝑇 ; 𝑛NaHCO3 =2 𝑝 𝑉𝑅 𝑇 =

2 · 2,5 atm · 2,0 L0,082 atm L mol–1 K–1 · 403 K = 0,30 mol

Por tanto quedarán sin descomponer: mf = m0 − mg = 100 g − 0,30 mol · 84 g mol−1 ≈ 75 g.

5

64.– A 200 ºC, el PCl5 se encuentra disociado en un 50 %, alcanzándose una presión de 2 atmósferas en el siguiente equilibrio: PCl5(g) PCl3(g) + Cl2(g). Calcule: a) la presión parcial de cada gas en el equilibrio; b) las constantes Kc y Kp a esa temperatura.

DDaattooss:: R = 0,082 atm L mol–1 K–1

Solución: a) Como el enunciado dice que se encuentra disociado en un 50 % (α = 0,5), sabemos que los moles gastados son x = 0,5 n

PCl5(g) PCl3(g) + Cl2(g) moles iniciales n 0 0

gastado x = 0,5 n 0 0 formado 0 x = 0,5 n x = 0,5 n

en el equilibrio 0,5 n 0,5 n 0,5 n El número total de moles en el equilibrio es de: nT = 0,5 n + 0,5 n + 0,5 n = 1,5 n. Las fracciones molares serán:

𝜒PCl5 =𝑛PCl5

𝑛T =

0,5 𝑛1,5 𝑛 = 0,33 ; 𝜒PCl3 =

𝑛PCl3

𝑛T=

0,5 𝑛1,5 𝑛 = 0,33 ; 𝜒Cl2 =

𝑛Cl2

𝑛T=

0,5 𝑛1,5 𝑛 = 0,33.

𝑝PCl5 = 𝜒PCl5 · 𝑝T = 0,33 · 2 atm = 0,67 atm 𝑝PCl3 = 𝜒PCl3 · 𝑝T = 0,33 · 2 atm = 0,67 atm 𝑝Cl2 = 𝜒PCl2 · 𝑝T = 0,33 · 2 atm = 0,67 atm.

b) La expresión de la constante Kp es:

𝐾𝑝 = 𝑝PCl3 𝑝Cl2

𝑝PCl5

=0,67 atm · 0,67 atm

0,67 atm = 0,67 atm.

Aplicando la fórmula que relaciona Kp y Kc: Kc = Kp (R T)–∆n = 0,67 atm · (0,082 atm L mol–1 K–1 · 473 K)–1 = 1,72·10–2 mol L–1.

Page 17: Solución: a) Verdadera. El estado gaseoso es un estado ... · Solución: Aplicando la ecuación de la Energía libre de Gibbs, ∆G = ∆H – T ∆S, por ser la reacción exotérmica,

PROBLEMAS DE QUÍMICA 2º BACHILLERATO − Termodinámica y equilibrio − 26/01/2013 Pág. 17

Licencia Creative Commons 3.0. Autor: Antonio José Vasco Merino

5

65.– A 25 ºC el producto de solubilidad del yodato de bario es 6,5· 10–10. Calcule: a) la solubilidad de la sal expresada en g L–1; b) la concentración molar de los iones yodato y la de los iones bario; c) la solubilidad de la citada sal, expresada en g L–1, en una disolución 0,1 M de yodato

potásico a la misma temperatura. DDaattooss:: Masas atómicas: Ba = 137,3 ; I = 126,9 ; O = 16

Solución: a) El equilibrio que tiene lugar es: Ba(IO3)2(s) ↓ Ba2+(ac) + 2 IO3

–(ac) Conc. iniciales 0 0

gastado 0 0 formado s 2s

en el equilibrio s 2s Aplicando la expresión del producto de solubilidad:

𝐾p𝑠 = [Ba2+][IO3−]2 = 𝑠(2𝑠)2 = 4𝑠3 ⇒ 𝑠 = �

𝐾p𝑠

43

= �6,5·10-10 mol3 L−3

43

= 5,5·10-4 mol L-1

𝑠 = 5,5·10-4 mol L-1 · 487,1 g de Ba(IO3)2

1 mol de Ba(IO3)2= 0,27 g L-1.

b) Sustituyendo s en las concentraciones del equilibrio: Ba(IO3)2(s) ↓ Ba2+(ac) + 2 IO3

–(ac) en el equilibrio s = 5,5·10−4 mol L–1 2s = 1,1·10−3 mol L–1

c) Una disolución de yodato potásico, KIO3, 0,1 M deja en disolución una concentración de iones yodato de 0,1 M. Estableciendo el equilibrio con las nuevas condiciones:

Ba(IO3)2(s) ↓ Ba2+(ac) + 2 IO3–(ac)

Conc. iniciales 0 0,1 M gastado 0 0 formado x 2x

en el equilibrio x 0,1 M + 2x Aplicando la expresión del producto de solubilidad (aproximamos 0,1 M + 2x a 0,1 M ya que, por el valor tan pequeño de la constante x es despreciable frente a 0,1 M):

𝐾p𝑠 = [Ba2+][IO3−]2 = 𝑥 (0,1 M + 2𝑥)2 ≅ 0,01 mol2 L−2 · 𝑥

𝑥 =6,5·10-10 mol3 L−3

0,01 mol2 L−2 = 6,5·10-8 mol L-1 · 487,1 g de Ba(IO3)2

1 mol de Ba(IO3)2= 3,2·10-5 g L-1.

5

66.– A 25 ºC el producto de solubilidad en agua del AgOH es 2,0·10−8. Para esa temperatura, calcule:

a) la solubilidad del compuesto en g L–1; b) la solubilidad del hidróxido de plata en una disolución de pH = 13.

DDaattooss:: Mat (g mol–1): Ag = 108 ; O = 16 ; H = 1

Solución: a) El equilibrio que tiene lugar es: AgOH(s) ↓ Ag+(ac) + OH–(ac)

Conc. iniciales 0 0 gastado 0 0 formado s s

en el equilibrio s s Aplicando la expresión del producto de solubilidad:

𝐾p𝑠 = [Ag+] [OH−] = 𝑠 · 𝑠 = 𝑠2 ⇒ 𝑠 = �𝐾p𝑠 = �2,0·10-8 mol2 L−2 = 1,4·10-4 mol L-1

𝑠 = 1,4·10-4 mol L-1 · 125 g de AgOH1 mol de AgOH = 1,8·10-2 g L-1.

b) Un pH = 13 implica que el pOH vale 1, ya que pH + pOH = 14. Por tanto [OH–] = 0,10 M. Aplicamos la expresión del equilibrio con estos datos (despreciando x frente a 0,1 M ya que su valor es muchísimo menor):

AgOH(s) ↓ Ag+(ac) + OH–(ac) Conc. iniciales 0 0,10 M

gastado 0 0 formado x x

en el equilibrio x 0,10 M + x ≈ 0,10 M Aplicando la expresión del producto de solubilidad:

𝐾p𝑠 = [Ag+] [OH−] = 𝑥 · 0,10 M ⇒ 𝑥 =𝐾p𝑠

0,10 M =2,0·10-8 mol2 L−2

0,10 mol L-1 = 2,0·10-7 mol L-1

𝑠 = 2,0·10-7 mol L-1 · 125 g de AgOH1 mol de AgOH = 2,5·10-5 g L-1.

Page 18: Solución: a) Verdadera. El estado gaseoso es un estado ... · Solución: Aplicando la ecuación de la Energía libre de Gibbs, ∆G = ∆H – T ∆S, por ser la reacción exotérmica,

PROBLEMAS DE QUÍMICA 2º BACHILLERATO − Termodinámica y equilibrio − 26/01/2013 Pág. 18

Licencia Creative Commons 3.0. Autor: Antonio José Vasco Merino

5

67.– A 25 ºC la constante del equilibrio de solubilidad del Mg(OH)2 sólido es Ks = 3’4·10−11. a) Establezca la relación que existe entre la constante Ks y la solubilidad, s, del Mg(OH)2. b) Explique, razonadamente, cómo se podría disolver, a 25 ºC y mediante procedimientos

químicos, un precipitado de Mg(OH)2. c) ¿Qué efecto tendría sobre la solubilidad del Mg(OH)2 a 25 ºC la adición de cloruro de

magnesio? Razone la respuesta.

Solución: a) El equilibrio que tiene lugar es: Mg(OH)2(s) ↓ Mg2+(ac) + 2 OH–(ac)

Conc. iniciales 0 0 gastado 0 0 formado s 2s

en el equilibrio s 2s Aplicando la expresión del producto de solubilidad:

𝐾p𝑠 = [Mg2+][OH−]2 = 𝑠(2𝑠)2 = 4𝑠3 ⇒ 𝑠 = �𝐾p𝑠

43

= �3,4·10-11 mol3 L−3

43

= 2,0·10-4 mol L-1.

Sustituyendo s en las concentraciones del equilibrio: Mg(OH)2(s) ↓ Mg2+(ac) + 2 OH–(ac)

en el equilibrio s = 2,0·10−4 mol L–1 2s = 4,0·10−4 mol L–1 b) Para disolver el precipitado, tendríamos que, siguiendo lo indicado en el Principio de Le Chatelier, eliminar alguno de los productos. Lo más sencillo, en el caso de hidróxidos, es eliminar los iones OH– por lo que, si añadimos un ácido a la disolución, se van neutralizando (eliminando) los OH– que se producen, por lo que el precipitado se disuelve. c) El cloruro de magnesio (MgCl2) se disocia en la disolución por lo que, al aumentar la concentración de Mg2+ presente en la disolución, el equilibrio se desplaza hacia la formación del precipitado por lo que estaríamos disminuyendo la solubilidad de este (efecto de ion común).

5

68.– A 25 ºC la solubilidad del PbI2 en agua pura es 0,70 g L–1. Calcule: a) el producto de solubilidad; b) la solubilidad del PbI2 a esa temperatura en una disolución 0,10 M de KI.

DDaattooss:: Mat (g mol–1): I = 127 ; Pb = 207

Solución: a) Calculamos la solubilidad molar del PbI2:

𝑠 = 0,70 g L-1 · 1 mol de PbI2

461 g de PbI2= 1,5·10-3 mol L-1.

El equilibrio que tiene lugar es: PbI2(s) ↓ Pb2+(ac) + 2 I–(ac)

Conc. iniciales 0 0 gastado 0 0 formado s 2s

en el equilibrio s 2s Aplicando la expresión del producto de solubilidad:

𝐾p𝑠 = [Pb2+][I−]2 = 𝑠(2𝑠)2 = 4𝑠3 = 4 · (1,5·10-3 mol L-1)3 = 1,4·10-8 mol3 L−3. b) Haciendo el nuevo equilibrio:

PbI2(s) ↓ Pb2+(ac) + 2 I–(ac) Conc. iniciales 0 c' = 0,10 M

gastado 0 0 formado x 2x

en el equilibrio x 0,1 M + 2x Aplicando la expresión del producto de solubilidad (teniendo en cuenta que, al ser tan pequeño el valor de Kps el valor de x es despreciable frente a la concentración inicial del ion yoduro):

𝐾p𝑠 = [Pb2+][IO3−]2 = 𝑥 (𝑐' + 2𝑥)2 = 𝑐'2 · 𝑥 = 0,010 mol2 L−2 · 𝑥

𝑥 =𝐾p𝑠

𝑐'2 =1,4·10-8 mol3 L−3

0,010 mol2 L−2 = 1,4·10-6 mol L-1 ·461 g de PbI2

1 mol de PbI2= 6,5·10-4 g L-1 .

5

69.– A 25 ºC y 1 atmósfera, se establece el equilibrio: N2(g) + O2(g) 2 NO(g): ∆H = −180, 2 kJ. Razone sobre la veracidad o falsedad

de las siguientes afirmaciones: a) La constante de equilibrio se duplica si se duplica la presión. b) La reacción se desplaza hacia la izquierda si se aumenta la temperatura. c) Si se aumenta la concentración de NO la constante de equilibrio aumenta.

Solución: a) Falsa. Las constantes son constantes siempre y solo les influye la variación de temperatura. b) Verdadera. La reacción es exotérmica y, al aumentar la temperatura, el equilibrio se desplaza en el sentido de "eliminar" energía, por lo que se desplaza hacia la formación de reactivos. c) Falsa. Como hemos dicho en el primer apartado, las constantes son constantes. Variarán las concentraciones hasta que el valor del cociente de reacción se iguale con la constante.

Page 19: Solución: a) Verdadera. El estado gaseoso es un estado ... · Solución: Aplicando la ecuación de la Energía libre de Gibbs, ∆G = ∆H – T ∆S, por ser la reacción exotérmica,

PROBLEMAS DE QUÍMICA 2º BACHILLERATO − Termodinámica y equilibrio − 26/01/2013 Pág. 19

Licencia Creative Commons 3.0. Autor: Antonio José Vasco Merino

5

70.– A 25 ºC, se prepara una solución saturada de hidróxido de zinc en agua cuyo pH es 8,5. a) Calcule el valor del producto de solubilidad, Kps, del hidróxido de zinc a 25 ºC y la masa

de hidróxido de zinc que se encuentra disuelta en 5 L de una disolución saturada de hidróxido de zinc en agua.

b) Calcule la solubilidad del hidróxido de zinc en una disolución de cloruro de zinc 1,5·10−2 M.

DDaattooss:: Masas atómicas: Mat (g mol–1): H = 1,0 ; O = 16,0 ; Zn = 65,4

Solución: a) Por ser el pH igual a 8,5 el pOH vale 5,5 y la concentración de OH es: [OH–]eq = 10−5,5 ≈ 3,2·10−6 mol L–1. El equilibrio que tiene lugar es:

Zn(OH)2(s) ↓ Zn2+(ac) + 2 OH–(ac) Conc. iniciales 0 0

gastado 0 0 formado s 2s

en el equilibrio s 2s = 3,2·10−6 mol L–1 s = 1,6·10−6 mol L–1 1,6·10−6 mol L–1 3,2·10−6 mol L–1 Aplicando la expresión del producto de solubilidad:

𝐾p𝑠 = [Zn2+][OH−]2 = 𝑠(2𝑠)2 = 4𝑠3 = 4 · (1,6·10-6 mol L-1)3 = 1,6·10-17 mol3 L−3. Para hallar la masa del hidróxido que se encuentra disuelta aplicamos la expresión de c:

𝑐 =𝑛𝑉 =

𝑚𝑀𝑚𝑉 ⇒ 𝑚 = 𝑐 𝑉 𝑀𝑚 = 1,6·10-6 mol L-1· 5,0 L· 99,4 g de Zn(OH)2mol-1 = 8,0·10-4 g.

b) El cloruro de zinc se disocia totalmente según la reacción: ZnCl2(s) Zn2+(ac) + 2 Cl–(ac). Como la proporción entre el sólido y el ion Zn2+ es de 1 a 1, la concentración presente inicial de Zn2+ será: [Zn2+]0 = 1,5·10−2 M. Aplicamos la expresión del equilibrio con estos datos (despreciando x frente a 0,015 M ya que su valor es muchísimo menor):

Zn(OH)2(s) ↓ Zn2+(ac) + 2 OH–(ac) Conc. iniciales 0,015 M 0

gastado 0 0 formado x 2x

en el equilibrio 0,015 M + x ≈ 0,015 M 2x Aplicando la expresión del producto de solubilidad:

𝐾p𝑠 = [Zn2+] [OH−]2 = 0,015 M · (2𝑥)2 = 0,060 M · 𝑥2

𝑥 = �𝐾p𝑠

0,060 M = �1,6·10-17 mol3 L−3

0,060 mol L-1 = 1,6·10-8 mol L-1· 99,4 g de Zn(OH)2

1 mol de Zn(OH)2=1,6·10-6 g L-1.

5

71.– A 298 K, la solubilidad en agua del CaBr2 es 2,0·10−4 mol L–1 . a) Calcule su Kps a esa temperatura. b) Justifique cualitativamente qué efecto tendría en la solubilidad de esta sal la adición de

0,1 mol de KBr a un litro de disolución saturada de CaBr2.

Solución: a) El equilibrio que tiene lugar es: CaBr2(s) ↓ Ca2+(ac) + 2 Br–(ac)

Conc. iniciales 0 0 gastado 0 0 formado s 2s

en el equilibrio s 2s Aplicando la expresión del producto de solubilidad:

𝐾p𝑠 = [Ca2+][Br−]2 = 𝑠(2𝑠)2 = 4𝑠3 = 4 · (2,0·10-4 mol L-1)3 = 3,2·10-11 mol3 L−3. b) El bromuro de potasio (KBr) se disocia en la disolución por lo que, al aumentar la concentración de Br– presente en la disolución, el equilibrio se desplaza hacia la formación del precipitado por lo que estaríamos disminuyendo la solubilidad de este (efecto de ion común).

Page 20: Solución: a) Verdadera. El estado gaseoso es un estado ... · Solución: Aplicando la ecuación de la Energía libre de Gibbs, ∆G = ∆H – T ∆S, por ser la reacción exotérmica,

PROBLEMAS DE QUÍMICA 2º BACHILLERATO − Termodinámica y equilibrio − 26/01/2013 Pág. 20

Licencia Creative Commons 3.0. Autor: Antonio José Vasco Merino

5

72.– A 30 ºC y 1,0 atm el N2O4 se encuentra disociado en un 20 % según el siguiente equilibrio: N2O4(g) 2 NO2(g). Calcule: a) el valor de las constantes Kp y Kc, a esa temperatura; b) el porcentaje de disociación a 30 ºC y 0,10 atm de presión total.

DDaattooss:: R = 0,082 atm L mol–1 K–1

Solución: a) Teniendo en cuenta que α = 20 % = 0,20, representamos la ecuación de equilibrio. N2O4(g) 2 NO2(g)

presiones iniciales p 0 presión de moles gastados x = p · 0,20 = 0,20 p 0 presión de moles formados 0 2x

presión en el equilibrio p − x = 0,80 p 2x = 0,40 p Teniendo en cuenta que la presión dada (la presión total en el equilibrio) es 1,0 atm: pT = 0,80 p + 0,40 p = 1,20 p = 1,0 atm ⇒ p = 0,83 atm.

N2O4(g) 2 NO2(g) presión en el equilibrio 0,80 p = 0,67 atm 0,40 p = 0,33 atm

La constante de presiones será, por tanto, igual a:

𝐾𝑝= pNO2

2

𝑝N2O4

= (0,33 atm) 2

0,67 atm = 0,17 atm.

Aplicando la fórmula que relaciona Kp y Kc: Kc = Kp (R T)–∆n = 0,17 atm · (0,082 atm L mol–1 K–1 · 303 K)–1 = 6,8·10–3 mol L–1. b) Aplicando otra vez el equilibrio con los nuevos datos:

N2O4(g) 2 NO2(g) presiones iniciales p' 0

presión de moles gastados x = p' α 0 presión de moles formados 0 2 p' α

presión en el equilibrio p' − p' α = p' (1 − α) 2 p' α Teniendo en cuenta que la presión dada (la presión total en el equilibrio) es 0,10 atm: pT' = p' (1 − α) + 2 p' α = p' (1 + α) = 0,10 atm ⇒ p' = 0,10 atm / 1 + α.

𝐾𝑝= pNO2

2

𝑝N2O4

= (2 𝑝′ 𝛼) 2

𝑝′(1 − 𝛼) =4 𝑝′𝛼2

(1 − 𝛼) =4 𝑝T

1 + 𝛼 𝛼2

(1 − 𝛼) =4 𝑝T

′𝛼2

1 − 𝛼2 ⇒ 𝐾𝑝 − 𝐾𝑝𝛼2 = 4 𝑝T′𝛼2

𝛼 = �𝐾𝑝

4 𝑝T′ + 𝐾𝑝

= �0,17 atm

4 · 0,10 atm + 0,17 atm = 0,55 = 55 %.

Page 21: Solución: a) Verdadera. El estado gaseoso es un estado ... · Solución: Aplicando la ecuación de la Energía libre de Gibbs, ∆G = ∆H – T ∆S, por ser la reacción exotérmica,

PROBLEMAS DE QUÍMICA 2º BACHILLERATO − Termodinámica y equilibrio − 26/01/2013 Pág. 21

Licencia Creative Commons 3.0. Autor: Antonio José Vasco Merino

5

73.– A 35 ºC La constante Kp para la reacción: N2O4(g) 2 NO2(g) tiene un valor de 0,32. a) Calcule el valor de Kc a la misma temperatura. b) Si se introducen 0,20 moles de NO2 en un recipiente vacío de 10 L y se calienta a 35 ºC,

determine la composición de la mezcla gaseosa y la presión en el interior del recipiente una vez alcanzado el equilibrio.

c) Si posteriormente se reduce el volumen a la mitad, manteniendo constante la temperatura, explique de forma cualitativa el sentido en el que va a evolucionar la reacción.

DDaattooss:: R = 0,082 atm L mol–1 K–1

Solución: a) Aplicando p = c R T, y transformando una en otra obtenemos la relación que es:

𝐾𝑐= [NO2]2

[N2O4] =�

𝑝NO2𝑅 𝑇 �

2

𝑝N2O4𝑅 𝑇

= pNO2

2

𝑝N2O4

(𝑅 𝑇)−∆𝑛 = 𝐾𝑝(𝑅 𝑇)−1

𝐾𝑐 = 0,32 atm

0,082 atm Lmol K · (273+35) K

= 1,27·10-2 mol L-1.

b) Primero calculamos la concentración inicial de dióxido de nitrógeno.

cNO2 = nNO2

V = 0,20 mol

10 L = 0,020 mol L–1

Representamos la ecuación de equilibrio y el proceso para alcanzarlo . (en mol L–1) N2O4(g) 2 NO2(g)

concentración inicial 0 0,020 gastado 0 2x formado x 0

en el equilibrio x 0,020 − 2x La expresión de la constante de concentraciones es:

𝐾𝑐= [NO2]2 [N2O4] =

(0,020 mol L–1 - 2x) 2

𝑥 ⇒ 4𝑥2-(𝐾𝑐+0,080 mol L–1) 𝑥+(0,020 mol L–1) 2= 0.

Resolviendo la ecuación de segundo grado:

𝑥 = 0,0927 M -��-9,27·10–2 M�

2- 4 · 4 · 4,0·10−4 M2

2 · 4 = 5,85·10−3 mol L-1. Aplicando el resultado obtenido a los compuestos que intervienen en el equilibrio: [NO2] = 8,3·10−3 mol L–1 ⇒ nNO2 = 0,083 mol [N2O4] = 5,9·10−3 mol L–1 ⇒ nN2O4 = 0,059 mol. La presión se obtiene aplicando la expresión: p = c R T a la concentración total (suma de las concentraciones parciales) de todas las sustancias gaseosas presentes por lo que: pT = ([NO2] + [N2O4]) R T = 1,4·10−2 mol L–1 · 0,082 atm L mol–1 K–1 · 308 K = 0,36 atm. c) Al reducir el volumen sin variar las cantidades de las sustancias que intervienen en la reacción, se aumentan las concentraciones por lo que el sistema evolucionará hacia el lado del equilibrio que menos moles estequiométricos gaseosos tenga y es el de la formación de N2O4 (hacia la izquierda).

Page 22: Solución: a) Verdadera. El estado gaseoso es un estado ... · Solución: Aplicando la ecuación de la Energía libre de Gibbs, ∆G = ∆H – T ∆S, por ser la reacción exotérmica,

PROBLEMAS DE QUÍMICA 2º BACHILLERATO − Termodinámica y equilibrio − 26/01/2013 Pág. 22

Licencia Creative Commons 3.0. Autor: Antonio José Vasco Merino

5

74.– A 360 ºC se determina la composición de una mezcla gaseosa que se encuentra en equilibrio en el interior de un matraz de dos litros de capacidad, encontrándose 0,10 moles de H2, 0,12 moles de I2 y 0,080 moles de HI. Calcule: a) Kc y Kp para la reacción: H2(g) + I2(g) 2 HI(g); b) la cantidad de hidrógeno que se ha de introducir en el matraz para duplicar el número de

moles de HI, manteniendo constante la temperatura. DDaattooss:: R = 0,082 atm L mol–1 K–1

Solución: a) Lo primero que tenemos que hacer es calcular las concentraciones en el equilibrio:

[H2] =𝑛H2

𝑉 =0,10 mol de H2

2,0 L = 0,050 M ; [I2] =𝑛I2

𝑉 =0,12 mol de I2

2,0 L = 0,060 M

[HI] = 𝑛HI

𝑉 =0,080 mol de HI

2,0 L = 0,040 M.

Hallamos el valor de Kc aplicando la expresión de la constante:

𝐾𝑐= [HI]2

[H2] [I2] = (0,040 mol L–1) 2

(0,50 mol L–1) · (0,60 mol L–1) = 0,53.

Aplicando la fórmula que relaciona Kp y Kc: Kp = Kc (R T)∆n = Kc = 0,53. b) Aplicando los datos que me dan al nuevo equilibrio:

H2(g) + I2(g) 2 HI(g) conc. iniciales 0,050 M + c 0,060 M 0,040 M

gastado x x 0 formado 0 0 2x

en el equilibrio 0,050 M + c − x 0,060 M − x 0,040 M + 2x = 0,080 M Hallamos el valor de x:

0,040 mol L–1 + 2𝑥 = 0,080 mol L–1 ⇒ 𝑥 =0,080 mol L–1-0,040 mol L–1

2 = 0,020 mol L–1. Por tanto las concentraciones en el nuevo equilibrio son:

H2(g) + I2(g) 2 HI(g) en el equilibrio 0,030 M + c 0,040 M 0,080 M Aplicando la expresión de la constante hallamos el valor de c:

𝐾𝑐= [HI]2

[I2] [H2] = (0,080 mol L–1) 2

(0,030 mol L–1 + 𝑐) · 0,040 mol L–1 = 0,53

𝑐 =(0,080 mol L–1) 2

0,53 · 0,040 mol L–1 − 0,030 mol L–1 = 0,27 mol L–1 ⇒ 𝑛 = 𝑐 𝑉 = 0,54 mol de H2 .

No se necesita el valor de R.

5

75.– A 375 K el SO2Cl2(g) se descompone parcialmente según el siguiente equilibrio: SO2Cl2(g) SO2(g) + Cl2(g) , Kp = 2,4 (a 375 K). Se introducen 0,050 moles de SO2Cl2(g) en un recipiente cerrado de 2,0 L de capacidad, en el que previamente se ha hecho el vacío, y se calienta a 375 K. Cuando se alcanza el equilibrio a dicha temperatura, calcule:

a) la presión parcial de cada uno de los gases presentes en el equilibrio a 375 K; b) el grado de disociación del SO2Cl2(g) a la citada temperatura.

DDaattooss:: R = 0,082 atm L mol–1 K–1

Solución: a) Empezamos calculando la presión inicial del reactivo:

𝑝SO2Cl20=

𝑛 𝑅 𝑇𝑉 =

0,050 mol de SO2Cl2 · 0,082 atm L mol–1 K–1 · 375 K2,0 L = 0,77 atm.

Aplicamos la expresión del equilibrio, trabajando con presiones: SO2Cl2(g) SO2(g) + Cl2(g)

presiones iniciales 0,77 atm 0 0 presión de moles gastados px 0 0 presión de moles formados 0 px px

presión en el equilibrio 0,77 atm − px = p0 − px px px Utilizando la expresión de la constante Kp:

𝐾𝑝 = 𝑝SO2 𝑝Cl2

𝑝SO2Cl2

=𝑝𝑥 · 𝑝𝑥

𝑝0 − 𝑝𝑥⇒ 𝑝𝑥

2+ 𝐾𝑝 𝑝𝑥 − 𝐾𝑝 𝑝0 ⇒ 𝑝𝑥 =−𝐾𝑝 ± �𝐾𝑝

2 + 4 𝐾𝑝 𝑝0

2

𝑝𝑥(+) = −2,4 atm ± �(2,4 atm)2 + 4 · 2,4 atm · 0,77 atm

2 = 0,61 atm.

Cogemos sólo la solución con el signo más porque la otra da presiones negativas. SO2Cl2(g) SO2(g) + Cl2(g)

presión en el equilibrio 0,16 atm 0,61 atm 0,61 atm b) El grado de disociación se calcula teniendo en cuenta que:

𝑥 = c 𝛼 ⇒ 𝛼 =xc =

𝑝𝑥𝑅 𝑇𝑝0

𝑅 𝑇=

𝑝𝑥

𝑝0 =

0,61 atm0,77 atm = 0,79 = 79 %.

Page 23: Solución: a) Verdadera. El estado gaseoso es un estado ... · Solución: Aplicando la ecuación de la Energía libre de Gibbs, ∆G = ∆H – T ∆S, por ser la reacción exotérmica,

PROBLEMAS DE QUÍMICA 2º BACHILLERATO − Termodinámica y equilibrio − 26/01/2013 Pág. 23

Licencia Creative Commons 3.0. Autor: Antonio José Vasco Merino

5

76.– A 423,15 K, la oxidación del cloruro de hidrógeno se produce según la reacción siguiente: 4 HCl(g) + O2(g) 2 Cl2(g) + 2 H2O(g). En un recipiente de 2,00 L de capacidad, introducimos 3,60 mol de HCl y 2,00 mol de O2. Sabiendo que el número de moles de O2 en el equilibrio es 1,40, calcule: a) el número de moles del resto de gases en el equilibrio; b) el valor de Kc; c) la presión total en el interior del recipiente una vez alcanzado el equilibrio.

DDaattooss:: R = 0,082 atm L mol–1 K–1

Solución: a) Calculamos las concentraciones inicial y en el equilibrio del SO3:

[HCl]0 =𝑛𝑉 =

3,60 mol de HCl2,00 L = 1,80 M ; [O2]0 =

𝑛𝑉 =

2,00 mol de O2

2,00 L = 1,00 M

[O2]eq =𝑛′𝑉 =

1,40 mol de O2

2,00 L = 0,70 M

Aplicando la expresión del equilibrio: 4 HCl(g) + O2(g) 2 Cl2(g) + 2 H2O(g)

Conc. iniciales 1,80 M 1,00 M 0 0 gastado 4x x 0 0 formado 0 0 2x 2x

en el equilibrio 1,80 M − 4x 1,00 M − x = 0,70 M 2x 2x De la cantidad de oxígeno en el equilibrio deducimos que x es 0,30 M, por lo que:

4 HCl(g) + O2(g) 2 Cl2(g) + 2 H2O(g) en el equilibrio 0,60 M 0,70 M 0,60 M 0,60 M

moles en equilibrio 1,20 mol 1,4 mol 1,20 mol 1,20 mol b) La expresión de la constante de concentraciones será, aplicando los valores obtenidos:

𝐾𝑐= [Cl2]2 [H2O]2

[HCl]4[O2] = (0,60 mol L–1)2· (0,60 mol L–1)2

(0,60 mol L–1)4 · 0,70 mol L–1 = 1,4 L mol–1.

c) La concentración total en el equilibrio de todas las sustancias que intervienen es: cT = 0,60 mol L–1 + 0,70 mol L–1 + 0,60 mol L–1 + 0,60 mol L–1 = 2,50 mol L–1. La presión total se obtiene aplicando la ecuación de los gases ideales:

𝑝T 𝑉 = 𝑛T 𝑅 𝑇 ; 𝑝T =𝑛T 𝑅 𝑇

𝑉 = 𝑐T 𝑅 𝑇 = 2,50 mol L–1 · 0,082 atm Lmol K · 423,15 K = 86,7 atm.

5

77.– A 425 ºC la Kc del equilibrio: I2(g) + H2(g) 2 HI(g) vale 54,8. a) Indique en qué sentido se desplazará el equilibrio si en un recipiente de 10,00 L se

introducen 12,69 g de I2, 1,01 g de H2 y 25,58 g de HI y se calientan a 425 ºC. b) Calcule las concentraciones de I2, H2 y HI cuando se alcance el equilibrio. c) Calcule el valor de Kp.

DDaattooss:: Masas atómicas (u): I = 126,9 ; H = 1,01 ; R = 0,082 atm L mol–1 K–1

Solución: a) Lo primero que tenemos que hacer es calcular las concentraciones presentes:

[I2] =𝑛𝑉 =

12,69 g de I2· 1 mol de I2253,8 g de I2

10,00 L = 5,000 · 10−3 M

[H2] =

1,01 g de H22,02 g mol−1 de H2

10,00 L = 5,000 · 10−2 M ; [HI] =

25,58 g de HI127,9 g mol−1 de HI

10,00 L = 2,000 · 10−2 M.

Calculamos el valor del cociente de reacción y lo comparamos con la constante:

𝑄 = [HI]2

[I2] [H2] = (2,000 · 10−2 mol L–1) 2

(5,000 · 10−3 mol L–1) · (5,000 · 10−2 mol L–1) = 1,6 < 𝐾𝑐 .

Como el cociente es menor que la constante de equilibrio, el sistema evoluciona hacia la formación del producto. b) Estableciendo el equilibrio:

I2(g) + H2(g) 2 HI(g) conc. iniciales 5,000·10−3 M 5,000·10−2 M 2,000·10−2 M

gastado x x 0 formado 0 0 2x

en el equilibrio 5,000·10−3 M − x 5,000·10−2 M − x 2,000·10−2 M + 2x Hallamos el valor de x aplicando la expresión de la constante:

𝐾𝑐= [HI]2

[I2] [H2] = (2,000 · 10−2 M + 2𝑥) 2

(5,000 · 10−3 mol L–1 − 𝑥) · (5,000 · 10−2 mol L–1 − 𝑥) = 54,8

50,8 𝑥2 − 3,014 mol L–1𝑥 + 1,317 · 10−2 mol2 L–2 = 0 ⇒ 𝑥(+) = 5,625 · 10−2 mol L–1 𝑥(−) = 4,654 · 10−3 mol L–1 .

Tomamos como válida la solución x(−) ya que la otra daría lugar a concentraciones negativas. Por tanto las concentraciones en el equilibrio son: [I2] = 3,46·10−4 mol L–1 ; [H2] = 4,535·10−2 mol L–1 ; [HI] = 2,931·10−2 mol L–1. c) Aplicando la fórmula que relaciona Kp y Kc: Kp = Kc (R T)∆n = Kc (R T)0 = Kc = 54,8.

Page 24: Solución: a) Verdadera. El estado gaseoso es un estado ... · Solución: Aplicando la ecuación de la Energía libre de Gibbs, ∆G = ∆H – T ∆S, por ser la reacción exotérmica,

PROBLEMAS DE QUÍMICA 2º BACHILLERATO − Termodinámica y equilibrio − 26/01/2013 Pág. 24

Licencia Creative Commons 3.0. Autor: Antonio José Vasco Merino

5

78.– A 532 K se introducen 0,10 moles de PCl5 en un recipiente X de 1,2 L y 0,10 moles en otro recipiente Y. Se establece el equilibrio PCl5 PCl3 + Cl2, y la cantidad de PCl5 se reduce un 50 % en el recipiente X y un 90 % en el recipiente Y. Todas las especies se encuentran en fase gaseosa. Calcule:

a) la presión en el equilibrio en el recipiente X; b) la constante de equilibrio Kc; c) el volumen del recipiente Y; d) la presión en el equilibrio en el recipiente Y.

DDaattooss:: R = 0,082 atm L mol–1 K–1

Solución: a) Calculamos las concentraciones iniciales del PCl5 en cada uno de los recipientes:

[PCl5]0X =𝑛𝑉 =

0,10 mol de PCl5

1,2 L = 0,083 M ; [PCl5]0Y =𝑛𝑉 =

0,10 mol de PCl5

V = 𝑐.

Como el enunciado dice que en el recipiente X se encuentra disociado en un 50 % (α = 0,50), sabemos que la concentración gastada es x = 0,50 n = 0,042 M.

Recipiente X PCl5(g) PCl3(g) + Cl2(g) concentraciones iniciales 0,083 M 0 0

gastado x = 0,042 M 0 0 formado 0 x = 0,042 M x = 0,042 M

en el equilibrio 0,041 M 0,042 M 0,042 M La concentración total en el equilibrio es de: cT = (0,041 M + 0,042 M + 0,042 M) = 0,125 M. La presión total se obtiene aplicando la ecuación de los gases ideales:

𝑝T 𝑉 = 𝑛T 𝑅 𝑇 ; 𝑝T =𝑛T 𝑅 𝑇

𝑉 = 𝑐T 𝑅 𝑇 = 0,125 mol L–1 · 0,082 atm Lmol K · 532 K = 5,5 atm.

b) La expresión de la constante de concentraciones será, aplicando los valores obtenidos:

𝐾𝑐= [PCl3] [Cl2]

[PCl5] = 0,042 mol L–1· 0,042 mol L–1

0,041 mol L–1 = 4,2·10−2 mol L–1.

c) Planteando ahora el mismo equilibrio en el otro recipiente: Recipiente Y PCl5(g) PCl3(g) + Cl2(g)

concentraciones iniciales c 0 0 gastado x = 0,90 c 0 0 formado 0 x = 0,90 c x = 0,90 c

en el equilibrio c − 0,90 c = 0,1 c 0,90 c 0,90 c Aplicando la expresión de la constante de concentraciones será, utilizando los valores obtenidos:

𝐾𝑐= [PCl3] [Cl2]

[PCl5] = 0,9 𝑐 · 0,9 𝑐

0,1 𝑐 = 8,1 𝑐 = 8,1 ·0,10 mol

𝑉 = 4,2·10−2 mol L–1.

𝑉 =0,81 mol

4,2·10−2 mol L–1 = 19 L.

d) El número de moles totales es: nT = 0,1 n + 0,9 n + 0,9 n = 1,9 n = 0,19 mol. La presión total se obtiene aplicando la ecuación de los gases ideales:

𝑝T 𝑉 = 𝑛T 𝑅 𝑇 ; 𝑝T =𝑛T 𝑅 𝑇

𝑉 =0,19 mol · 0,082 atm L mol–1 K–1 · 532 K

19 L = 0,43 atm.

5

79.– A 670 K, un recipiente de 2,0 L contiene una mezcla gaseosa en equilibrio de 0,0030 moles de hidrógeno, 0,0030 moles de iodo y 0,0240 moles de ioduro de hidrógeno, según la reacción: H2(g) + I2(g) 2 HI(g). En estas condiciones, calcule:

a) el valor de Kc y Kp; b) la presión total en el recipiente y las presiones parciales de los gases en la mezcla.

DDaattooss:: R = 0,082 atm L mol–1 K–1 = 8,31 J mol–1 K–1

Solución: a) Primero calculamos las concentraciones en el equilibrio de todos los compuestos:

[H2]eq =𝑛H2

𝑉 =0,0030 mol

2,0 L = 0,0015 mol L–1 ; [I2]eq =𝑛I2

𝑉 =0,0030 mol

2,0 L = 0,0015 mol L–1

[HI]eq =𝑛HI

𝑉 =0,0240 mol

2,0 L = 0,0120 mol L–1.

La expresión de la constante de concentraciones es:

𝐾𝑐 = [HI]2

[H2] [I2] =(0,0120 mol L–1) 2

0,0015 mol L–1 · 0,0015 mol L–1 = 64.

Aplicando la relación entre las constantes (con ∆n = 0): Kp = Kc (R T)∆n = Kc = 64. b) La concentración total en el equilibrio de todas las sustancias que intervienen es: cT = 0,0015 mol L–1 + 0,0015 mol L–1 + 0,0120 mol L–1 = 0,0150 mol L–1. La presión total, y las parciales, se obtienen aplicando la ecuación de los gases ideales:

𝑝T 𝑉 = 𝑛T 𝑅 𝑇 ; 𝑝T =𝑛T 𝑅 𝑇

𝑉 = 𝑐T 𝑅 𝑇 = 0,0150 mol L–1 · 0,082 atm Lmol K · 670 K = 0,82 atm

𝑝H2 = 𝑝I2 = 𝑐 𝑅 𝑇 = 0,0015 mol L–1 · 0,082 atm L mol–1 K–1 · 670 K = 0,082 atm𝑝HI = 𝑐 𝑅 𝑇 = 0,0120 mol L–1 · 0,082 atm L mol–1 K–1 · 670 K = 0,66 atm.

Page 25: Solución: a) Verdadera. El estado gaseoso es un estado ... · Solución: Aplicando la ecuación de la Energía libre de Gibbs, ∆G = ∆H – T ∆S, por ser la reacción exotérmica,

PROBLEMAS DE QUÍMICA 2º BACHILLERATO − Termodinámica y equilibrio − 26/01/2013 Pág. 25

Licencia Creative Commons 3.0. Autor: Antonio José Vasco Merino

5

80.– A cierta temperatura el valor de Kc es 783 para el equilibrio: 3 H2(g) + N2(g) 2 NH3(g). A la misma temperatura y de forma razonada: a) calcule Kc para el equilibrio 2 NH3(g) 3 H2(g) + N2(g); b) calcule Kc para el equilibrio 3/2 H2(g) + ½ N2(g) NH3(g); c) explique qué ocurrirá en los equilibrios anteriores si repentinamente aumentase la presión; d) explique qué ocurrirá en los equilibrios anteriores si se añade un catalizador.

Solución: a) La constante de equilibrio será:

𝐾′𝑐 = [H2]3 [N2]

[NH3]2 =1

[NH3]2

[H2]3 [N2]

=1

𝐾𝑐=

1783 = 1,28 · 10−3.

b) La constante de equilibrio de esta otra reacción será:

𝐾′′𝑐 = [NH3]

[H2]32� [N2]1

2�= �

[NH3]2

[H2]3 [N2] = �𝐾𝑐 ≅ 28.

c) Un aumento de presión hará que aumente más rápidamente el lugar del cociente de la expresión de la constante de equilibrio en el que haya más moles estequiométricos gaseosos, que en ambos casos es el lado donde se encuentran el nitrógeno y el hidrógeno. Por lo tanto el sistema evolucionará para conmpensar este exceso de presión (para reestablecer el equilibrio) con lo que el sistema, en ambos casos, evolucionará hacia la formación de NH3. d) Un catalizador aumenta las velocidades tanto de la reacción directa como de la inversa, y en la misma medida, por lo que ambos equilibrios permanecerán invariables.

5

81.– A cierta temperatura, el valor de la constante Kc para el equilibrio: 2 HCl(g) H2(g) + Cl2(g) es 0,82. Si la reacción se inicia en un recipiente de 5,0 litros, introduciendo en él 15,0 g de cloruro de hidrógeno (HCl), calcule: a) el grado de disociación del cloruro de hidrógeno; b) la concentración de cada uno de los gases presentes en el equilibrio.

DDaattooss:: Masas atómicas: Mat (g mol–1): H = 1 ; Cl = 35,5

Solución: a) Calculamos la concentración inicial del HCl:

[HCl]0 =𝑛𝑉 =

15,0 g de HCl · 1 mol de HCl36,5 g de HCl

5,0 L = 0,082 M.

Aplicando la expresión del equilibrio (teniendo en cuenta que 2 x = c α): 2 HCl(g) H2(g) + Cl2(g)

Conc. iniciales c = 0,082 M 0 0 gastado 2x = c α 0 0 formado 0 x = c α / 2 x = c α / 2

en el equilibrio c − c α c α / 2 c α / 2 La expresión de la constante de concentraciones será, aplicando los valores obtenidos:

𝐾𝑐= [H2] [Cl2]

[HCl]2 = 𝑐 𝛼2 · 𝑐 𝛼

2(𝑐 − 𝑐 𝛼)2 =

�𝑐 𝛼2 �

2

(𝑐 − 𝑐 𝛼)2 ⇒ �𝐾𝑐 = 𝑐 𝛼2

𝑐 − 𝑐 𝛼 = 𝛼

2 (1 − 𝛼)

𝛼 =2 �𝐾𝑐

1 + 2 �𝐾𝑐 =

2 · √0,821 + 2 · √0,82

= 0,64 = 64 %.

b) Las concentraciones de las especies en el equilibrio son: 2 HCl(g) H2(g) + Cl2(g)

en el equilibrio 0,030 mol L–1 0,026 mol L–1 0,026 mol L–1

5

82.– A partir de la reacción: 4 NH3(g) + 5 O2(g) 4 NO(g) + 6 H2O(g), a) escriba las expresiones de las constantes Kc y Kp de la reacción; b) establezca la relación entre los valores de Kc y Kp en esta reacción; c) razone cómo influiría en el equilibrio un aumento de la presión. d) Si se aumentase la concentración de O2, explique en qué sentido se desplaza el equilibrio.

¿Se modificaría la constante de equilibrio?

Solución: a) Las constantes son:

𝐾𝑐 = [NO]4 [H2O]6

[NH3]4 [O2]5 ; 𝐾𝑝 = pNO

4 pH2O6

pNH34 pO2

5 .

b) Aplicando p = c R T, y transformando una en otra obtenemos la relación que es:

𝐾𝑝 = pNO

4 · pH2O6

pNH34 · pO2

5 =([NO] 𝑅 𝑇)4([H2O] 𝑅 𝑇)6

([NH3] 𝑅 𝑇)4([O2] 𝑅 𝑇)5 =[NO]4 [H2O]6

[NH3]4 [O2]5 (𝑅 𝑇)∆𝑛 = 𝐾𝑐 (𝑅 𝑇)1 = 𝐾𝑐 𝑅 𝑇.

c) Al aumentar la presión, el sistema tiende a eliminar el exceso de presión, por lo que se desplazará hacia el lado de la reacción que contenga menos moles estequiométricos gaseosos, que es el lado de los reactivos, por lo que la reacción se desplaza hacia la izquierda. d) Al aumentar la concentración de oxígeno, el sistema tiende a eliminar el exceso formado, por lo que se desplazará hacia el lado de la reacción que no forma oxígeno, que es el lado de los productos, por lo que la reacción se desplaza hacia la derecha. En ninguno de los dos apartados últimos se modifica el valor de la constante ya que es constante. Lo que se modifican son los valores de las concentraciones y las presiones ya que sigue cumpliéndose la expresión de las constantes.

Page 26: Solución: a) Verdadera. El estado gaseoso es un estado ... · Solución: Aplicando la ecuación de la Energía libre de Gibbs, ∆G = ∆H – T ∆S, por ser la reacción exotérmica,

PROBLEMAS DE QUÍMICA 2º BACHILLERATO − Termodinámica y equilibrio − 26/01/2013 Pág. 26

Licencia Creative Commons 3.0. Autor: Antonio José Vasco Merino

5

83.– A una temperatura de 200 ºC y a una presión de 1,00 atm, el PCl5 se disocia un 49,5 % en PCl3 y Cl2. Calcule:

a) las constantes Kc y Kp; b) el grado disociación del PCl5 a la misma temperatura pero a 10,0 atmósferas de presión.

Solución: a) Como el enunciado dice que ee encuentra disociado en un 49,5 % (α = 0,495), aplicamos la situación de equilibrio:

PCl5(g) PCl3(g) + Cl2(g) concentraciones

iniciales c 0 0

gastado x = c α 0 0 formado 0 x x

en el equilibrio c − x = c (1 − α) x = c α x = c α La concentración total en el equilibrio es: cT = (c − x) + x + x = c + x = c (1 + α). Por otro lado, y como sabemos la presión total, aplicando la ecuación de los gases ideales:

𝑝T 𝑉 = 𝑛T 𝑅 𝑇 ; 𝑝T = 𝑐T 𝑅 𝑇 𝑐T= 𝑐 (1 + 𝑎) ⇒ 𝑐 =

𝑝T𝑅 𝑇

1 + 𝑎 =

1,0 atm0,082 atm L

mol K · 473 K1,495 = 1,72·10−2 mol L–1.

La expresión de la constante de concentraciones será, aplicando los valores obtenidos:

𝐾𝑐= [PCl3] [Cl2]

[PCl5] = 𝑐 𝛼 · 𝑐 𝛼𝑐(1 − 𝛼) =

𝑐 𝛼2

1 − 𝛼 =1,72·10−2 mol L–1· (0,495)2

1 - 0,495 = 8,37·10−3 mol L–1.

Teniendo en cuenta que el incremento de moles estequiométricos gaseosos vale 1, y aplicando la fórmula que relaciona Kp y Kc: Kp = Kc (R T)∆n = 8,37·10−3 mol L–1 · (0,082 atm L mol–1 K–1 · 473 K)1 = 0,325 atm. b) Aplicando lo obtenido en el apartado anterior:

𝐾𝑐= 𝑐 𝛼2

1 − 𝛼

𝑐 =𝑝T

𝑅 𝑇1 + 𝑎

⇒ 𝐾𝑐=

𝑝T𝑅 𝑇

1 + 𝑎 𝛼2

1 − 𝛼 ⇒ 𝐾𝑐 − 𝐾𝑐𝛼2 =𝑝T

𝑅 𝑇 𝛼2 ⇒ 𝛼 = �𝐾𝑐

𝐾𝑐 + 𝑝T𝑅 𝑇

𝛼 =�

8,37·10−3 mol L–1

8,37·10−3 mol L–1 + 10,0 atm0,082 atm L

mol K · 473 K

= 0,177 = 17,7 %.

5

84.– Al calentar, el dióxido de nitrógeno se disocia en fase gaseosa en monóxido de nitrógeno y oxígeno: a) Formule la reacción química que tiene lugar. b) Escriba Kp para esta reacción. c) Explique el efecto que produce un aumento de presión total sobre el equilibrio. d) Explique cómo se verá afectada la constante de equilibrio al aumentar la temperatura.

Solución: a) La reacción química que tiene lugar es: NO2(g) NO(g) + 1/2 O2(g). Es un equilibrio puesto que tiene constante y le afectan distintas variables. b) Las constantes se pueden expresar como:

𝐾𝑐= [NO] [O2]1

2�

[NO2] ; 𝐾𝑝 = 𝑝NO 𝑝O2

12�

𝑝NO2

.

c) Como hay más moles estequiométricos gaseosos en el lado de los productos, el equilibrio tiende a compensar el aumento de la presión haciendo que el equilibrio la disminuya (Ley de Le Chatelier) por lo que el sistema evoluciona hacia la formación del reactivo, NO2. d) Como la reacción inversa es una oxidación total, normalmente sería exotérmica por lo que la reacción directa es endotérmica. Por el Principio de Le Chatelier el sistema ha de evolucionar intentando eliminar el exceso de temperatura por lo que evolucionará hacia el lado endotérmico por lo que la reacción se desplaza hacia la derecha (formación de productos).

Page 27: Solución: a) Verdadera. El estado gaseoso es un estado ... · Solución: Aplicando la ecuación de la Energía libre de Gibbs, ∆G = ∆H – T ∆S, por ser la reacción exotérmica,

PROBLEMAS DE QUÍMICA 2º BACHILLERATO − Termodinámica y equilibrio − 26/01/2013 Pág. 27

Licencia Creative Commons 3.0. Autor: Antonio José Vasco Merino

5

85.– Calcula la solubilidad del AgCl(s) en g L–1, sabiendo que su producto de solubilidad es 4,0·10−11. DDaattooss:: Masas atómicas: Mat (g mol–1): Cl = 35,5 ; Ag = 107,9

Solución: El equilibrio que tiene lugar es: AgCl(s) ↓ Ag+(ac) + Cl–(ac)

Conc. iniciales 0 0 gastado 0 0 formado s s

en el equilibrio s s Aplicando la expresión del producto de solubilidad:

𝐾p𝑠 = [Ag+] [Cl−] = 𝑠 · 𝑠 = 𝑠2 ⇒ 𝑠 = �𝐾p𝑠 = �4,0·10-11 mol2 L−2 = 6,3·10-6 mol L-1.

Utilizando factores de conversión:

𝑠 = 6,3·10-6 mol L-1 · 143,4 g de AgCl1 mol de AgCl = 9,1·10-4 g L-1.

5

86.– Considerando el equilibrio existente entre el oxígeno molecular y el ozono, de acuerdo a la reacción 3 O2(g) 2 O3(g), cuya entalpía de reacción ∆Hr = 284 kJ, justifique: a) el efecto que tendría sobre el equilibrio un aumento de la presión del sistema. b) el efecto que tendría sobre la cantidad de ozono en el equilibrio una disminución de la

temperatura. c) el efecto que tendría sobre el equilibrio la adición de un catalizador. d) el efecto que tendría sobre la constante de equilibrio Kp añadir más ozono al sistema.

Solución: a) Como hay más moles estequiométricos gaseosos en el lado de los reactivos, el equilibrio tiende a compensar el aumento de la presión haciendo que el equilibrio la disminuya (Ley de Le Chatelier) por lo que el sistema evoluciona hacia la formación del producto, O3. b) Por ser la entalpía positiva, la reacción directa es endotérmica. Por el Principio de Le Chatelier el sistema ha de evolucionar intentando anular la disminución de temperatura (calor) por lo que evolucionará hacia el lado exotérmico por lo que la reacción se desplaza hacia la izquierda (formación de oxígeno). c) La adición de un catalizador aumenta la velocidad de ambas reacciones, directa e inversa, sin afectar al equilibrio. d) La adición de reactivos y productos no puede variar una constante de equilibrio porque, como su propio nombre indica, es constante y sólo depende de la temperatura.

5

87.– Considerando la reacción 2 SO2(g) + O2(g) 2 SO3(g) razone si las siguientes afirmaciones son verdaderas o falsas. a) Un aumento de la presión conduce a una mayor producción de SO3. b) Una vez alcanzado el equilibrio, dejan de reaccionar las moléculas de SO2 y O2

entre sí. c) El valor de Kp es superior al de Kc, a temperatura ambiente. d) La expresión de la constante de equilibrio en función de las presiones parciales es:

Kp = pSO22 · pO2 / pSO3

2 DDaattooss:: R = 0,082 atm L mol–1 K–1

Solución: a) Verdadera. Como hay más moles estequiométricos gaseosos en el lado de los reactivos, el equilibrio tiende a compensar dicho aumento haciendo que el equilibrio disminuya la presión (Ley de Le Chatelier) por lo que el sistema evoluciona hacia la formación de SO3. b) Falsa. Los equilibrios químicos son dinámicos, esto es, siempre se están formando productos y reactivos. El equilibrio se alcanza cuando estos dos procesos transcurren a la misma velocidad. c) Falsa. Aplicando la fórmula que relaciona Kp y Kc: Kp = Kc (R T)∆n = Kc (0,082 atm L mol–1 K–1 · 300 K)−1 ≈ 0,04 Kc ⇒ Kp < Kc. b) Falsa. La constante de presiones de la reacción directa es:

𝐾𝑝= 𝑝SO3

2

𝑝SO22 𝑝O2

.

5

88.– Considerando la reacción: 2 NO(g) + O2(g) 2 NO2(g) ; ∆H0 = –982 kJ, ¿qué efectos tendrán sobre el equilibrio las siguientes acciones?

a) Se añade oxígeno. b) Se sube la temperatura. c) Se aumenta la presión. d) Se añade un catalizador.

Solución: a) Al aumentar la concentración de uno de los reactivos, el cociente de reacción se hace menor, por lo que el sistema evoluciona incrementando los valores del numerador y disminuyendo los del denominador para volver a alcanzar el equilibrio. Por tanto evoluciona hacia la formación de productos (hacia la derecha). Se puede deducir también aplicando directamente el Principio de Le Chatelier. b) Como al aumentar la temperatura un sistema tiende a eliminar ese exceso de energía térmica, se favorece el proceso endotérmico. La reacción, tal y como está planteada, es un proceso exotérmico (∆H < 0). Por lo tanto el equilibrio se desplaza hacia la formación de reactivos (hacia la izquierda). La misma conclusión se puede obtener aplicando el Principio de Le Chatelier. c) Si se aumenta la presión, el sistema evoluciona intentando eliminar ese exceso, lo que consigue desplazándose hacia el lado que tiene menor número de moles estequiométricos gaseosos, que es la formación del producto (hacia la derecha). d) La presencia de un catalizador no influye nunca en el estado de equilibrio, ya que lo único que varía es la velocidad a la que transcurren tanto la reacción directa como la inversa.

Page 28: Solución: a) Verdadera. El estado gaseoso es un estado ... · Solución: Aplicando la ecuación de la Energía libre de Gibbs, ∆G = ∆H – T ∆S, por ser la reacción exotérmica,

PROBLEMAS DE QUÍMICA 2º BACHILLERATO − Termodinámica y equilibrio − 26/01/2013 Pág. 28

Licencia Creative Commons 3.0. Autor: Antonio José Vasco Merino

5

89.– Considere el equilibrio 2 NOBr(g) 2 NO(g) + Br2(g). Razone cómo variará el número de moles de Br2 en el recipiente si: a) se añade NOBr; b) se aumenta el volumen del recipiente; c) se añade NO; d) se pone un catalizador.

Solución: a) Partiendo del valor de la constante y el cociente de reacción:

𝑄 = [NO]2 [Br2]

[NOBr]2 ; 𝐾𝑐= [NO]eq

2 [Br2]eq [NOBr]eq

2

Como se añade un reactivo, el denominador del cociente de reacción se hace mayor, por lo que el valor de Q con respecto a Kc es menor. Para poder reestablecer el equilibrio, el sistema tiene que aumentar el numerador de Q al mismo tiempo que disminuye el valor del denominador por lo que la reacción se desplaza hacia la formación de productos (hacia la derecha). Se podría haber llegado a la misma conclusión utilizando el Principio de Le Chatelier. Aumenta la cantidad de Br2. b) La concentración de todas las especies disminuye al no cambiar el número de moles pero sí el volumen. Por haber más moles estequiométricos gaseosos en el lado de los productos, el sistema se desplaza en ese sentido para compensar la disminución de las concentraciones. Aumenta la cantidad de Br2. c) Sería el efecto contrario al del primer apartado por lo que la cantidad de Br2 disminuiría. d) Aumenta la velocidad tanto de la reacción directa como de la inversa sin que varíe el equilibrio. Por tanto no influye en la cantidad de Br2 presente en el equilibrio.

5

90.– Cuando se trata agua líquida con exceso de azufre sólido en un recipiente cerrado, a 25 ºC, se obtienen los gases sulfuro de hidrógeno y dióxido de azufre.

a) Formule el equilibrio que se establece entre reactivos y productos. b) Escriba las expresiones de Kc y Kp. c) Indique cómo afecta al equilibrio un aumento de presión. d) Indique el signo de la variación de entropía del proceso.

Solución: a) El equilibrio que tiene lugar es: 2 H2O(ℓ) + 3 S(s) 2 H2S(g) + SO2(g). b) Las constantes, teniendo en cuenta que la concentración de los sólidos y líquidos es constante y no deben aparecer en la expresión de la constante:

𝐾𝑐 = [H2S]2 [SO2] ; 𝐾𝑝 = 𝑝H2S2 𝑝SO2.

c) Como hay más moles estequiométricos gaseosos en el lado de los productos, un aumento de presión hace que el sistema evolucione disminuyendo la presión por lo que se desplaza hacia la formación de reactivos (hacia la izquierda). d) Como la entropía representa el desorden de un sistema y este tiene claramente un aumento de ese desorden, puesto que solo hay gases entre los productos que implica un desorden claramente mayor, el valor de ∆S ha de ser positivo.

5

91.– Dada la reacción endotérmica para la obtención de hidrógeno CH4(g) C(s) + 2 H2(g), a) escriba la expresión de la constante de equilibrio Kp; b) justifique cómo afecta un aumento de presión al valor de Kp; c) justifique cómo afecta una disminución de volumen a la cantidad de H2 obtenida; d) justifique cómo afecta un aumento de temperatura a la cantidad de H2 obtenida.

Solución: a) Teniendo en cuenta que el carbono es sólido y no ejerce presión, la constante es:

𝐾𝑝 = pH2

2

𝑝CH4

.

b) La constante de presiones es constante (como toda constante) y solo depende de la temperatura por lo que un aumento de presión hará que el equilibrio se desplace hacia la formación de reactivos pero manteniendo el valor de la constante. c) Como una disminución de volumen conlleva un aumento de la presión, el efecto será el mismo del apartado anterior, por lo que disminuirá la concentración (y por tanto la cantidad) de hidrógeno obtenido. d) Un aumento de temperatura provoca una mayor presencia de energía térmica y la reacción tiende a transcurrir hacia el lado que absorbe dicho exceso por lo que evoluciona hacia la formación de productos, que es el proceso endotérmico (hacia la derecha). Por tanto, aumenta la cantidad de hidrógeno.

5

92.– Dado el siguiente equilibrio: 2 NO(g) + O2(g) 2 NO2(g), con ∆H0 < 0, razone en cada caso cómo se modifica el equilibrio y Kc cuando:

a) se eleva la temperatura; b) se disminuye la presión; c) se añade un catalizador.

Solución: a) Al aumentar la temperatura en un sistema, este tiende a eliminar ese exceso de energía térmica favoreciendo el proceso endotérmico. La reacción, tal y como está planteada, es un proceso exotérmico (∆H < 0). Por lo tanto el equilibrio se desplaza hacia la formación de reactivos (hacia la izquierda). La misma conclusión se puede obtener aplicando el Principio de Le Chatelier. c) Si se disminuye la presión, el sistema evoluciona intentando eliminar ese defecto, lo que consigue desplazándose hacia el lado que tiene mayor número de moles estequiométricos gaseosos, que es la formación de los reactivos (hacia la izquierda). d) La presencia de un catalizador no influye nunca en el estado de equilibrio, ya que lo único que varía es la velocidad a la que transcurren tanto la reacción directa como la inversa.

Page 29: Solución: a) Verdadera. El estado gaseoso es un estado ... · Solución: Aplicando la ecuación de la Energía libre de Gibbs, ∆G = ∆H – T ∆S, por ser la reacción exotérmica,

PROBLEMAS DE QUÍMICA 2º BACHILLERATO − Termodinámica y equilibrio − 26/01/2013 Pág. 29

Licencia Creative Commons 3.0. Autor: Antonio José Vasco Merino

5

93.– El CO2 reacciona a 337 ºC con H2S según: CO2(g) + H2S(g) COS(g) + H2O(g), ∆H0

f = 12,2 kJ mol–1. En un reactor de 2,5 L se introducen 4,4 g de CO2 y suficiente cantidad de H2S para que una vez alcanzado el equilibrio la presión total sea 10 atm y los moles de agua 0,01.

a) Calcule la composición de la mezcla en equilibrio. b) El valor de las constantes Kp y Kc. c) Diga cómo afectaría a:

c.1) Kc un aumento de la temperatura, suponiendo ΔHf independiente de la misma; c.2) la cantidad de agua la adición de CO2; c.3) la cantidad de COS un aumento de la presión.

DDaattooss:: Masas atómicas: C = 12 ; O = 16 ; R = 0,082 atm L mol–1 K–1

Solución: a) Calculamos la concentración inicial del dióxido de carbono y la final de agua:

[CO2]0 =𝑛𝑉 =

4,4 g de CO2 · 1 mol de CO244 g de CO2

2,5 L = 0,040 M ; [H2O]eq = 0,01 mol

2,5 L = 0,0040 M.

Aplicando la expresión del equilibrio (teniendo en cuenta que x vale 0,039 M): CO2(g) + H2S(g) COS(g) + H2O(g)

Conc. iniciales 0,040 M c 0 0 gastado x x 0 0 formado 0 0 x x = 0,0040 M

en el equilibrio 0,036 M c − 0,0040 M x = 0,0040 M x = 0,0040 M La concentración total de todas las sustancias que intervienen es: cT = (0,040 M − x) + (c − x) + x + x = 0,040 M + c = [CO2]0 + c. El valor de c se obtiene de aplicar la expresión de la presión total (ecuación de los gases ideales):

𝑝T 𝑉 = 𝑛T 𝑅 𝑇 ; 𝑝T = 𝑐T 𝑅 𝑇 ⇒ 0,040 M + 𝑐 = 𝑝T

𝑅 𝑇

𝑐 =10 atm

0,082 atm L mol–1 K–1 · 610 K - 0,040 mol L−1 = 0,16 mol L−1.

Por tanto las concentraciones y los moles (multiplicando por el volumen) en el equilibrio serán: en el equilibrio 0,036 M 0,156 M 0,0040 M 0,0040 M

moles en eq 0,090 mol 0,39 mol 0,010 mol 0,010 mol b) La constante de concentraciones será:

𝐾𝑐= [COS] [H2O][CO2] [H2S] =

0,0040 mol L−1 · 0,0040 mol L−1

0,036 mol L−1 · 0,16 mol L−1 = 2,8 · 10−3.

Como no hay incremento de moles, si aplicamos la fórmula que relaciona Kp y Kc: Kp = Kc (R T)∆n = Kc (R T)0 = Kc = 2,8·10−3. c) Aplicando el Principio de L Chatelier: c.1) Como la reacción es endotérmica (∆H > 0) un aumento de la temperatura desplaza el equilibrio hacia el lado que "consume" energía, el de los productos. Sus concentraciones aumentan mientras que las de los reactivos disminuyen, por lo que el valor de la constante se hace mayor. c.2) Un aumento de CO2 desplaza la reacción hacia la formación de productos, por lo que aumenta la concentración y la cantidad de agua presente. c.3) Un aumento de presión no modifica la situación de equilibrio ya que hay el mismo número de moles estequiométricos gaseosos en ambos lados del equilibrio. No varía la cantidad de COS.

Page 30: Solución: a) Verdadera. El estado gaseoso es un estado ... · Solución: Aplicando la ecuación de la Energía libre de Gibbs, ∆G = ∆H – T ∆S, por ser la reacción exotérmica,

PROBLEMAS DE QUÍMICA 2º BACHILLERATO − Termodinámica y equilibrio − 26/01/2013 Pág. 30

Licencia Creative Commons 3.0. Autor: Antonio José Vasco Merino

5

94.– El CO2 reacciona con el H2S a altas temperaturas según la reacción: CO2(g) + H2S(g) COS(g) + H2O(g). Se introducen 4,4 g de CO2 en un recipiente de 2,55 L a 337 °C, y una cantidad suficiente de H2S para que, una vez alcanzado el equilibrio, la presión total sea de 10 atm (1013,1 kPa). Si en la mezcla en equilibrio hay 0,01 moles de agua, calcule:

a) el número de moles de cada una de las especies en el equilibrio; b) el valor de Kc y Kp a esa temperatura.

DDaattooss:: R = 0,082 atm L mol–1 K–1 = 8,31 J K–1 mol–1

Solución: a) Calculamos la concentración inicial del dióxido de carbono y la final de agua:

[CO2]0 =𝑛𝑉 =

4,4 g de CO2 · 1 mol de CO244 g de CO2

2,55 L = 0,039 M ; [H2O]eq = 0,01 mol

2,55 L = 0,0039 M.

Aplicando la expresión del equilibrio (teniendo en cuenta que x vale 0,039 M): CO2(g) + H2S(g) COS(g) + H2O(g)

Conc. iniciales 0,039 M c 0 0 gastado x x 0 0 formado 0 0 x x = 0,0039 M

en el equilibrio 0,035 M c − 0,0039 M x = 0,0039 M x = 0,0039 M La concentración total de todas las sustancias que intervienen es: cT = (0,039 M − x) + (c − x) + x + x = 0,039 M + c = [CO2]0 + c. El valor de c se obtiene de aplicar la expresión de la presión total (ecuación de los gases ideales):

𝑝T 𝑉 = 𝑛T 𝑅 𝑇 ; 𝑝T = 𝑐T 𝑅 𝑇 ⇒ 0,039 M + 𝑐 = 𝑝T

𝑅 𝑇

𝑐 =10 atm

0,082 atm L mol–1 K–1 · 610 K - 0,039 mol L−1 = 0,16 mol L−1.

Por tanto las concentraciones y los moles (multiplicando por el volumen) en el equilibrio serán: en el equilibrio 0,035 M 0,156 M 0,0039 M 0,0039 M

moles en eq 0,090 mol 0,40 mol 0,0010 mol 0,0010 mol b) La constante de concentraciones será:

𝐾𝑐= [COS] [H2O][CO2] [H2S] =

0,0039 mol L−1 · 0,0039 mol L−1

0,035 mol L−1 · 0,156 mol L−1 = 2,8 · 10−3.

Como no hay incremento de moles, si aplicamos la fórmula que relaciona Kp y Kc: Kp = Kc (R T)∆n = Kc (R T)0 = Kc = 2,8·10−3.

5

95.– El dióxido de nitrógeno es uno de los gases que contribuyen a la formación de la lluvia ácida, obteniéndose a partir del proceso: 2 NO(g) + O2(g) 2 NO2(g) ∆H < 0. Explique razonadamente, tres formas distintas de actuar sobre dicho equilibrio que reduzcan la formación del dióxido de nitrógeno.

Solución: El equilibrio se puede modificar, desplazándolo hacia la izquierda: • Añadiendo mayor cantidad o aumentando la concentración del producto (NO2). No es

muy útil ya que precisamente queremos disminuir su presencia. • Eliminando cantidades significativas de algún reactivo (NO o/y O2). • Aumentando la temperatura, ya que favorecemos el proceso endotérmico. • Disminuyendo la presión, ya que el sistema evoluciona hacia el lado donde más moles

estequiométricos gaseosos hay. • Aumentando si es posible el volumen del recipiente, por lo mismo del anterior.

Page 31: Solución: a) Verdadera. El estado gaseoso es un estado ... · Solución: Aplicando la ecuación de la Energía libre de Gibbs, ∆G = ∆H – T ∆S, por ser la reacción exotérmica,

PROBLEMAS DE QUÍMICA 2º BACHILLERATO − Termodinámica y equilibrio − 26/01/2013 Pág. 31

Licencia Creative Commons 3.0. Autor: Antonio José Vasco Merino

5

96.– El equilibrio PCl5(g) PCl3(g) + Cl2(g) se alcanza calentando 3,0 g de pentacloruro de fósforo hasta 300 ºC en un recipiente de medio litro, siendo la presión final de 2,0 atm. Calcule: a) el grado de disociación del pentacloruro de fósforo; b) el valor de Kp a dicha temperatura.

DDaattooss:: R = 0,082 atm L mol–1 K–1 ; Masas atómicas: Mat (g mol–1): P = 31,0 ; Cl = 35,5

Solución: a) Calculamos la concentración inicial del PCl5:

[PCl5]0 =𝑛𝑉 =

3,0 g de PCl5 · 1 mol de PCl5208,5 g de PCl5

0,50 L = 0,029 M.

Aplicando la expresión del equilibrio: PCl5(g) PCl3(g) + Cl2(g)

Conc. iniciales 0,029 M 0 0 gastado x 0 0 formado 0 x x

en el equilibrio 0,029 M − x x x La concentración total de todas las sustancias que intervienen es: cT = (0,029 M − x) + x + x = 0,029 M + x = [PCl5]0 + x. El valor de x se obtiene de aplicar la expresión de la presión total (ecuación de los gases ideales):

𝑝T 𝑉 = 𝑛T 𝑅 𝑇 ; 𝑝T = 𝑐T 𝑅 𝑇 ⇒ 0,029 M + 𝑥 = 𝑝T

𝑅 𝑇

𝑥 =2,0 atm

0,082 atm Lmol K · 573 K

- 0,029 mol L−1 = 0,014 mol L−1.

El grado de disociación se calcula teniendo en cuenta que: x = c α ⇒ α = x / c = 0,014 M / 0,029 M = 0,48 = 48 %. b) Teniendo en cuenta en el equilibrio el valor de x y la relación utilizada anteriormente p = c R T:

PCl5(g) PCl3(g) + Cl2(g) conc. en el equilibrio 0,015 M 0,014 M 0,014 M

presión en el equilibrio 0,70 atm 0,66 atm 0,66 atm Aplicando la expresión de la constante Kp:

𝐾𝑝 = 𝑝PCl3 𝑝Cl2

𝑝PCl5

=0,66 atm · 0,66 atm

0,70 atm = 0,62 atm.

5

97.– El níquel metálico se obtiene a partir de la siguiente reacción: NiO(s) + CO(g) Ni(s) + CO2(g).

a) Indique la expresión de Kp y Kc. b) ¿Coincidirá Kc con Kp para esta reacción? c) ¿En qué sentido se desplazará el equilibrio si se aumenta la presión? d) ¿En qué sentido se desplazará el equilibrio si añadimos más cantidad de NiO sólido?

Solución: a) Dado que el NiO(s) y el Ni(s) son sólidos, su concentración es constante por lo que se pueden incluir en la constante. Por tanto, la expresión de ambas constantes sería:

𝐾𝑝 =𝑝CO2

𝑝CO ; 𝐾𝑐 =

[CO2] [CO] .

b) Como el incremento de moles estequiométricos gaseosos es nulo, el valor de las constantes será el mismo ya que: Kp = Kc (R T)∆n = Kc (R T)0 = Kc. c) En ninguno, ya que al haber el mismo número de moles estequiométricos gaseosos la presión total permanece constante se desplace lo que se desplace el equilibrio. d) En ninguno, ya que la adición de NiO(s) no varía la concentración de éste −por ello, no es un término que aparezca en la constante−.

5

98.– El óxido de mercurio(II) contenido en un recipiente cerrado se descompone a 380 ºC según: 2 HgO(s) 2 Hg(g) + O2(g). Sabiendo que a esa temperatura el valor de Kp es 0,186, calcule: a) las presiones parciales de O2 y de Hg en el equilibrio; b) la presión total en el equilibrio y el valor de Kc a esa temperatura.

DDaattooss:: R = 0,082 atm L mol–1 K–1

Solución: a) Representamos la ecuación de equilibrio en presiones: 2 HgO(s) 2 Hg(g) + O2(g)

presiones iniciales 0 0 presión de moles formados 2p p

presión en el equilibrio 2p p Hallamos el valor de p aplicando la expresión de la constante de presiones:

𝐾𝑝= pHg2 𝑝O2 = (2𝑝)2 𝑝 = 4𝑝3 = 0,186 atm3 ⇒ 𝑝 = �𝐾𝑝

43

= �0,186 atm3

43

= 0,360 atm

2 HgO(s) 2 Hg(g) + O2(g) presión en el equilibrio 0,720 atm 0,360 atm

b) La presión total se obtiene sumando las presiones parciales: pT = pHg + pO2 = 0,720 atm + 0,360 atm = 1,080 atm. Aplicando la fórmula que relaciona Kp y Kc (teniendo en cuenta que ∆n = 3): Kc = Kp (R T)–∆n = 0,186 atm3 · (0,082 atm L mol–1 K–1 · 653 K)–3 = 1,21·10–6 mol3 L–3.

Page 32: Solución: a) Verdadera. El estado gaseoso es un estado ... · Solución: Aplicando la ecuación de la Energía libre de Gibbs, ∆G = ∆H – T ∆S, por ser la reacción exotérmica,

PROBLEMAS DE QUÍMICA 2º BACHILLERATO − Termodinámica y equilibrio − 26/01/2013 Pág. 32

Licencia Creative Commons 3.0. Autor: Antonio José Vasco Merino

5

99.– El producto de solubilidad del Pb(IO3)2 es de 2,5·10−13. ¿Cuál es la solubilidad del Pb(IO3)2 expresada en:

a) moles por litro?; b) gramos por litro?

DDaattooss:: Mat (g mol–1): O = 16 ; I = 127 ; Pb = 207

Solución: a) El equilibrio que tiene lugar es: Pb(IO3)2(s) ↓ Pb2+(ac) + 2 IO3

–(ac) Conc. iniciales 0 0

gastado 0 0 formado s 2s

en el equilibrio s 2s Aplicando la expresión del producto de solubilidad:

𝐾p𝑠 = [Pb2+][IO3−]2 = 𝑠(2𝑠)2 = 4𝑠3 ⇒ 𝑠 = �

𝐾p𝑠

43

= �2,5·10-13 mol3 L−3

43

= 4,0·10-5 mol L-1.

b) Utilizando factores de conversión:

𝑠 = 4,0·10-5 mol L-1 · 557 g de Pb(IO3)2

1 mol de Pb(IO3)2= 2,2·10-2 g L-1.

5

100.– El yoduro de hidrógeno se descompone a 400 ºC de acuerdo con la ecuación: 2 HI(g) H2(g) + I2(g), siendo el valor de Kc = 0,0156. Una muestra de 0,60 moles de HI se introduce en un matraz de 1,0 L y parte del HI se descompone hasta que el sistema alcanza el equilibrio. a) ¿Cuál es la concentración de cada especie en el equilibrio? b) Calcule Kp. c) Calcule la presión total en el equilibrio.

DDaattooss:: R = 0,082 atm L mol–1 K–1

Solución: a) Calculamos la concentración inicial del HI:

[HI]0 =𝑛𝑉 =

0,60 mol de HI1,0 L = 0,60 M.

Aplicando la expresión del equilibrio: 2 HI(g) H2(g) + I2(g)

Conc. iniciales 0,60 M 0 0 gastado 2x 0 0 formado 0 x x

en el equilibrio 0,60 M − 2x x x La expresión de la constante de concentraciones será, aplicando los valores obtenidos:

𝐾𝑐= [H2] [I2]

[HI]2 = 𝑥 · 𝑥

(0,60 mol L–1 − 2𝑥)2 = 0,0156 ⇒ 𝑥

0,60 mol L–1 − 2𝑥 = �0,0156 = 0,125

𝑥 =0,075 mol L–1

1,25 = 0,060 mol L–1.

Las concentraciones de las especies en el equilibrio son: 2 HI(g) H2(g) + I2(g)

en el equilibrio 0,48 M 0,06 M 0,06 M b) Teniendo en cuenta que no hay incremento de moles estequiométricos gaseosos, y aplicando la fórmula que relaciona Kp y Kc: Kp = Kc (R T)∆n = Kc (R T)0 = Kc = 0,0156. c) La concentración total en el equilibrio de todas las sustancias que intervienen es: cT = (0,60 mol L–1 − 2x) + x + x = 0,60 mol L–1. La presión total se obtiene aplicando la ecuación de los gases ideales:

𝑝T 𝑉 = 𝑛T 𝑅 𝑇 ; 𝑝T =𝑛T 𝑅 𝑇

𝑉 = 𝑐T 𝑅 𝑇 = 0,60 mol L–1 · 0,082 atm Lmol K · 673 K = 33 atm.

Page 33: Solución: a) Verdadera. El estado gaseoso es un estado ... · Solución: Aplicando la ecuación de la Energía libre de Gibbs, ∆G = ∆H – T ∆S, por ser la reacción exotérmica,

PROBLEMAS DE QUÍMICA 2º BACHILLERATO − Termodinámica y equilibrio − 26/01/2013 Pág. 33

Licencia Creative Commons 3.0. Autor: Antonio José Vasco Merino

5

101.– En un matraz de 1,41 L, en el que previamente se ha realizado el vacío, se introducen 1,00 g de CO(g), 1,00 g de H2O(g) y 1,00 g de H2(g). La mezcla se calienta a 600 ºC, alcanzándose el equilibrio: CO(g) + H2O(g) CO2(g) + H2(g), Kc = 23,2. Calcule la presión total de la mezcla en el equilibrio. DDaattooss:: Mat (g mol–1): H = 1,0 ; C = 12,0 ; O = 16,0 ; R = 0,082 atm L mol–1 K–1

Solución: a) Calculamos las concentraciones iniciales de cada sustancia:

[CO] =𝑛𝑉 =

1,00 g de CO· 1 mol de CO28,0 g de CO

1,41 L = 0,0253 M

[H2O] = 1,00 g de H2O· 1 mol de H2O

18,0 g de H2O1,41 L = 0,0394 M ; [H2] =

1,00 g de H22,0 g mol−1 de H2

1,41 L = 0,355 M.

Aplicando la expresión del equilibrio: CO(g) + H2O(g) CO2(g) + H2(g)

Conc. iniciales 0,0253 M 0,0394 M 0 0,355 M gastado x x 0 0 formado 0 0 x x

en el equilibrio 0,0253 M − x 0,0394 M − x x 0,355 M + x La concentración total de todas las sustancias que intervienen es: cT = (0,0253 M − x) + (0,0394 M − x) + x + (0,355 M + x) = 0,420 M La presión total se obtiene aplicando la ecuación de los gases ideales:

𝑝T 𝑉 = 𝑛T 𝑅 𝑇 ; 𝑝T = 𝑐T 𝑅 𝑇 = 0,420 mol L−1 · 0,082 atm L mol–1 K–1 · 873 K = 30,1 atm. No se necesita utilizar el dato de la constante, que solo dificultaría la resolución del problema.

5

102.– En un matraz de 1,75 L, en el que previamente se ha realizado el vacío, se introducen 0,10 moles de CO(g) y 1,0 mol de COCl2(g). A continuación, se establece el equilibrio a 668 K: CO(g) + Cl2(g) COCl2(g). Si en el equilibrio la presión parcial del Cl2(g) es 10 atm, calcule:

a) las presiones parciales de CO(g) y de COCl2(g) en el equilibrio; b) los valores de Kp y Kc para la reacción a 668 K.

DDaattooss:: R = 0,082 atm L mol–1 K–1

Solución: a) Calculamos las presiones parciales iniciales:

𝑝CO 𝑉 = 𝑛CO 𝑅 𝑇 ; 𝑝CO =𝑛CO 𝑅 𝑇

𝑉 =0,10 mol L–1·0,082 atm L

mol K ·668 K1,75 L = 3,1 atm

𝑝COCl2 𝑉 = 𝑛COCl2 𝑅 𝑇 ; 𝑝COCl2 =𝑛COCl2 𝑅 𝑇

𝑉 =1,0 mol L–1·0,082 atm L

mol K ·668 K1,75 L = 31 atm.

Hallamos las presiones parciales representando la ecuación de equilibrio en presiones: CO(g) + Cl2(g) COCl2(g)

presiones iniciales 3,1 atm 0 31 atm presión de moles gastados 0 0 p presión de moles formados p p 0

presión en el equilibrio 3,1 atm + p p = 10 atm 31 atm − p 13 atm 10 atm 21 atm

b) La constante de presiones será igual a:

𝐾𝑝= 𝑝COCl2

𝑝CO 𝑝Cl2

= 21 atm

13 atm·10 atm = 0,16 atm−1.

Aplicando la fórmula que relaciona Kp y Kc (teniendo en cuenta que ∆n = −1): Kc = Kp (R T)–∆n = 0,16 atm−1 · (0,082 atm L mol–1 K–1 · 668 K)1 = 8,8 L mol−1.

Page 34: Solución: a) Verdadera. El estado gaseoso es un estado ... · Solución: Aplicando la ecuación de la Energía libre de Gibbs, ∆G = ∆H – T ∆S, por ser la reacción exotérmica,

PROBLEMAS DE QUÍMICA 2º BACHILLERATO − Termodinámica y equilibrio − 26/01/2013 Pág. 34

Licencia Creative Commons 3.0. Autor: Antonio José Vasco Merino

5

103.– En un recipiente cerrado de volumen constante igual a 22,0 L y a la temperatura de 305 K se introduce 1,00 mol de N2O4(g). Este gas se descompone parcialmente según la reacción: N2O4(g) 2 NO2(g), cuya constante de equilibrio Kp, vale 0,249 a dicha temperatura. a) Calcule el valor de la constante de equilibrio, Kc. b) Determine las fracciones molares de los componentes de la mezcla en el equilibrio. c) ¿Cuál es la presión total cuando se ha alcanzado el equilibrio?

DDaattooss:: R = 0,082 atm L mol–1 K–1

Solución: a) Aplicando la fórmula que relaciona Kp y Kc: Kc = Kp (R T)–∆n = 0,249 atm · (0,082 atm L mol–1 K–1 · 305 K)–1 = 1,0·10–2 mol L–1. b) Calculamos la presión inicial del N2O4:

𝑝N2O40𝑉 = 𝑛N2O40

𝑅 𝑇 ; 𝑝T =𝑛N2O40

𝑅 𝑇𝑉 =

1,00 mol · 0,082 atm Lmol K · 305 K

22,0 L = 1,14 atm.

Representamos la ecuación de equilibrio en presiones: N2O4(g) 2 NO2(g)

presiones iniciales 1,14 atm 0 presión de los moles gastados x 0

presión de moles formados 0 2x presión en el equilibrio 1,14 atm − x 2x

La constante de presiones será, por tanto, igual a:

𝐾𝑝= pNO2

2

𝑝N2O4

= (2𝑥) 2

1,14 atm - 𝑥 = 0,249 atm ⇒ 4𝑥2 + 0,249 atm · 𝑥 − 0,284 atm2 = 0

𝑥(+) = 0,237 atm ; 𝑥(−) = −0,299 atm. Tomamos solo la solución positiva.

N2O4(g) 2 NO2(g) presión total en el equilibrio presión en el equilibrio 0,90 atm 0,47 atm 1,37 atm

Las fracciones molares serán:

𝜒N2O4 =𝑛N2O4

𝑛T=

𝑝N2O4

𝑝T=

0,90 atm1,37 atm = 0,66 ; 𝜒NO2 =

𝑛NO2

𝑛T=

𝑝NO2

𝑝T=

0,47 atm1,37 atm = 0,34.

c) Como ya hemos calculado antes, la presión total es: pT = 1,37 atm.

5

104.– En un recipiente cerrado tiene lugar la reacción ½ H2(g) + ½ F2(g) HF(g), con un ∆H0 de –270,9 kJ mol–1. Justifique qué le ocurrirá al equilibrio si se efectúan las modificaciones siguientes. a) Se añade un mol de F2 permaneciendo constantes la temperatura y el volumen del

recipiente. b) Se disminuye el volumen del recipiente. c) Se introduce un mol de helio sin variar la temperatura ni el volumen del recipiente. d) Se eleva la temperatura, manteniendo la presión constante.

Solución: a) Partiendo del valor de la constante y el cociente de reacción:

𝑄 = [HF]

[H2]1/2 [F2]1/2 ; 𝐾𝑐= [HF]eq

[H2]eq1/2 [F2]eq

1/2

Como se añade una cantidad extra de un reactivo, el denominador del cociente de reacción se hace mayor, por lo que el valor de Q con respecto a Kc es menor. Para poder reestablecer el equilibrio, el sistema tiene que aumentar el numerador de Q al mismo tiempo que disminuye el valor del denominador por lo que la reacción se desplaza hacia la formación de productos (hacia la derecha). Se podría haber llegado a la misma conclusión utilizando el Principio de Le Chatelier. b) La concentración de todas las especies aumenta al no cambiar el número de moles pero sí el volumen. Por haber los mismos moles estequiométricos gaseosos en ambos lados del equilibrio, el sistema no evoluciona permaneciendo en equilibrio. c) La adición de helio no varía ni la concentración ni la presión de las sustancias que intervienen en el equilibrio por lo que no se experimenta variación alguna. d) Una primera consideración es que aumenta la velocidad tanto de la reacción directa como de la inversa. Por otro lado, al estar a mayor temperatura, el sistema evoluciona intentando eliminar el exceso de energía térmica presente por lo que se desplazará siguiendo el proceso endotérmico que es la reacción inversa, ya que la variación de entalpía del equilibrio dado es negativa. Aumenta la concentración de los reactivos.

Page 35: Solución: a) Verdadera. El estado gaseoso es un estado ... · Solución: Aplicando la ecuación de la Energía libre de Gibbs, ∆G = ∆H – T ∆S, por ser la reacción exotérmica,

PROBLEMAS DE QUÍMICA 2º BACHILLERATO − Termodinámica y equilibrio − 26/01/2013 Pág. 35

Licencia Creative Commons 3.0. Autor: Antonio José Vasco Merino

5

105.– En un recipiente de 0,40 L se introduce 1,0 mol de N2 y 3,0 mol de H2 a la temperatura de 780 K. Cuando se establece el equilibrio para la reacción N2 + 3 H2 2 NH3, se tiene una mezcla con un 28 % en moles de NH3. Determine: a) el número de moles de cada componente en el equilibrio; b) la presión final del sistema; c) el valor de la constante de equilibrio, Kp.

DDaattooss:: R = 0,082 atm L mol–1 K–1

Solución: a) Primero calculamos la concentración inicial de nitrógeno e hidrógeno.

𝑐N2 =𝑛N2

𝑉 =1,0 mol0,40 L = 2,5 mol L–1 ; 𝑐H2 =

𝑛H2

𝑉 =3,0 mol0,40 L = 7,5 mol L–1.

Planteamos el equilibrio (en mol L–1) N2(g) + 3 H2(g) 2 NH3(g)

concentración inicial 2,5 M 7,5 M 0 gastado x 3x 0 formado 0 0 2x

en el equilibrio 2,5 M − x 7,5 M − 3x 2x La concentración total de todas las sustancias que intervienen es: cT = (2,5 M − x) + (7,5 M − 3x) + 2x = 10 M − 2x. Como el 28 % de la mezcla es NH3 podemos deducir que:

2𝑥10 M − 2𝑥 = 0,28 ⇒ 2𝑥 = 2,8 M − 0,56 𝑥 ⇒ 𝑥 =

2,8 M2,56 ≅ 1,1 M.

Por tanto las concentraciones en el equilibrio son: [N2] = 1,4 mol L–1 ; [H2] = 4,2 mol L–1 ; [NH3] = 2,2 mol L–1. Y el número de moles (n = c V): nN2 = 0,56 mol ; nH2 = 1,68 mol ; nNH3 = 0,88 mol. b) La presión total (final) se obtiene aplicando la ecuación de los gases ideales:

𝑝T 𝑉 = 𝑛T 𝑅 𝑇 ; 𝑝T = 𝑐T 𝑅 𝑇 = 7,8 mol L−1 · 0,082 atm L mol–1 K–1 · 780 K = 5,0·102 atm. c) La expresión de la constante de concentraciones es:

𝐾𝑐 = [NH3]2

[N2] [H2]3 =(2,2 mol L–1) 2

1,4 mol L–1 · (4,2 mol L–1)3 = 4,7·10−2 L2 mol–2.

Kp = Kc (R T)∆n = 1,71 L2 mol–2 · (0,082 atm L mol–1 K–1 · 780 K)–2 = 1,14·10–5 atm–2.

5

106.– En un recipiente de 15 litros se introducen 3,0 mol de compuesto A y 2,0 mol del compuesto B. Cuando se calienta el recipiente a 400 K se establece el siguiente equilibrio: 2 A(g) + B(g) 3 C(g). Sabiendo que cuando se alcanza el equilibrio las presiones parciales de B y C son iguales, calcule:

a) las concentraciones de A, B y C en el equilibrio; b) la presión total en el equilibrio; c) el valor de las constantes de equilibrio Kc y Kp a 400 K.

DDaattooss:: R = 0,082 atm L mol–1 K–1

Solución: a) Calculamos las concentraciones iniciales de cada sustancia:

[A] =𝑛𝑉 =

3,0 mol de A15 L = 0,20 M ; [B] =

𝑛𝑉 =

2,0 mol de A15 L = 0,13 M.

Aplicando la expresión del equilibrio: 2 A(g) + B(g) 3 C(g)

Conc. iniciales 0,20 M 0,13 M 0 gastado 2x x 0 formado 0 0 3x

en el equilibrio 0,20 M − 2x 0,13 M − x 3x Como nos dan como dato que las presiones parciales de B y C son iguales, entonces sus concentraciones también lo son por lo que: 0,13 M − x = 3x ⇒ x = 0,033 M.

2 A(g) + B(g) 3 C(g) en el equilibrio 0,13 M 0,10 M 0,10 M

b) La concentración total de todas las sustancias que intervienen es: cT = (0,20 M − 2x) + (0,13 M − x) + 3x = 0,33 M La presión total se obtiene aplicando la ecuación de los gases ideales:

𝑝T 𝑉 = 𝑛T 𝑅 𝑇 ; 𝑝T = 𝑐T 𝑅 𝑇 = 0,33 mol L−1 · 0,082 atm L mol–1 K–1 · 400 K = 11 atm. c) La constante de concentraciones será:

𝐾𝑐= [C]3

[A]2 [B] = (0,10 mol L−1)3

(0,13 mol L−1)2 · 0,10 mol L−1 = 0,56.

Como no hay incremento de moles, si aplicamos la fórmula que relaciona Kp y Kc: Kp = Kc (R T)∆n = Kc (R T)0 = Kc = 0,56.

Page 36: Solución: a) Verdadera. El estado gaseoso es un estado ... · Solución: Aplicando la ecuación de la Energía libre de Gibbs, ∆G = ∆H – T ∆S, por ser la reacción exotérmica,

PROBLEMAS DE QUÍMICA 2º BACHILLERATO − Termodinámica y equilibrio − 26/01/2013 Pág. 36

Licencia Creative Commons 3.0. Autor: Antonio José Vasco Merino

5

107.– En un recipiente de 2,0 litros de volumen introducimos 2,0 moles de HI. Al elevar la temperatura hasta 900 K se establece el siguiente equilibrio: 2 HI(g) I2(g) + H2(g). Si el valor de la constante de equilibrio, a 900 K, para esta reacción es Kc = 3,8·10−2, calcule:

a) el grado de disociación del HI; b) las concentraciones de las especies en equilibrio; c) el valor de la constante Kp.

Solución: a) Calculamos la concentración inicial del HI:

[HI]0 =𝑛𝑉 =

2,0 mol de HI2,0 L = 1,0 M

Aplicando la expresión del equilibrio (teniendo en cuenta que 2 x = c α): 2 HI(g) I2(g) + H2(g)

Conc. iniciales c = 1,0 M 0 0 gastado 2x = c α 0 0 formado 0 x = c α / 2 x = c α / 2

en el equilibrio c − c α c α / 2 c α / 2 La expresión de la constante de concentraciones será, aplicando los valores obtenidos:

𝐾𝑐= [I2] [H2]

[HI]2 = 𝑐 𝛼2 · 𝑐 𝛼

2(𝑐 − 𝑐 𝛼)2 =

�𝑐 𝛼2 �

2

(𝑐 − 𝑐 𝛼)2 ⇒ �𝐾𝑐 = 𝑐 𝛼2

𝑐 − 𝑐 𝛼 = 𝛼

2 (1 − 𝛼)

𝛼 =2 �𝐾𝑐

1 + 2 �𝐾𝑐 =

2 · �3,8 · 10−2

1 + 2 · �3,8 · 10−2= 0,28 = 28 %.

b) Las concentraciones de las especies en el equilibrio son: 2 HI(g) I2(g) + H2(g)

en el equilibrio 0,72 M 0,14 M 0,14 M c) Teniendo en cuenta que no hay incremento de moles estequiométricos gaseosos, y aplicando la fórmula que relaciona Kp y Kc: Kp = Kc (R T)∆n = Kc (R T)0 = Kc = 3,8·10−2.

5

108.– En un recipiente de 25 L se introducen dos moles de hidrógeno, un mol de nitrógeno y 3,2 moles de amoniaco. Cuando se alcanza el equilibrio a 400 °C, el número de moles de amoniaco se ha reducido a 1,8. Para la reacción 3 H2(g) + N2(g) 2 NH3(g), calcule: a) el número de moles de H2 y de N2 en el equilibrio; b) los valores de las constantes de equilibrio Kc y Kp a 400 °C.

DDaattooss:: R = 0,082 atm L mol–1 K–1

Solución: a) Primero calculamos las concentraciones iniciales de todos los compuestos y la concentración en el equilibrio del amoniaco:

[H2]0 =𝑛H2

𝑉 =2,0 mol

25 L = 0,080 mol L–1 ; [N2]0 =𝑛N2

𝑉 =1,0 mol

25 L = 0,040 mol L–1

[NH3]0 =𝑛NH3

𝑉 =3,2 mol

25 L = 0,128 mol L–1 ; [NH3]eq =𝑛′NH3

𝑉 =1,8 mol

25 L = 0,072 mol L–1.

El equilibrio se produce de la siguiente manera: 3 H2(g) + N2(g) 2 NH3(g)

concentración inicial 0,080 M 0,040 M 0,128 M formado 3x x 0 gastado 0 0 2x

en el equilibrio 0,080 M + 3x 0,040 M + x 0,128 M − 2x = 0,072 M x = 0,028 M 0,164 M 0,068 M 0,072 M

moles ⇒ n = c V 4,1 mol 1,7 mol 1,8 mol b) La expresión de la constante de concentraciones es:

𝐾𝑐 = [NH3]2

[H2]3 [N2] =(0,072 mol L–1) 2

(0,164 mol L–1)3 · 0,068 mol L–1 = 17 L2 mol–2.

Kp = Kc (R T)∆n = 17 L2 mol–2 · (0,082 atm L mol–1 K–1 · 673 K)–2 = 5,6·10–3 atm–2.

Page 37: Solución: a) Verdadera. El estado gaseoso es un estado ... · Solución: Aplicando la ecuación de la Energía libre de Gibbs, ∆G = ∆H – T ∆S, por ser la reacción exotérmica,

PROBLEMAS DE QUÍMICA 2º BACHILLERATO − Termodinámica y equilibrio − 26/01/2013 Pág. 37

Licencia Creative Commons 3.0. Autor: Antonio José Vasco Merino

5

109.– En un recipiente de 3,0 L, en el que previamente se ha hecho el vacío, se introducen 0,040 moles de SO3(g) y se calienta a 900 K en presencia de un catalizador de Pt. Una vez alcanzado el equilibrio, se encuentra que hay presentes 0,028 moles de SO3(g) como consecuencia de la reacción que tiene lugar: 2 SO3(g) 2 SO2(g) + O2(g). a) Calcule Kc y Kp a esa temperatura. b) Razone si la cantidad del catalizador puede afectar a las constantes de equilibrio y cuál

sería su papel principal. DDaattooss:: R = 0,082 atm L mol–1 K–1

Solución: a) Calculamos las concentraciones inicial y en el equilibrio del SO3:

[SO3]0 =𝑛𝑉 =

0,040 mol de SO3

3,0 L = 0,013 M ; [SO3]eq =𝑛′𝑉 =

0,028 mol3,0 L = 0,0093 M

Aplicando la expresión del equilibrio: 2 SO3(g) 2 SO2(g) + O2(g)

Conc. iniciales 0,013 M 0 0 gastado 2x 0 0 formado 0 2x x

en el equilibrio 0,013 M − 2x = 0,0093 M x = 0,002 M

2x = 0,004 M x = 0,002 M

La expresión de la constante de concentraciones será, aplicando los valores obtenidos:

𝐾𝑐= [SO2]2 [O2]

[SO3]2 = (0,004 mol L–1)2· (0,002 mol L–1)

(0,0093 mol L–1)2 = 3,7·10−4 mol L–1.

Aplicando la fórmula que relaciona Kp y Kc: Kp = Kc (R T)∆n = 3,7·10−4 mol L–1 · (0,082 atm L mol–1 K–1 · 900 K)1 = 2,7·10–2 atm. b) La presencia de un catalizador, en cualquier cantidad, no influye nunca en el valor de las constantes de equilibrio, ya que lo único que varía es la velocidad a la que transcurren tanto la reacción directa como la reacción inversa.

5

110.– En un recipiente de 3,0 litros se introducen 8,4 g de monóxido de carbono y 5,4 g de agua. La mezcla se calienta a 600 K, estableciéndose el equilibrio: CO(g) + H2O(g) CO2(g) + H2(g), cuya Kc vale 23,2. Calcule, para el equilibrio a 600 K:

a) la concentración de todas las especies en el equilibrio; b) el grado de disociación del monóxido de carbono; c) la presión total de la mezcla.

DDaattooss:: R = 0,082 atm L mol–1 K–1 ; Masas atómicas: Mat (g mol–1): H = 1,0 ; C = 12,0 ; O = 16,0

Solución: a) Calculamos las concentraciones iniciales de cada sustancia:

[CO] =𝑛𝑉 =

8,4 g de CO· 1 mol de CO28,0 g de CO

3,0 L = 0,10 M

[H2O] = 5,4 g de H2O· 1 mol de H2O

18,0 g de H2O3,0 L = 0,10 M.

Aplicando la expresión del equilibrio: CO(g) + H2O(g) CO2(g) + H2(g)

Conc. iniciales 0,10 M 0,10 M 0 0 gastado x x 0 0 formado 0 0 x x

en el equilibrio 0,10 M − x 0,10 M − x x x Hallamos el valor de x aplicando la expresión de la constante:

𝐾𝑐= [CO2] [H2][CO] [H2O] =

𝑥 · 𝑥(0,10 M − 𝑥) · (0,10 M − 𝑥) = 23,2 ⇒

𝑥(0,10 M − 𝑥) = �23,2

𝑥 =0,10 mol L–1 · √23,2

1 + √23,2= 0,083 mol L–1.

Por tanto las concentraciones en el equilibrio son: [CO] = [H2O] = 1,7·10−2 mol L–1 ; [CO2] = [H2] = 8,3·10−2 mol L–1. b) Podemos calcular α mediante la relación:

𝑥 = 𝑐 𝛼 ⇒ 𝛼 =𝑥𝑐 =

0,083 mol L–1

0,10 mol L–1 = 0,83 = 83 %.

c) La concentración total de todas las sustancias que intervienen es: cT = (0,10 M − x) + (0,10 M − x) + x + x = 0,20 M La presión total se obtiene aplicando la ecuación de los gases ideales:

𝑝T 𝑉 = 𝑛T 𝑅 𝑇 ; 𝑝T = 𝑐T 𝑅 𝑇 = 0,20 mol L−1 · 0,082 atm L mol–1 K–1 · 600 K = 9,8 atm.

Page 38: Solución: a) Verdadera. El estado gaseoso es un estado ... · Solución: Aplicando la ecuación de la Energía libre de Gibbs, ∆G = ∆H – T ∆S, por ser la reacción exotérmica,

PROBLEMAS DE QUÍMICA 2º BACHILLERATO − Termodinámica y equilibrio − 26/01/2013 Pág. 38

Licencia Creative Commons 3.0. Autor: Antonio José Vasco Merino

5

111.– En una vasija de 10 L mantenida a 270 ºC y previamente evacuada se introducen 2,5 moles de pentacloruro de fósforo y se cierra herméticamente. La presión en el interior comienza entonces a elevarse debido a la disociación térmica del pentacloruro: PCl5(g) PCl3(g) + Cl2(g). Cuando se alcanza el equilibrio la presión es de 15,6 atm.

a) Calcule el número de moles de cada especie en el equilibrio. b) Obtenga los valores de Kc y Kp.

DDaattooss:: R = 0,082 atm L mol–1 K–1

Solución: a) Calculamos la concentración inicial del PCl5:

[PCl5]0 =𝑛𝑉 =

2,5 mol de PCl5

10 L = 0,25 M. Aplicando la expresión del equilibrio:

PCl5(g) PCl3(g) + Cl2(g) Conc. iniciales 0,25 M 0 0

gastado x 0 0 formado 0 x x

en el equilibrio 0,25 M − x x x La concentración total de todas las sustancias que intervienen es: cT = (0,25 M − x) + x + x = 0,25 M + x = [PCl5]0 + x. El valor de x se obtiene de aplicar la expresión de la presión total (ecuación de los gases ideales):

𝑝T 𝑉 = 𝑛T 𝑅 𝑇 ; 𝑝T = 𝑐T 𝑅 𝑇 ⇒ 0,25 M + 𝑥 = 𝑝T

𝑅 𝑇

𝑥 =15,6 atm

0,082 atm Lmol K · 543 K

- 0,25 mol L−1 = 0,10 mol L−1.

PCl5(g) PCl3(g) + Cl2(g) conc. en el equilibrio 0,15 M 0,10 M 0,10 M

neq = ceq V 1,5 mol 1,0 mol 1,0 mol b) La expresión de la constante de concentraciones será, aplicando los valores obtenidos:

𝐾𝑐= [PCl3] [Cl2]

[PCl5] = 0,10 mol L–1· 0,10 mol L–1

0,15 mol L–1 = 6,7·10-2 mol L–1.

Kp = Kc (R T)∆n = 0,067 mol L–1 · (0,082 atm L mol–1 K–1 · 543 K)1 = 3,0 atm.

5

112.– La reacción CO(g) + H2O(g) H2(g) + CO2(g), tiene una constante Kc de 8,25 a 900 °C. En un recipiente de 25 litros, se mezclan 10 moles de CO y 5,0 moles de H2O a 900 °C. Calcule en el equilibrio: a) las concentraciones de todos los compuestos; b) la presión total de la mezcla.

DDaattooss:: R = 0,082 atm L mol–1 K–1

Solución: a) Calculamos las concentraciones iniciales de cada sustancia:

[CO] =𝑛CO

𝑉 =10 mol de CO

25 L = 0,40 M ; [H2O] =𝑛H2O

𝑉 =5,0 mol de H2O

25 L = 0,20 M. Aplicando la expresión del equilibrio:

CO(g) + H2O(g) CO2(g) + H2(g) Conc. iniciales 0,40 M 0,20 M 0 0

gastado x x 0 0 formado 0 0 x x

en el equilibrio 0,40 M − x 0,20 M − x x x Hallamos el valor de x aplicando la expresión de la constante:

𝐾𝑐= [CO2] [H2][CO] [H2O] =

𝑥 · 𝑥(0,40 mol L–1 − 𝑥) · (0,20 mol L–1 − 𝑥) = 8,25

7,25 𝑥2 − 4,95 mol L–1𝑥 + 0,66 mol2 L–2 = 0 ⇒ 𝑥(+) = 0,50 mol L–1 𝑥(−) = 0,18 mol L–1 .

Tomamos sólo el valor x(−) ya que el otro daría lugar a concentraciones negativas. CO(g) + H2O(g) CO2(g) + H2(g)

en el equilibrio 0,22 M 0,02 M 0,18 M 0,18 M b) La concentración total de todas las sustancias que intervienen es: cT = (0,40 M − x) + (0,20 M − x) + x + x = 0,60 M La presión total se obtiene aplicando la ecuación de los gases ideales:

𝑝T 𝑉 = 𝑛T 𝑅 𝑇 ; 𝑝T = 𝑐T 𝑅 𝑇 = 0,60 mol L−1 · 0,082 atm L mol–1 K–1 · 1173 K = 58 atm.

Page 39: Solución: a) Verdadera. El estado gaseoso es un estado ... · Solución: Aplicando la ecuación de la Energía libre de Gibbs, ∆G = ∆H – T ∆S, por ser la reacción exotérmica,

PROBLEMAS DE QUÍMICA 2º BACHILLERATO − Termodinámica y equilibrio − 26/01/2013 Pág. 39

Licencia Creative Commons 3.0. Autor: Antonio José Vasco Merino

5

113.– Las especies químicas NO, O2 y NO2, se encuentran en equilibrio gaseoso a una determinada temperatura, según la siguiente reacción: 2 NO(g) + O2(g) 2 NO2(g) ∆H < 0. Justifique en qué sentido se desplazará el equilibrio cuando: a) se eleva la temperatura; b) se retira parte del O2; c) se añade un catalizador.

Solución: a) Como al aumentar la temperatura un sistema tiende a eliminar ese exceso de energía térmica, se favorece el proceso endotérmico. La reacción, tal y como está planteada, es un proceso exotérmico (∆H < 0). Por lo tanto el equilibrio se desplaza hacia la formación de reactivos (hacia la izquierda). La misma conclusión se puede obtener aplicando el Principio de Le Chatelier. b) Al retirar parte de uno de los reactivos, el cociente de reacción se hace mayor, por lo que el sistema evoluciona incrementando los valores del denominador y disminuyendo los del numerador para volver a alcanzar el equilibrio. Por tanto evoluciona hacia la formación de reactivos (hacia la izquierda). Lo mismo se puede deducir aplicando directamente el Principio de Le Chatelier. c) La presencia de un catalizador no influye nunca en el estado de equilibrio, ya que lo único que varía es la velocidad a la que transcurren tanto la reacción directa como la inversa.

5

114.– Para la obtención de amoniaco, se introduce una mezcla de 15,0 moles de nitrógeno y 15,0 moles de hidrógeno en un reactor de 10,0 litros y la mezcla se calienta a 400 ºC.

a) Sabiendo que al alcanzar el equilibrio se ha transformado el 20,0 % del nitrógeno inicial, calcule el valor de la constante de equilibrio Kc, a 400º C para la reacción: N2(g) + 3 H2(g) 2 NH3(g).

b) Calcule la presión total en el equilibrio. c) Indique cómo variaría el rendimiento de la reacción si se trabajara a una presión superior.

Solución: a) Primero calculamos la concentración inicial de nitrógeno e hidrógeno.

𝑐N2 =𝑛N2

𝑉 =15,0 mol

10,0 L = 1,50 mol L–1 ; 𝑐H2 =𝑛H2

𝑉 =15,0 mol

10,0 L = 1,50 mol L–1.

Como el enunciado dice que ha reaccionado un 20,0 % (α = 0,200), sabemos que los moles gastados son x = 0,200 · 1,50 mol L–1 = 0,300 mol L–1.

N2(g) + 3 H2(g) 2 NH3(g) concentración inicial 1,50 M 1,50 M 0

gastado x = 0,300 M 3x = 0,900 M 0 formado 0 0 2x = 0,600 M

en el equilibrio 1,20 M 0,600 M 0,600 M La expresión de la constante de concentraciones es:

𝐾𝑐 = [NH3]2

[N2] [H2]3 =(0,600 mol L–1) 2

1,20 mol L–1 · (0,600 mol L–1)3 = 1,39 L2 mol–2.

b) La concentración total de todas las sustancias que intervienen es: cT = (1,20 mol L–1) + (0,600 mol L–1) + (0,600 mol L–1) = 2,40 mol L–1. La presión total se obtiene aplicando la ecuación de los gases ideales:

𝑝T 𝑉 = 𝑛T 𝑅 𝑇 ; 𝑝T =𝑛T 𝑅 𝑇

𝑉 = 𝑐T 𝑅 𝑇 = 2,40 mol L–1 · 0,082 atm Lmol K · 673 K = 132 atm.

c) El rendimiento de la reacción aumentaría notablemente, ya que en el lado de los reactivos hay más moles estequiométricos gaseosos, por lo que la reacción se desplaza hacia la derecha, para eliminar el exceso de presión.

Page 40: Solución: a) Verdadera. El estado gaseoso es un estado ... · Solución: Aplicando la ecuación de la Energía libre de Gibbs, ∆G = ∆H – T ∆S, por ser la reacción exotérmica,

PROBLEMAS DE QUÍMICA 2º BACHILLERATO − Termodinámica y equilibrio − 26/01/2013 Pág. 40

Licencia Creative Commons 3.0. Autor: Antonio José Vasco Merino

5

115.– Para la reacción N2(g) + O2(g) 2 NO(g) el valor de la constante de equilibrio, Kc, es 8,8·10–4 a 1930 °C. Si se introducen 2,0 moles de N2 y 1,0 mol de O2 en un recipiente vacío de 2,0 L y se calienta hasta 1930 °C, calcule: a) la concentración de cada una de las especies en equilibrio; b) la presión parcial de cada especie y el valor de la constante de equilibrio Kp.

DDaattooss:: R = 0,082 atm L mol–1 K–1

Solución: a) Lo primero que tenemos que hacer es calcular las concentraciones iniciales:

[N2] =𝑛N2

𝑉 =2,0 mol de N2

2,0 L = 1,0 M ; [O2] =𝑛O2

𝑉 =1,0 mol de O2

2,0 L = 0,50 M.

b) Aplicando los datos que se dan al equilibrio: N2(g) + O2(g) 2 NO(g)

conc. iniciales 1,0 M 0,50 M 0 gastado x x 0 formado 0 0 2x

en el equilibrio 1,0 M − x 0,50 M − x 2x Hallamos el valor de x utilizando la Ley de Acción de Masas (LAM):

𝐾𝑐= [NO]2

[N2] [O2] = (2𝑥)2

(1,0 M − 𝑥) · (0,5 M − 𝑥) = 8,8·10-4≅⇒ 4𝑥2 + 1,32·10-4 𝑥 − 4,4·10-4 = 0

𝑥(+) = 1,05 · 10−2 mol L–1 ; 𝑥(−) = −1,05 · 10−2 mol L–1. Tomamos solo la solución positiva. .

Por tanto las concentraciones en el nuevo equilibrio son: N2(g) + O2(g) 2 NO(g)

en el equilibrio 0,99 M 0,49 M 2,1·10−2 M b) Las presiones parciales se obtienen aplicando la ecuación de los gases ideales:

𝑝N2 𝑉 = 𝑛N2 𝑅 𝑇 ; 𝑝N2 =𝑛N2 𝑅 𝑇

𝑉 = 𝑐N2 𝑅 𝑇 = 0,99 mol L–1 · 0,082 atm Lmol K · 2203 K ≈ 180 atm

𝑝O2 𝑉 = 𝑛O2 𝑅 𝑇 ; 𝑝O2 = 𝑐O2 𝑅 𝑇 = 0,49 mol L–1 · 0,082 atm Lmol K · 2203 K ≈ 90 atm

𝑝NO 𝑉 = 𝑛NO 𝑅 𝑇 ; 𝑝N2 = 𝑐NO 𝑅 𝑇 = 0,021 mol L–1 · 0,082 atm Lmol K · 2203 K ≈ 3,4 atm.

Aplicando la fórmula que relaciona Kp y Kc: Kp = Kc (R T)∆n = Kc = 8,8·10−4.

5

116.– Para la reacción: SbCl5(g) SbCl3(g) + Cl2(g), se sabe que a 182 ºC el valor de Kp = 0,0932. Si se introducen 0,20 moles de SbCl5 en un recipiente de 400 mL y se calienta hasta los 182 ºC estableciéndose el equilibrio anterior,

a) calcule el valor de Kc; b) calcule las concentraciones de las especies presentes en el equilibrio; c) calcule la presión de la mezcla gaseosa.

DDaattooss:: R = 0,082 atm L mol–1 K–1

Solución: a) Aplicando la fórmula que relaciona Kp y Kc: Kc = Kp (R T)−∆n = 0,0932 atm · (0,082 atm L mol–1 K–1 · 455 K)−1 = 2,5·10−3 mol L–1. b) Hallamos la concentración inicial del pentacloruro de antimonio:

[SbCl5]0 =𝑛𝑉 =

0,20 mol de SbCl5

400 mL · 1 L1000 mL

= 0,50 M.

Establecemos el equilibrio que va a tener lugar: SbCl5(g) SbCl3(g) + Cl2(g)

concentraciones iniciales c = 0,50 M 0 0 gastado x 0 0 formado 0 x x

en el equilibrio c − x x x Aplicando la expresión de la constante de concentraciones podemos hallar x:

𝐾𝑐= [SbCl3] [Cl2]

[SbCl5] = 𝑥 · 𝑥

𝑐 − 𝑥 ⇒ 𝑥2 + 𝐾𝑐 𝑥 − 𝐾𝑐 𝑐 = 0 ⇒ 𝑥 =−𝐾𝑐 ± �(𝐾𝑐)2 + 4 𝐾𝑐 𝑐

2

𝑥(+) =−2,5 · 10−3 + �(2,5 · 10−3)2 + 4 (2,5 · 10−3) · 0,50

2 = 3,4·10−2 mol L–1.

Solo consideramos la solución positiva porque no puede existir una concentración negativa. b) Las concentraciones en el equilibrio son:

SbCl5(g) SbCl3(g) + Cl2(g) en el equilibrio 0,47 M 0,034 M 0,034 M

c) La concentración total de todas las sustancias que intervienen es: cT = (0,47 mol L–1) + (0,034 mol L–1) + (0,034 mol L–1) = 0,53 mol L–1. La presión total se obtiene aplicando la ecuación de los gases ideales:

𝑝T 𝑉 = 𝑛T 𝑅 𝑇 ; 𝑝T =𝑛T 𝑅 𝑇

𝑉 = 𝑐T 𝑅 𝑇 = 0,53 mol L–1 · 0,082 atm Lmol K · 455 K = 20 atm.

Page 41: Solución: a) Verdadera. El estado gaseoso es un estado ... · Solución: Aplicando la ecuación de la Energía libre de Gibbs, ∆G = ∆H – T ∆S, por ser la reacción exotérmica,

PROBLEMAS DE QUÍMICA 2º BACHILLERATO − Termodinámica y equilibrio − 26/01/2013 Pág. 41

Licencia Creative Commons 3.0. Autor: Antonio José Vasco Merino

5

117.– Para los siguientes equilibrios: 1º) 2 N2O5(g) 4 NO2(g) + O2(g); 2º) N2(g) + 3 H2(g) 2 NH3(g); 3º) H2CO3(ac) H+(ac) + HCO3

–(ac), a) escriba las expresiones de Kc para los dos primeros y Kp para todos ellos; b) razone qué sucederá en los equilibrios lº y 2º si se aumenta la presión a temperatura

constante.

Solución: a) Las constantes se pueden expresar como:

1ª) 𝐾𝑐= [NO2]4 [O2]

[N2O5]2 ; 𝐾𝑝 = 𝑝NO2

4 𝑝O2 𝑝N2O5

2 ; 2ª) 𝐾𝑐= [NH3]2

[N2] [H2]3 ; 𝐾𝑝 =𝑝NH3

2 𝑝N2 𝑝H2

3 .

El tercero no tiene constante de presiones porque son sustancias en disolución y no gases. b) Un aumento de presión para la primera de las reacciones, y dado que presenta más moles estequiométricos gaseosos en el lado de los productos, hace que el equilibrio tienda a compensar el aumento de la presión haciendo que el equilibrio la disminuya (Ley de Le Chatelier) por lo que el sistema evoluciona hacia la formación del reactivo, N2O5. En la segunda sucede justo lo contrario por lo que aumentará la producción de amoniaco, NH3.

5

118.– Responda, razonadamente, las siguientes cuestiones: a) ¿Qué entiende por solubilidad de un compuesto? b) Deduzca una expresión que relacione la solubilidad y la constante del producto de

solubilidad para una sal tipo AmBn.

Solución: a) Solubilidad de un compuesto es la máxima cantidad de dicho compuesto (expresada en mol L–1 o en g L–1) que puede disolverse completamente en un disolvente (normalmente agua) o en una disolución. Suele variar con la temperatura. b) Aplicando el equilibrio:

AmBn(s) ↓ m An+(ac) + n Bm−(ac) Conc. iniciales 0 0

gastado 0 0 formado m s n s

en el equilibrio m s n s Aplicando la expresión del producto de solubilidad:

𝐾p𝑠 = [An+]m [Bm−]n = (m 𝑠)m (n 𝑠)n = mm nn 𝑠(m+n) ⇒ 𝑠 = �𝐾p𝑠

mm nn

m+n.

5

119.– Se introduce en un recipiente de 3,0 L, en el que previamente se ha hecho el vacío, 0,040 moles de SO3 a 900 K. Una vez alcanzado el equilibrio, se encuentra que hay presentes 0,028 moles de SO3. a) Calcule el valor de Kc para la reacción: 2 SO3(g) 2 SO2(g) + O2(g) a dicha

temperatura. b) Calcule el valor de Kp para dicha disociación.

DDaattooss:: R = 0,082 atm L mol–1 K–1

Solución: a) Calculamos las concentraciones inicial y en el equilibrio del SO3:

[SO3]0 =𝑛𝑉 =

0,040 mol de SO3

3,0 L = 0,013 M ; [SO3]eq =𝑛′𝑉 =

0,028 mol3,0 L = 0,0093 M

Aplicando la expresión del equilibrio: 2 SO3(g) 2 SO2(g) + O2(g)

Conc. iniciales 0,013 M 0 0 gastado 2x 0 0 formado 0 2x x

en el equilibrio 0,013 M − 2x = 0,0093 M x = 0,002 M

2x = 0,004 M x = 0,002 M

La expresión de la constante de concentraciones será, aplicando los valores obtenidos:

𝐾𝑐= [SO2]2 [O2]

[SO3]2 = (0,004 mol L–1)2· (0,002 mol L–1)

(0,0093 mol L–1)2 = 3,7·10−4 mol L–1.

b) Aplicando la fórmula que relaciona Kp y Kc: Kp = Kc (R T)∆n = 3,7·10−4 mol L–1 · (0,082 atm L mol–1 K–1 · 900 K)1 = 2,7·10–2 atm.

5

120.– Se introduce en un recipiente de 3,0 L, en el que previamente se ha hecho el vacío, 0,040 moles de SO3 a 900 K. Una vez alcanzado el equilibrio, se encuentra que hay presentes 0,028 moles de SO3. a) Calcule el valor de Kc para la reacción 2 SO3(g) 2 SO2(g) + O2(g) a dicha

temperatura. b) Calcule la presión parcial de O2 en el equilibrio.

DDaattooss:: R = 0,082 atm L mol–1 K–1

Solución: a) Calculamos las concentraciones inicial y en el equilibrio del SO3:

[SO3]0 =𝑛𝑉 =

0,040 mol de SO3

3,0 L = 0,013 M ; [SO3]eq =𝑛′𝑉 =

0,028 mol3,0 L = 0,0093 M

Aplicando la expresión del equilibrio: 2 SO3(g) 2 SO2(g) + O2(g)

Conc. iniciales 0,013 M 0 0 gastado 2x 0 0 formado 0 2x x

en el equilibrio 0,013 M − 2x = 0,0093 M 2x x x = 0,002 M 0,0093 M 0,004 M 0,002 M

La expresión de la constante de concentraciones será, aplicando los valores obtenidos:

𝐾𝑐= [SO2]2 [O2]

[SO3]2 = (0,004 mol L–1)2· (0,002 mol L–1)

(0,0093 mol L–1)2 = 3,7·10−4 mol L–1.

b) La presión parcial del oxígeno se obtiene aplicando la ecuación de los gases ideales: 𝑝O2 𝑉 = 𝑛O2 𝑅 𝑇 ; 𝑝O2 = 𝑐O2 𝑅 𝑇 = 0,002 mol L−1·0,082 atm L mol–1 K–1·900 K = 0,15 atm.

Page 42: Solución: a) Verdadera. El estado gaseoso es un estado ... · Solución: Aplicando la ecuación de la Energía libre de Gibbs, ∆G = ∆H – T ∆S, por ser la reacción exotérmica,

PROBLEMAS DE QUÍMICA 2º BACHILLERATO − Termodinámica y equilibrio − 26/01/2013 Pág. 42

Licencia Creative Commons 3.0. Autor: Antonio José Vasco Merino

5

121.– Se introduce una mezcla de 0,50 moles de H2 y 0,50 moles de I2 en un recipiente de 1 litro y se calienta a la temperatura de 430 ºC. Calcule: a) las concentraciones de H2, I2 y HI en el equilibrio, sabiendo que, a esa temperatura, la

constante de equilibrio Kc es 54,3 para la reacción: H2(g) + I2(g) 2 HI(g); b) el valor de la constante Kp a la misma temperatura.

Solución: a) Como el volumen es 1 L todos los valores de número de moles coinciden con concentraciones por lo que:

H2(g) + I2(g) 2 HI(g) concentraciones iniciales 0,5 M 0,5 M 0

gastado x x 0 formado 0 0 2x

en el equilibrio 0,5 M − x 0,5 M − x 2x Hallamos el valor de x aplicando la expresión de la constante:

𝐾𝑐= [HI]2

[H2] [I2] = (2𝑥) 2

(0,5 M − 𝑥) · (0,5 M − 𝑥) = 54,3 ; �𝐾𝑐 = 2𝑥

0,5 mol L–1 − 𝑥 = 7,37

𝑥 = 7,37 · 0,5 mol L–1

2 + 7,37 = 0,39 mol L–1.

Por tanto las concentraciones en el equilibrio son: [H2] = [I2] = 0,11 M ; [HI] = 0,39 M. b) Aplicando la fórmula que relaciona Kp y Kc: Kp = Kc (R T)∆n = Kc (R T)0 = Kc = 54,3.

5

122.– Se introduce una mezcla de 0,50 moles de H2 y 0,50 moles de I2 en un recipiente de 1,0 L y se calienta a la temperatura de 430 °C. Calcule:

a) las [I2] y [H2] en equilibrio si Kc para H2(g) + I2(g) 2 HI(g) es 54,3; b) el valor de la Kp a esa temperatura.

Solución: a) Lo primero que tenemos que hacer es calcular las concentraciones iniciales:

[H2] =𝑛𝑉 =

0,50 mol de H2

1,0 L = 0,50 M ; [I2] =𝑛𝑉 =

0,50 mol de I2

1,0 L = 0,50 M

Estableciendo el equilibrio: H2(g) + I2(g) 2 HI(g)

conc. iniciales 0,50 M 0,50 M 0 gastado x x 0 formado 0 0 2x

en el equilibrio 0,50 M − x 0,50 M − x 2x Hallamos el valor de x aplicando la expresión de la constante:

𝐾𝑐= [HI]2

[I2] [H2] = (2𝑥) 2

(0,50 M − 𝑥) · (0,50 M − 𝑥) = 54,3 ⇒ 2𝑥

(0,50 M − 𝑥) = �54,3

𝑥 =0,50 mol L–1 · �54,3

2 + �54,3= 0,39 mol L–1.

Por tanto las concentraciones en el equilibrio son: [H2] = 1,1·10−1 mol L–1 ; [I2] = 1,1·10−1 mol L–1 ; [HI] = 7,8·10−1 mol L–1. b) Aplicando la fórmula que relaciona Kp y Kc: Kp = Kc (R T)∆n = Kc (R T)0 = Kc = 54,3.

Page 43: Solución: a) Verdadera. El estado gaseoso es un estado ... · Solución: Aplicando la ecuación de la Energía libre de Gibbs, ∆G = ∆H – T ∆S, por ser la reacción exotérmica,

PROBLEMAS DE QUÍMICA 2º BACHILLERATO − Termodinámica y equilibrio − 26/01/2013 Pág. 43

Licencia Creative Commons 3.0. Autor: Antonio José Vasco Merino

5

123.– Se introducen 0,20 moles de Br2 en un recipiente de 0,50 L de capacidad a 600 ºC. Una vez establecido el equilibrio Br2(g) 2 Br(g) en estas condiciones, el grado de disociación es 0,80. a) Calcule Kp y Kc. b) Determine las presiones parciales ejercidas por cada componente de la mezcla en

equilibrio. c) Si al aumentar la temperatura aumenta la cantidad de Br(g), indique razonadamente si la

reacción es endotérmica o exotérmica. Así mismo, discuta el efecto que tendría sobre el equilibrio anterior la introducción de gas argón en el reactor si el volumen se mantiene constante.

DDaattooss:: R = 0,082 atm L mol–1 K–1

Solución: a) Calculamos la concentración inicial del Br2:

[Br2]0 =𝑛𝑉 =

0,20 mol de Br2

0,50 L = 0,40 M

Aplicando la expresión del equilibrio (teniendo en cuenta que x = c α) y α = 0,80: Br2(g) 2 Br(g)

Conc. iniciales c = 0,40 M 0 gastado x = c α = 0,32 M 0 formado 0 2x

en el equilibrio c − x = 0,08 M 0,64 M La expresión de la constante de concentraciones será, aplicando los valores obtenidos:

𝐾𝑐= [Br]2 [Br2] =

(0,64 mol L–1) 2

0,08 mol L–1 = 5,1 mol L–1.

Teniendo en cuenta que el incremento de moles estequiométricos gaseosos es 1, y aplicando la fórmula que relaciona Kp y Kc: Kp = Kc (R T)∆n = 5,1 mol L–1 · (0,082 atm L mol–1 K–1·873 K)1 = 3,7·102 atm. b) La presión parcial de cada compuesto se obtiene aplicando la ecuación de los gases ideales:

𝑝Br2 𝑉 = 𝑛Br2 𝑅 𝑇 ; 𝑝Br2 = 𝑐Br2 𝑅 𝑇 = 0,08 mol L–1 · 0,082 atm Lmol K · 873 K = 5,7 atm

𝑝Br 𝑉 = 𝑛Br 𝑅 𝑇 ; 𝑝Br = 𝑐Br 𝑅 𝑇 = 0,64 mol L–1 · 0,082 atm Lmol K · 873 K = 46 atm.

c) Como al aumentar la temperatura el sistema tiende a eliminar ese exceso de energía térmica, se favorece el proceso endotérmico. El enunciado plantea que aumenta la cantidad de Br(g) por lo que se desplaza hacia la formación del producto, por lo que la reacción, tal y como está planteada, es un proceso endotérmico. La introducción de gas argón hace que aumente la presión, pero no varía ni las concentraciones ni las presiones parciales del reactivo y del producto, por lo que no influye en el equilibrio.

5

124.– Sea el equilibrio en fase gaseosa: 2 NO2(g) 2 NO(g) + O2(g). Se coloca 1,00 mol de NO2 en un recipiente de 70 litros y se calienta a 700 K, estableciéndose el equilibrio anterior. Sabiendo que la mezcla en equilibrio contiene 0,46 mol de NO, calcule:

a) el porcentaje de disociación del NO2; b) la fracción molar de todas las especies en el equilibrio; c) la presión total en el recipiente; d) el valor de Kp.

DDaattooss:: R = 0,082 atm L mol–1 K–1

Solución: a) Aplicando el equilibrio a número de moles: 2 NO2(g) 2 NO(g) + O2(g)

moles iniciales 1,00 0 0 gastado 2x 0 0 formado 0 2x x

en el equilibrio 1,00 mol − 2x 2x = 0,46 mol x Podemos calcular α mediante la relación:

2𝑥 = 𝑐 𝛼 ⇒ 𝛼 =2𝑥𝑐 =

0,46 mol1,00 mol = 0,46 = 46 %.

b) Los moles en el equilibrio son: 2 NO2(g) 2 NO(g) + O2(g)

en el equilibrio 0,54 mol 0,46 mol 0,23 mol El número total de moles en el equilibrio es de: nT = 0,54 mol + 0,46 mol + 0,23 mol = 1,23 mol. Por tanto, las fracciones molares serán:

𝜒NO2 =𝑛NO2

𝑛T=

0,54 mol1,23 mol = 0,44 ; 𝜒NO =

𝑛NO

𝑛T=

0,461,23 = 0,37 ; 𝜒O2 =

𝑛O2

𝑛T=

0,23 mol1,23 mol = 0,19.

c) La presión total se calcula aplicando la Ley de los gases ideales:

𝑝T 𝑉 = 𝑛T 𝑅 𝑇 ; 𝑝T =𝑛T 𝑅 𝑇

𝑉 = 1,23 mol · 0,082 atm L

mol K · 700 K70 L = 1,0 atm.

d) Con las fracciones molares podemos calcular las presiones parciales y luego la expresión de la constante:

𝐾𝑝 =𝑝NO

2 𝑝O2

𝑝NO22 =

(𝜒NO 𝑝T)2�𝜒O2 𝑝T�

�𝜒NO2 𝑝T�2 =𝜒NO

2 𝜒O2

𝜒NO22 𝑝T

∆𝑛 = (0,37)2 · 0,19

(0,44)2 · 1,0 atm = 0,13 atm.

Page 44: Solución: a) Verdadera. El estado gaseoso es un estado ... · Solución: Aplicando la ecuación de la Energía libre de Gibbs, ∆G = ∆H – T ∆S, por ser la reacción exotérmica,

PROBLEMAS DE QUÍMICA 2º BACHILLERATO − Termodinámica y equilibrio − 26/01/2013 Pág. 44

Licencia Creative Commons 3.0. Autor: Antonio José Vasco Merino

5

125.– Si se introduce 1,0 mol de trióxido de azufre (SO3) en un recipiente de 1 litro a 25 ºC y 1 atm de presión, se produce el siguiente equilibrio: 2 SO3(g) 2 SO2(g) + O2(g). Calcule: a) la composición de la mezcla resultante una vez alcanzado el equilibrio; b) el grado de disociación del trióxido de azufre; c) el valor de Kp.

DDaattooss:: Kc = 6,75·10–8 (El que esté a 1 atm de presión es un error de enunciado que hace inviable el problema. Lo resolvemos sin tenerlo en cuenta)

Solución: a) Calculamos la concentración inicial del SO3:

[SO3]0 =𝑛𝑉 =

1,0 mol de SO3

1,0 L = 1,0 M

Aplicando la expresión del equilibrio: 2 SO3(g) 2 SO2(g) + O2(g)

Conc. iniciales 1,0 M 0 0 gastado 2x 0 0 formado 0 2x x

en el equilibrio 1,0 M − 2x 2x x La expresión de la constante de concentraciones será (hacemos una aproximación para resolver ya que el valor de la constante es muy bajo y eso implica que x es despreciable frente a c):

𝐾𝑐= [SO2]2 [O2]

[SO3]2 = (2𝑥)2 · 𝑥(1 − 𝑥)2 =

4𝑥3

(1 − 𝑥)2 ≅ 4𝑥3 ⇒ 𝑥 ≅ �𝐾𝑐

43

= �6,75 · 10−8

43

= 0,0026 M.

Por tanto, las concentraciones de las especies en el equilibrio son: 2 SO3(g) 2 SO2(g) + O2(g)

en el equilibrio 1,0 M 2x = 0,0052 M x = 0,0026 M b) El grado de disociación se calcula teniendo en cuenta que: 2x = c α ⇒ α = 2x / c = 0,0052 M / 1,0 M = 0,0052 = 0,52 %. c) Teniendo en cuenta que el incremento de moles estequiométricos gaseosos vale 1, y aplicando la fórmula que relaciona Kp y Kc:

Kp = Kc (R T)∆n = 6,75·10−8 mol L–1 · (0,082 atm L mol–1 K–1 · 298 K)1 = 1,65·10–6 atm.

5

126.– Si tenemos el equilibrio: CO2(g) + H2(g) CO(g) + H2O(g) , ∆H < 0, razone cómo afectará al mismo: a) un aumento de temperatura; b) una reducción a la mitad del volumen del recipiente. c) ¿Qué relación existe entre Kp y Kc en este equilibrio?

Solución: a) Como la reacción es exotérmica, un aumento de temperatura hará que el sistema evolucione para contrarrestar este aumento eliminando energía calorífica por lo que evolucionará hacia el lado endotérmico, esto es hacia la formación de reactivos (Principio de Le Chatelier). b) Una reducción de volumen aumentará la concentración de todas las especies presentes. Como hay el mismo número de moles estequiométricos gaseosos en ambos lados del equilibrio la variación afectará por igual a reactivos y productos por lo que el equilibrio se mantendrá igual (aunque será mayor la velocidad de ambas reacciones ya que las concentraciones son mayores). c) Aplicando la relación: Kp = Kc (R T)∆n = Kc (R T)0 = Kc. Son iguales.

5

127.– Suponga las siguientes reacciones químicas: 2 H2(g) + O2(g) 2 H2O(g) H2(g) + ½ O2(g) H2O(g) 2 H2O(g) 2 H2(g) + O2(g) Si las correspondientes constantes de equilibrio, Kc, son, respectivamente, K1, K2 y K3,

¿cuál será la relación entre K1 y K2? ¿Y entre K2 y K3?

Solución: Partiendo de la expresión de la constante de equilibrio K2:

𝐾2 = [H2O]

[H2] [O2]12�

= �[H2O]2

[H2]2 [O2] = �𝐾1.

Igualmente, si partimos de la expresión de la constante de equilibrio K3:

𝐾3 = [H2]2 [O2]

[H2O]2 =1

[H2O]2

[H2]2 [O2]

=1

� [H2O] [H2] [O2]1

2��

2 =1

𝐾22 = 𝐾2

−2.

Page 45: Solución: a) Verdadera. El estado gaseoso es un estado ... · Solución: Aplicando la ecuación de la Energía libre de Gibbs, ∆G = ∆H – T ∆S, por ser la reacción exotérmica,

PROBLEMAS DE QUÍMICA 2º BACHILLERATO − Termodinámica y equilibrio − 26/01/2013 Pág. 45

Licencia Creative Commons 3.0. Autor: Antonio José Vasco Merino

5

128.– Tras llenar un recipiente de 5,00 L con 1,000 mol de dióxido de azufre y 1,000 mol de oxígeno, se calienta la mezcla a 727 ºC dejando que alcance el equilibrio con formación de trióxido de azufre. Todas las especies son gaseosas y en el equilibrio quedan 0,125 moles de dióxido de azufre. Calcule:

a) los gramos de trióxido de azufre en el equilibrio; b) las constantes Kc y Kp para el equilibrio. c) ¿Qué hay que hacer para formar más trióxido de azufre, bajar o subir la presión del

recipiente? ¿Por qué? DDaattooss:: Masas atómicas: Mat (g mol–1): O = 16 ; S = 32

Solución: a) Calculamos las concentraciones iniciales dadas y la del equilibrio del SO3:

[SO2]0 =𝑛𝑉 =

1,000 mol de SO2

5,00 L = 0,200 M ; [O2]0 =𝑛𝑉 =

1,000 mol de O2

5,0 L = 0,200 M ;

[SO2]eq =𝑛′𝑉 =

0,125 mol de SO2

5,0 L = 0,025 M

Aplicando la expresión del equilibrio: 2 SO2(g) + O2(g) 2 SO3(g)

Conc. iniciales 0,200 M 0,200 M 0 gastado 2x x 0 formado 0 0 2x

en el equilibrio 0,200 − 2x = 0,025 M 0,200 − x 2x x = 0,087 M 0,025 M 0,113 M 0,175 M

La cantidad de SO3 presente en el equilibrio será:

𝑐 =𝑛𝑉 =

𝑚𝑀𝑚𝑉 ⇒ 𝑚 = 𝑐 𝑉 𝑀𝑚 = 0,175 mol L-1 · 5,0 L · 80 g de SO3 mol-1 = 70 g de SO3.

b) La expresión de la constante de concentraciones será, aplicando los valores obtenidos:

𝐾𝑐= [SO3]2

[SO2]2 [O2] = (0,175 mol L–1)2

(0,025 mol L–1)2· (0,113 mol L–1) = 4,3·102 L mol–1.

Kp = Kc (R T)∆n = 4,3·102 L mol–1 · (0,082 atm L mol–1 K–1 · 1000 K)–1 = 5,3 atm–1. c) Como el número de moles estequiométricos del producto (gaseoso) es menor que el de los reactivos, un aumento de presión hará que el sistema intente compensarlo evolucionando hacia el lado de los productos. Por lo tanto, habrá que aumentar la presión (Principio de Le Chatelier).

Page 46: Solución: a) Verdadera. El estado gaseoso es un estado ... · Solución: Aplicando la ecuación de la Energía libre de Gibbs, ∆G = ∆H – T ∆S, por ser la reacción exotérmica,

PROBLEMAS DE QUÍMICA 2º BACHILLERATO − Termodinámica y equilibrio − 26/01/2013 Pág. 46

Licencia Creative Commons 3.0. Autor: Antonio José Vasco Merino

5

129.– Un recipiente de 20,0 L se llena con 56,0 g de nitrógeno y 8,0 g de hidrógeno. La mezcla de gases se calienta a 350 ºC y 10,0 atm de presión. La reacción exotérmica se equilibra formándose algo de amoníaco. Calcule:

a) la composición molar de los gases en el equilibrio; b) las constantes Kc y Kp a 350 ºC. c) Si la presión de la mezcla se eleva a 20 atm sin cambiar la temperatura, ¿cómo cambiará

cualitativamente (aumento, disminución) la concentración de cada gas? Razónelo. DDaattooss:: R = 0,082 atm L mol–1 K–1

Solución: a) Primero calculamos la concentración inicial de nitrógeno e hidrógeno.

𝑐N2 =𝑛N2

𝑉 =56,0 g N2· 1 mol N2

28,0 g N220,0 L = 0,100 mol L–1 ; 𝑐H2 =

8,0 g H2· 1 mol H22,0 g H2

20,0 L = 0,200 mol L–1.

Planteamos el equilibrio que tiene lugar: (en mol L–1) N2(g) + 3 H2(g) 2 NH3(g)

concentración inicial 0,100 M 0,200 M 0 gastado x 3x 0 formado 0 0 2x

en el equilibrio 0,100 M − x 0,200 M − 3x 2x La concentración total de todas las sustancias que intervienen es: cT = (0,100 M − x) + (0,200 M − 3x) + 2x = 0,300 M − 2x. El valor de x se obtiene de aplicar la expresión de la presión total (ecuación de los gases ideales):

𝑝T 𝑉 = 𝑛T 𝑅 𝑇 ; 𝑝T = 𝑐T 𝑅 𝑇 ⇒ 0,300 M − 2𝑥 = 𝑝T

𝑅 𝑇

𝑥 =

0,300 mol L-1- 10,0 atm0,082 atm L

mol K · 623 K2 = 0,052 mol L−1.

Por tanto, en el equilibrio hay: N2(g) + 3 H2(g) 2 NH3(g)

en el equilibrio 0,048 M 0,044 M 0,104 M moles en el equilibrio 0,96 mol 0,88 mol 2,08 mol

b) La expresión de la constante de concentraciones es:

𝐾𝑐 = [NH3]2

[N2] [H2]3 =(0,104 mol L–1) 2

0,048 mol L–1 · (0,044 mol L–1)3 = 2,6·103 L2 mol–2.

Aplicando la relación entre las constantes: Kp = Kc (R T)∆n = 2,6·103 L2 mol–2 · (0,082 atm L mol–1 K–1·623 K)–2 = 1,0 atm–2.

c) Un aumento de la presión implica una reducción de volumen por lo que aumentará la concentración de todas las especies presentes. Como hay menos número de moles estequiométricos gaseosos en el lado de los productos, la variación afectará menos a estos que a los reactivos por lo que el nuevo valor del cociente de reacción es menor que la constante. Así que el numerador tiene que tender a aumentar, mientras que el denominador tiene que tender a disminuir; ambas cosas suceden si el equilibrio se desplaza hacia la formación de los productos (hacia la derecha). Aumenta la concentración de amoniaco (NH3) mientras que disminuye la concentración tanto de hidrógeno (H2) como de nitrógeno (N2). Principio de Le Chatelier.

Page 47: Solución: a) Verdadera. El estado gaseoso es un estado ... · Solución: Aplicando la ecuación de la Energía libre de Gibbs, ∆G = ∆H – T ∆S, por ser la reacción exotérmica,

PROBLEMAS DE QUÍMICA 2º BACHILLERATO − Termodinámica y equilibrio − 26/01/2013 Pág. 47

Licencia Creative Commons 3.0. Autor: Antonio José Vasco Merino

5

130.– Una muestra de 10 gramos de SO2Cl2 gaseoso se descompone a 450 ºC en un recipiente de 3,0 litros, hasta alcanzarse el equilibrio, SO2Cl2(g) SO2(g) + Cl2(g). En el equilibrio a 450 ºC, el SO2Cl2 se encuentra disociado en un 79 %. Calcule: a) los moles de cada una de las especies en el equilibrio; b) el valor de las constantes Kc y Kp a 450 ºC; c) la presión total en el recipiente.

DDaattooss:: Masas atómicas: S = 32 ; O = 16 ; Cl = 35,5 ; R = 0,082 atm L mol–1 K–1

Solución: a) Empezamos calculando la concentración inicial del reactivo:

[SO2Cl2]0 =𝑛𝑉 =

10 g de SO2Cl2 · 1 mol de SO2Cl2135 g de SO2Cl2

3,0 L = 0,025 M.

Como el enunciado dice que se encuentra disociado en un 79 % (α = 0,79), sabemos que los moles gastados son x = 0,025 M · 0,79 = 0,0195 M

SO2Cl2(g) SO2(g) + Cl2(g) concentraciones iniciales 0,025 M 0 0

gastado x = 0,0195 M 0 0 formado 0 x = 0,0195 M x = 0,0195 M

en el equilibrio 0,0055 M 0,0195 M 0,0195 M moles en el equilibrio 0,0165 mol 0,0585 mol 0,0585 mol

b) La expresión de la constante Kc es:

𝐾𝑐= [SO2] [Cl2]

[SO2Cl2] = 0,0195 mol L–1 · 0,0195 mol L–1

0,0055 mol L–1 = 0,069 mol L–1.

Aplicando la fórmula que relaciona Kp y Kc: Kp = Kc (R T)∆n = 0,069 mol L–1 · (0,082 atm L mol–1 K–1 · 723 K)1 = 4,1 atm. c) El número de moles totales de todas las sustancias que intervienen es: nT = (0,0165 mol) + (0,0585 mol) + (0,0585 mol) = 0,1335 mol. La presión total se obtiene aplicando la ecuación de los gases ideales:

𝑝T 𝑉 = 𝑛T 𝑅 𝑇 ; 𝑝T =𝑛T 𝑅 𝑇

𝑉 =0,1335 mol · 0,082 atm L mol–1 K–1 · 723 K

3,0 L = 2,6 atm.

5

131.– Uno de los problemas que tuvieron los químicos del siglo pasado fue la manera de conseguir algún compuesto de nitrógeno a partir del nitrógeno atmosférico, ya que el nitrógeno es un elemento imprescindible en la fabricación de abonos y explosivos. La respuesta la encontró el químico alemán Fritz Haber, que diseñó un proceso para obtener amoníaco a partir del nitrógeno del aire, que sigue la reacción siguiente: N2(g) + 3 H2(g) 2 NH3(g): Kc (a 375 ºC) = 1,2. En un matraz de 3,0 L, a 375 ºC, introducimos 9,0 mol de nitrógeno, 6,0 mol de hidrógeno y 12,0 mol de amoníaco.

a) Justifique por qué el sistema no está en equilibrio y explique razonadamente hacia dónde se desplazará la reacción.

b) Una vez alcanzado el equilibrio, ¿obtendremos más amoníaco si disminuimos el volumen del recipiente? ¿Y si añadimos un catalizador?

Justifique las respuestas.

Solución: Para ver si el sistema se encuentra en equilibrio calculamos el valor del cociente de reacción:

𝑄 = [NH3]2

[N2] [H2]3 =�12,0 mol

3,0 L �2

9,0 mol3,0 L · �6,0 mol

3,0 L �3 = 0,67 L2 mol–2 < 𝐾𝑐.

La expresión del cociente de reacción procede de la comparación entre las velocidades de reacción de las reacciones directa e inversa. Cuando es igual a la constante de equilibrio, las velocidades de ambas reacciones hacen que la cantidad de moles (y con ella la concentración y la presión) de cada compuesto no varíe en el equilibrio. Como es menor que el valor de la constante de concentraciones, el numerador tiene que tender a aumentar, mientras que el denominador tiene que tender a disminuir; ambas cosas suceden si el equilibrio se desplaza hacia la formación de los productos (hacia la derecha). b) Una reducción de volumen aumentará la concentración de todas las especies presentes. Como hay menos número de moles estequiométricos gaseosos en el lado de los productos, la variación afectará menos a estos que a los reactivos por lo que el nuevo valor del cociente de reacción es menor que la constante. Por tanto, vuelve a evolucionar el sistema hacia la formación de productos (hacia la derecha). Obtendremos, por tanto, más amoniaco. El añadir un catalizador no influye en el equilibrio, aunque aumenta al mismo tiempo las velocidades tanto de la reacción directa como de la inversa.